The vagina and the neck of the uterus comprise the: A. Womb. B. Cervix. C. Fundus. D. Birth Canal.
D. Birth Canal.
From what internal female organ is the fetus expelled during delivery? A. Vagina. B. Uterus. C. Cervix. D. Perineum.
B. Uterus.
When the cervix begins to dilate: A. A mucus plug is expelled from the vagina. B. The baby is forced feet-first from the uterus. C. The placenta detaches from the uterine wall. D. The inner lining of the uterus becomes thick.
A. A mucus plug is expelled from the vagina.
Which of the following statements regarding the placenta is MOST correct? A. The placenta allows oxygen, carbon dioxide, and other products to transfer between the mother and fetus but does not allow blood to mix between the mother and fetus. B. The placental barrier consists of two layers of cells and allows the mother's blood that contains high concentrations of oxygen to directly mix with the blood of the fetus. C. The placenta, also referred to as the afterbirth, provides oxygen and nutrients to the fetus and is expelled from the vagina about 30 minutes before the baby is born. D. The placenta allows for the transfer of oxygen and carbon dioxide between the mother and fetus but
A. The placenta allows oxygen, carbon dioxide, and other products to transfer between the mother and fetus but does not allow blood to mix between the mother and fetus.
The umbilical cord: A. Separates from the placenta shortly after birth. B. Carries blood away from the baby via the artery. C. Carries oxygen to the baby via the umbilical vein. D. Contains two veins and one large umbilical artery.
C. Carries oxygen to the baby via the umbilical vein.
The amniotic fluid serves to: A. Transfer oxygen to the fetus. B. Insulate and protect the fetus. C. Remove viruses from the fetus. D. Assist in fetal development.
B. Insulate and protect the fetus.
A mother who is pregnant with her first baby is typically in the first stage of labor for approximately: A. 4 hours. B. 8 hours. C. 10 hours. D. 16 hours.
D. 16 hours.
The onset of labor begins with: A. Thinning of the uterus. B. Full dilation of the cervix. C. Increased fetal movement. D. Contractions of the uterus.
D. Contractions of the uterus.
Which of the following occurs during true labor? A. Uterine contractions decrease in intensity. B. The uterus becomes very soft and movable. C. Uterine contractions become more regular. D. Uterine contractions last about 10 seconds.
C. Uterine contractions become more regular.
A 30-year-old pregnant female is gravida-3 and para-2. This means that she has: A. Given birth to 2 live babies. B. Given birth to 5 live babies. C. Had 3 miscarriages in the past. D. Been pregnant a total of 3 times.
A. Given birth to 2 live babies.
The term primipara refers to a woman who: A. Has never been pregnant. B. Has had only one live birth. C. Is pregnant for the first time. D. Has had more than one live baby.
B. Has had only one live birth.
Braxton-Hicks contractions are characterized by: A. Regular contractions of progressively increasing intensity. B. Alleviation of pain with movement or changing positions. C. Pink or red bloody show in conjunction with the contractions. D. A rupture of the amniotic sac just before the contractions begin.
B. Alleviation of pain with movement or changing positions.
The third stage of labor begins when the: A. Placenta is fully delivered. B. Cervix is completely dilated. C. Umbilical cord has been clamped. D. Baby is expelled from the vagina.
D. Baby is expelled from the vagina.
Signs and symptoms of pregnancy-induced hypertension (preeclampsia) include all of the following, EXCEPT: A. Blood in the urine. B. Visual disturbances. C. Headache and anxiety. D. Swelling of the hands.
A. Blood in the urine.
Eclampsia is MOST accurately defined as: A. High levels of protein in the patient's urine. B. Hypertension in the 20th week of pregnancy. C. Seizures that result from severe hypertension. D. A blood pressure greater than 140/90 mm Hg.
C. Seizures that result from severe hypertension.
The leading cause of maternal death during the first trimester of pregnancy is: A. Massive brain damage secondary to a prolonged seizure. B. Unrecognized or untreated supine hypotensive syndrome. C. Internal bleeding caused by a ruptured ectopic pregnancy. D. Blunt trauma to the abdomen during a motor-vehicle crash.
C. Internal bleeding caused by a ruptured ectopic pregnancy.
Supine hypotensive syndrome occurs when: A. The aorta is kinked secondary to a supine position. B. The gravid uterus compresses the inferior vena cava. C. The superior vena cava is compressed by the uterus. D. Blood pressure falls as a result of hypovolemia.
B. The gravid uterus compresses the inferior vena cava.
A history of pelvic inflammatory disease or tubal ligations increases a woman's risk for: A. Preeclampsia. B. A placenta previa. C. Gestational diabetes. D. An ectopic pregnancy.
D. An ectopic pregnancy.
A placenta previa is MOST accurately defined as: A. Delivery of a portion of the placenta before the baby. B. Premature placental separation from the uterine wall. C. Abnormal development and functioning of the placenta. D. Development of the placenta over the cervical opening.
D. Development of the placenta over the cervical opening.
In contrast to a placenta previa, a placenta abruptio occurs when: A. The placenta prematurely separates from the uterine wall. B. A tear in the placenta causes severe internal hemorrhage. C. The placenta affixes itself to the outer layer of the uterus. D. The placenta develops over and covers the cervical opening.
A. The placenta prematurely separates from the uterine wall.
A pregnant trauma patient may lose a significant amount of blood before showing signs of shock because: A. Pregnant patients can dramatically increase their heart rate. B. Pregnancy causes vasodilation and a lower blood pressure. C. Pregnant patients have an overall increase in blood volume. D. Blood is shunted to the uterus and fetus during major trauma.
C. Pregnant patients have an overall increase in blood volume.
All of the following questions are pertinent to ask a mother when determining whether or not her baby will deliver within the next few minutes, EXCEPT: A. "When are you due?" B. "Is this your first baby?" C. "Have you had a sonogram?" D. "Do you feel the urge to push?"
C. "Have you had a sonogram?"
Which of the following is an indication of imminent birth? A. Rupture of the amniotic sac B. Crowning of the baby's head C. Irregular contractions lasting 10 minutes D. Expulsion of the mucus plug from the vagina.
B. Crowning of the baby's head
When preparing a pregnant patient for delivery, you should position her: A. In a supine position with her legs spread. B. On her left side with the right leg elevated. C. In a sitting position with her hips elevated 12". D. On a firm surface in a semi-Fowler's position.
D. On a firm surface in a semi-Fowler's position.
During delivery, it is MOST important to position your partner at the mother's head because: A. The mother may become nauseated and vomit. B. The mother needs to be apprised of the situation. C. She may need emotional support during the delivery. D. Mothers often need assisted ventilation during delivery.
A. The mother may become nauseated and vomit.
When determining the frequency of contractions, you should: A. Time the contractions from the end of one to the start of the next. B. Time the contractions from the start of one to the end of the next. C. Time the contractions from the end of one to the end of the next. D. Time the contractions from the start of one to the start of the next.
D. Time the contractions from the start of one to the start of the next.
When the mother is experiencing a contraction, you should instruct her to: A. Hold her breath. B. Push for 30 seconds. C. Take quick short breaths. D. Rest and breathe deeply.
C. Take quick short breaths.
A nuchal cord is defined as an umbilical cord that: A. Has separated from the placenta. B. Is wrapped around the baby's neck. C. Is lacerated due to a traumatic delivery. D. Has abnormally developed blood vessels.
B. Is wrapped around the baby's neck.
The presence of thick meconium in the amniotic fluid indicates: A. An expected finding in full-term infants. B. That the baby's airway may be obstructed. C. That the fetus is at least 4 weeks' premature. D. That full newborn resuscitation will be needed.
B. That the baby's airway may be obstructed.
During delivery of the baby's head, you should suction the mouth before the nose because: A. Suctioning the nose first may cause the baby to gasp and aspirate fluid. B. It is easier to suction larger volumes of fluid from the baby's oropharynx. C. Babies are primarily mouth breathers and do not breathe through their nose. D. The mucosa of the nose is fragile and is easily damaged by vigorous suctioning.
A. Suctioning the nose first may cause the baby to gasp and aspirate fluid.
After a baby is born, it is important to: A. Ensure that it is thoroughly dried and warmed. B. Position it so that its head is higher than its body. C. Cool the infant to stimulate effective breathing. D. Immediately clamp and cut the umbilical cord.
A. Ensure that it is thoroughly dried and warmed.
Which of the following is NOT considered an obstetrical emergency? A. Failure of the placenta to deliver after 30 minutes. B. Significant bleeding after delivery of the placenta. C. Return of contractions following delivery of the baby. D. More than 500 mL of blood loss before placental delivery.
C. Return of contractions following delivery of the baby.
A newborn infant will usually begin breathing spontaneously within _____ seconds following birth. A. 3 to 5. B. 5 to 10. C. 15 to 20. D. 30 to 60.
C. 15 to 20.
Which of the following is NOT a component of the APGAR score? A. Pulse. B. Activity. C. Grimace. D. Body size.
D. Body size.
If a baby is born at 7:52 p.m., the second APGAR score should be calculated at: A. 7:53 p.m. B. 7:57 p.m. C. 7:59 p.m. D. 8:00 p.m.
B. 7:57 p.m.
If a newborn has gasping respirations after being dried and suctioned: A. You should suspect hyperthermia and cool the infant. B. Continue to stimulate the infant for another 30 seconds. C. Normal breathing will usually begin within 20 seconds. D. Further stimulation is not likely to improve ventilation.
D. Further stimulation is not likely to improve ventilation.
Which of the following statements regarding a breech presentation is MOST correct? A. A breech presentation occurs when the buttocks are the presenting part. B. There is minimal risk of trauma to the infant with a breech presentation. C. It is impossible to deliver a breech presentation in the prehospital setting. D. Breech deliveries occur rapidly so the EMT-B should deliver at the scene.
A. A breech presentation occurs when the buttocks are the presenting part.
The ONLY indications for placing your gloved fingers in the vagina during delivery are: A. Breech presentation and prolapsed umbilical cord. B. Limb presentation and severe vaginal hemorrhage. C. Vertex presentation and delivery of the placenta. D. Nuchal cord and presentation of an arm or leg.
A. Breech presentation and prolapsed umbilical cord.
An abortion occurs when the fetus and placenta deliver before: A. 20 weeks. B. 24 weeks. C. 26 weeks. D. 28 weeks.
A. 20 weeks.
Which of the following statements regarding twins is MOST correct? A. Twins are typically larger than single infants. B. Identical twins are typically of different gender. C. Most twins are born within 45 minutes of each other. D. Fraternal twins have two cords coming from one placenta.
C. Most twins are born within 45 minutes of each other.
Fetal complications associated with drug- or alcohol-addicted mothers include all of the following, EXCEPT: A. Low birth weight. B. Profound tachycardia. C. Premature delivery. D. Respiratory depression.
B. Profound tachycardia.
An infant is considered to be premature if it: A. Is born before 38 weeks' gestation or weighs less than 6 pounds. B. Weighs less than 5.5 pounds or is born before 37 weeks' gestation. C. Is born before 40 weeks' gestation or weighs less than 7 pounds. D. Weighs less than 5 pounds or is born before 36 weeks' gestation.
D. Weighs less than 5 pounds or is born before 36 weeks' gestation.
General treatment guidelines when caring for a woman with traumatic vaginal bleeding includes: A. Carefully removing impaled objects. B. Transporting to an appropriate facility. C. Packing the vagina with sterile dressings. D. Cleaning external wounds with sterile water.
B. Transporting to an appropriate facility.
Following delivery of a pulseless and apneic infant that has a foul odor, skin sloughing, and diffuse blistering, you should: A. Begin full resuscitation and transport. B. Report the case to the medical examiner. C. Provide emotional support to the mother. D. Dry the infant off to stimulate breathing.
C. Provide emotional support to the mother.
Compared to full-term infants, premature infants are at a higher risk for: A. Infection. B. Breech Births. C. Hyperthermia. D. Explosive Deliveries.
A. Infection.
You have delivered an infant with gasping respirations. After suctioning and physical stimulation, there is no improvement. You should: A. Clamp the umbilical cord. B. Begin chest compressions. C. Administer free-flow oxygen. D. Initiate ventilatory assistance.
D. Initiate ventilatory assistance.
A 33-year-old woman who is 36 weeks' pregnant is experiencing scant vaginal bleeding. During transport, you note that she suddenly becomes diaphoretic, tachycardic, and hypotensive. In addition to administering 100% oxygen, you should: A. Place her in a left lateral recumbent position. B. Position her supine and elevate her legs 12". C. Carefully place sterile gauze into her vagina. D. Begin assisting her ventilations with a BVM.
A. Place her in a left lateral recumbent position.
A 35-year-old woman who is 30 weeks' pregnant presents with a severe headache and swelling in her hands and feet. She is conscious and alert with a blood pressure of 148/94 mm Hg, a pulse of 100 beats/min, and respirations of 24 breaths/min. During transport, you should be MOST concerned with: A. Her prematurely delivering the baby. B. The possibility that she may experience a seizure. C. Using your lights and siren to clear the traffic. D. Her hypertension causing a hemorrhagic stroke.
B. The possibility that she may experience a seizure.
You are assessing a 25-year-old woman who is 39 weeks' pregnant. She is experiencing regular contractions that are approximately 3 minutes apart and states that her amniotic sac broke 2 hours ago. After taking the appropriate BSI precautions, you should: A. Apply 100% oxygen. B. Place her on her left side. C. Assess her for crowning. D. Transport her immediately.
C. Assess her for crowning.
While examining a woman in labor, you see the umbilical cord protruding from the vagina. You should: A. Carefully push the cord back into the vagina. B. Push the infant's head away from the cord. C. Cover the umbilical cord with a dry dressing. D. Gently pull on the cord to facilitate delivery.
B. Push the infant's head away from the cord.
A 26-year-old woman complains of a stabbing pain in the right lower quadrant of her abdomen. She states that her last menstrual period was 7 weeks ago and that she had pelvic inflammatory disease approximately 3 months ago. You should be MOST suspicious that this patient is experiencing: A. An acute appendicitis. B. An intrapelvic infection. C. A spontaneous abortion. D. An ectopic pregnancy.
D. An ectopic pregnancy.
Upon delivery of the baby's head, you note that its face is encased in the unruptured amniotic sac. You should: A. Give the mother 100% oxygen and transport at once. B. Puncture the sac and suction the baby's mouth and nose. C. Leave the amniotic sac intact until arrival at the hospital. D. Note the color of the amniotic fluid before breaking the sac.
B. Puncture the sac and suction the baby's mouth and nose.
Your 22-year-old patient is in active labor. Upon visual inspection, you note that the infant's leg is protruding from the vagina. Appropriate management of this situation includes: A. Placing the mother supine with her head down and pelvis elevated. B. Gently pulling on the infant's leg in an attempt to facilitate delivery. C. Placing the mother in a recumbent position and rapidly transporting. D. Carefully attempting to push the infant's leg off of the umbilical cord.
A. Placing the mother supine with her head down and pelvis elevated.
You have just delivered a baby boy. His body is pink, but his hands and feet are blue. His heart rate is approximately 110 beats/min and his respirations are rapid and irregular. He has a weak cry when stimulated and resists attempts to straighten his legs. His APGAR score is: A. 6. B. 7. C. 8. D. 9.
C. 8.
You have just delivered a full-term infant. His respirations are rapid and irregular and he has a strong cry. What should you do next? A. Allow the mother to hold her baby. B. Clamp and cut the umbilical cord. C. Assess the brachial or umbilical pulse. D. Assist ventilations with a BVM device.
C. Assess the brachial or umbilical pulse.
In contrast to a placenta previa, a placenta abruptio occurs when: A. The placenta prematurely separates from the uterine wall. B. A tear in the placenta causes severe internal hemorrhage. C. The placenta affixes itself to the outer layer of the uterus. D. The placenta develops over and covers the cervical opening.
A. The placenta prematurely separates from the uterine wall.
Following delivery of a pulseless and apneic infant that has a foul odor, skin sloughing, and diffuse blistering, you should: A. Begin full resuscitation and transport. B. Report the case to the medical examiner. C. Provide emotional support to the mother. D. Dry the infant off to stimulate breathing.
C. Provide emotional support to the mother.
General treatment guidelines when caring for a woman with traumatic vaginal bleeding includes: A. Carefully removing impaled objects. B. Transporting to an appropriate facility. C. Packing the vagina with sterile dressings. D. Cleaning external wounds with sterile water.
B. Transporting to an appropriate facility.
Fetal complications associated with drug- or alcohol-addicted mothers include all of the following, EXCEPT: A. Low birth weight. B. Profound tachycardia. C. Premature delivery. D. Respiratory depression.
B. Profound tachycardia.
Which of the following statements regarding twins is MOST correct? A. Twins are typically larger than single infants. B. Identical twins are typically of different gender. C. Most twins are born within 45 minutes of each other. D. Fraternal twins have two cords coming from one placenta.
C. Most twins are born within 45 minutes of each other.
An abortion occurs when the fetus and placenta deliver before: A. 20 weeks. B. 24 weeks. C. 26 weeks. D. 28 weeks.
A. 20 weeks.
When determining the frequency of contractions, you should: A. Time the contractions from the end of one to the start of the next. B. Time the contractions from the start of one to the end of the next. C. Time the contractions from the end of one to the end of the next. D. Time the contractions from the start of one to the start of the next.
D. Time the contractions from the start of one to the start of the next.
When the mother is experiencing a contraction, you should instruct her to: A. Hold her breath. B. Push for 30 seconds. C. Take quick short breaths. D. Rest and breathe deeply.
C. Take quick short breaths.
The ONLY indications for placing your gloved fingers in the vagina during delivery are: A. Breech presentation and prolapsed umbilical cord. B. Limb presentation and severe vaginal hemorrhage. C. Vertex presentation and delivery of the placenta. D. Nuchal cord and presentation of an arm or leg.
A. Breech presentation and prolapsed umbilical cord.
Which of the following statements regarding a breech presentation is MOST correct? A. A breech presentation occurs when the buttocks are the presenting part. B. There is minimal risk of trauma to the infant with a breech presentation. C. It is impossible to deliver a breech presentation in the prehospital setting. D. Breech deliveries occur rapidly so the EMT-B should deliver at the scene.
A. A breech presentation occurs when the buttocks are the presenting part.
If a baby is born at 7:52 p.m., the second APGAR score should be calculated at: A. 7:53 p.m. B. 7:57 p.m. C. 7:59 p.m. D. 8:00 p.m.
B. 7:57 p.m.
A newborn infant will usually begin breathing spontaneously within _____ seconds following birth. A. 3 to 5. B. 5 to 10. C. 15 to 20. D. 30 to 60.
C. 15 to 20.
Which of the following is NOT a component of the APGAR score? A. Pulse. B. Activity. C. Grimace. D. Body size.
D. Body size.
Which of the following is NOT considered an obstetrical emergency? A. Failure of the placenta to deliver after 30 minutes B. Significant bleeding after delivery of the placenta C. Return of contractions following delivery of the baby D. More than 500 mL of blood loss before placental delivery.
C. Return of contractions following delivery of the baby
After a baby is born, it is important to: A. Ensure that it is thoroughly dried and warmed. B. Position it so that its head is higher than its body. C. Cool the infant to stimulate effective breathing. D. Immediately clamp and cut the umbilical cord.
A. Ensure that it is thoroughly dried and warmed.
A history of pelvic inflammatory disease or tubal ligations increases a woman's risk for: A. Preeclampsia. B. A placenta previa. C. Gestational diabetes. D. An ectopic pregnancy.
D. An ectopic pregnancy.
A placenta previa is MOST accurately defined as: A. Delivery of a portion of the placenta before the baby. B. Premature placental separation from the uterine wall. C. Abnormal development and functioning of the placenta. D. Development of the placenta over the cervical opening.
D. Development of the placenta over the cervical opening.
Pain that may be perceived at a distant point on the surface of the body, such as the back or shoulder, it called: A. Referred Pain. B. Radiating Pain. C. Visceral Pain. D. Remote Pain.
A. Referred Pain.
Pain that radiates to the right lower quadrant from the umbilical area, nausea and vomiting, and anorexia are MOST indicative of: A. Pancreatitis. B. Appendicitis. C. Cholecystitis. D. Gastroenteritis.
B. Appendicitis.
Most patients with abdominal pain prefer to: A. Lie on their side with their knees drawn into the abdomen. B. Sit in a semi-Fowler position with their knees slightly bent. C. Lie in a supine position with their knees in a flexed position. D. Sit fully upright because it helps relax the abdominal muscles.
A. Lie on their side with their knees drawn into the abdomen.
A 47-year-old male presents with severe abdominal pain of 3 hours' duration. His abdomen is distended and guarded. Your MOST important consideration for this patient should be able to: A. Transport him in a supine position. B. Be alert for signs and symptoms of shock. C. Assess his blood pressure to determine perfusion adequacy. D. Determine the exact location and cause of his pain.
B. Be alert for signs and symptoms of shock.
A pregnant trauma patient may lose a significant amount of blood before showing signs of shock because: A. Pregnant patients can dramatically increase their heart rate. B. Pregnancy causes vasodilation and a lower blood pressure. C. Pregnant patients have an overall increase in blood volume. D. Blood is shunted to the uterus and fetus during major trauma.
C. Pregnant patients have an overall increase in blood volume.
All of the following questions are pertinent to ask a mother when determining whether or not her baby will deliver within the next few minutes, EXCEPT: A. "When are you due?" B. "Is this your first baby?" C. "Have you had a sonogram?" D. "Do you feel the urge to push?
C. "Have you had a sonogram?"
Which of the following is an indication of imminent birth? A. Rupture of the amniotic sac. B. Crowning of the baby's head. C. Irregular contractions lasting 10 minutes. D. Expulsion of the mucus plug from the vagina.
B. Crowning of the baby's head.
If a newborn has gasping respirations after being dried and suctioned: A. You should suspect hyperthermia and cool the infant. B. Continue to stimulate the infant for another 30 seconds. C. Normal breathing will usually begin within 20 seconds. D. Further stimulation is not likely to improve ventilation.
D. Further stimulation is not likely to improve ventilation.
A 59-year-old male presents with a sudden onset of severe lower back pain. He is conscious and alert, but very restless and diaphoretic. Your assessment reveals a pulsating mass to the left of his umbilicus. You should: A. Vigorously palpate the abdomen to establish pain severity. B. Administer oxygen and prepare for immediate transport. C. Place the patient in a sitting position and transport at once. D. Request a paramedic unit to give the patient pain medication.
B. Administer oxygen and prepare for immediate transport.
When preparing a pregnant patient for delivery, you should position her: A. In a supine position with her legs spread. B. On her left side with the right leg elevated. C. In a sitting position with her hips elevated 12". D. On a firm surface in a semi-Fowler's position.
D. On a firm surface in a semi-Fowler's position.
During delivery, it is MOST important to position your partner at the mother's head because: A. The mother may become nauseated and vomit. B. The mother needs to be apprised of the situation. C. She may need emotional support during the delivery. D. Mothers often need assisted ventilation during delivery.
A. The mother may become nauseated and vomit.
An infant is considered to be premature if it: A. Is born before 38 weeks' gestation or weighs less than 6 pounds. B. Weighs less than 5.5 pounds or is born before 37 weeks' gestation. C. Is born before 40 weeks' gestation or weighs less than 7 pounds. D. Weighs less than 5 pounds or is born before 36 weeks' gestation.
D. Weighs less than 5 pounds or is born before 36 weeks' gestation.
A nuchal cord is defined as an umbilical cord that: A. Has separated from the placenta. B. Is wrapped around the baby's neck. C. Is lacerated due to a traumatic delivery. D. Has abnormally developed blood vessels.
B. Is wrapped around the baby's neck.
The presence of thick meconium in the amniotic fluid indicates: A. An expected finding in full-term infants. B. That the baby's airway may be obstructed. C. That the fetus is at least 4 weeks' premature. D. That full newborn resuscitation will be needed.
B. That the baby's airway may be obstructed.
During delivery of the baby's head, you should suction the mouth before the nose because: A. Suctioning the nose first may cause the baby to gasp and aspirate fluid. B. It is easier to suction larger volumes of fluid from the baby's oropharynx. C. Babies are primarily mouth breathers and do not breathe through their nose. D. The mucosa of the nose is fragile and is easily damaged by vigorous suctioning.
A. Suctioning the nose first may cause the baby to gasp and aspirate fluid.
Compared to full-term infants, premature infants are at a higher risk for: A. Infection. B. Breech births. C. Hyperthermia. D. Explosive deliveries.
A. Infection.
You have delivered an infant with gasping respirations. After suctioning and physical stimulation, there is no improvement. You should: A. Clamp the umbilical cord. B. Begin chest compressions. C. Administer free-flow oxygen. D. Initiate ventilatory assistance.
D. Initiate ventilatory assistance.
A 33-year-old woman who is 36 weeks' pregnant is experiencing scant vaginal bleeding. During transport, you note that she suddenly becomes diaphoretic, tachycardic, and hypotensive. In addition to administering 100% oxygen, you should: A. Place her in a left lateral recumbent position. B. Position her supine and elevate her legs 12". C. Carefully place sterile gauze into her vagina. D. Begin assisting her ventilations with a BVM.
A. Place her in a left lateral recumbent position.
A 35-year-old woman who is 30 weeks' pregnant presents with a severe headache and swelling in her hands and feet. She is conscious and alert with a blood pressure of 148/94 mm Hg, a pulse of 100 beats/min, and respirations of 24 breaths/min. During transport, you should be MOST concerned with: A. Her prematurely delivering the baby. B. The possibility that she may experience a seizure. C. Using your lights and siren to clear the traffic. D. Her hypertension causing a hemorrhagic stroke.
B. The possibility that she may experience a seizure.
You are assessing a 25-year-old woman who is 39 weeks' pregnant. She is experiencing regular contractions that are approximately 3 minutes apart and states that her amniotic sac broke 2 hours ago. After taking the appropriate BSI precautions, you should: A. Apply 100% oxygen. B. Place her on her left side. C. Assess her for crowning. D. Transport her immediately.
C. Assess her for crowning.
While examining a woman in labor, you see the umbilical cord protruding from the vagina. You should: A. Carefully push the cord back into the vagina. B. Push the infant's head away from the cord. C. Cover the umbilical cord with a dry dressing. D. Gently pull on the cord to facilitate delivery.
B. Push the infant's head away from the cord.
A 26-year-old woman complains of a stabbing pain in the right lower quadrant of her abdomen. She states that her last menstrual period was 7 weeks ago and that she had pelvic inflammatory disease approximately 3 months ago. You should be MOST suspicious that this patient is experiencing: A. An acute appendicitis. B. An intrapelvic infection. C. A spontaneous abortion. D. An ectopic pregnancy.
D. An ectopic pregnancy.
Upon delivery of the baby's head, you note that its face is encased in the unruptured amniotic sac. You should: A. Give the mother 100% oxygen and transport at once. B. Puncture the sac and suction the baby's mouth and nose. C. Leave the amniotic sac intact until arrival at the hospital. D. Note the color of the amniotic fluid before breaking the sac.
B. Puncture the sac and suction the baby's mouth and nose.
Your 22-year-old patient is in active labor. Upon visual inspection, you note that the infant's leg is protruding from the vagina. Appropriate management of this situation includes: A. Placing the mother supine with her head down and pelvis elevated. B. Gently pulling on the infant's leg in an attempt to facilitate delivery. C. Placing the mother in a recumbent position and rapidly transporting. D. Carefully attempting to push the infant's leg off of the umbilical cord.
A. Placing the mother supine with her head down and pelvis elevated.
You have just delivered a baby boy. His body is pink, but his hands and feet are blue. His heart rate is approximately 110 beats/min and his respirations are rapid and irregular. He has a weak cry when stimulated and resists attempts to straighten his legs. His APGAR score is: A. 6. B. 7. C. 8. D. 9.
C. 8.
Upon delivery of an infant's head, you note that the umbilical cord is wrapped around its neck. You should: A. Immediately clamp and cut the umbilical cord. B. Administer 100% oxygen to the mother and transport at once. C. Provide free-flow oxygen to the infant and transport. D. Attempt to slip the cord gently over the infant's head.
D. Attempt to slip the cord gently over the infant's head.
You are assessing a 30-year-old woman who is 35 weeks' pregnant. She tells you that her amniotic sac has not ruptured, but she is experiencing irregular contractions that "come and go." Upon visual inspection, you note a small amount of brown mucus draining from her vagina. You should: A. Allow her to drive to the hospital. B. Administer oxygen and transport. C. Prepare for an emergency delivery. D. Perform a rapid medical assessment.
B. Administer oxygen and transport.
During your visual inspection of a 19-year-old patient in labor, you see the baby's head crowning at the vaginal opening. What should you do? A. Apply gentle pressure to the baby's head as it delivers. B. Tell the mother not to push and transport her immediately. C. Place your fingers in the vagina to assess for a nuchal cord. D. Maintain firm pressure to the head until it completely delivers.
A. Apply gentle pressure to the baby's head as it delivers.
Following delivery of a full-term baby, you have properly cared for the baby and have clamped and cut the umbilical cord. During transport, you note that the mother is experiencing moderate vaginal bleeding. You should: A. Elevate her legs 6" to 8" and cover her with a blanket. B. Firmly massage the uterine funds with a circular motion. C. Carefully insert a sterile trauma dressing into her vagina. D. Place her legs together and position her on her left side.
B. Firmly massage the uterine funds with a circular motion.
You have just delivered a premature baby. Your assessment reveals that he is breathing adequately; however, his heart rate is 90 beats/min. You should: A. Keep him warm and provide ventilatory assistance. B. Begin chest compressions and reassess in 30 seconds. C. Clamp and cut the umbilical cord and keep him warm. D. Assess his skin color and give free-flow oxygen as needed.
A. Keep him warm and provide ventilatory assistance.
During your assessment of a young female with non traumatic vaginal bleeding, you note that her level of consciousness is decreased, her respirations are rapid and shallow, her skin is cool and moist, and her pulse is rapid and weak. You should: A. Perform a rapid secondary assessment. B. Assist her ventilations with a bag-mask device. C. Assess her blood pressure and elevate her legs. D. Perform a visual assessment of her vaginal area.
B. Assist her ventilations with a bag-mask device.
A 26-year-old female presents with heavy vaginal bleeding. She is conscious, but restless. Her blood pressure is 84/54 mm Hg, her pulse is 120 beats/min and weak, and her respirations are 22 breaths/min with adequate depth. She tells you that she inserted a tampon about 2 hours ago. You should: A. Administer high-flow oxygen, ask her to remove the tampon, perform a detailed secondary assessment, and transport promptly. B. Administer high-flow oxygen, perform a detailed assessment of her vaginal area for signs of trauma, place her on her side, and transport. C. Administer high-flow oxygen, place a sterile pad over her vagina, keep her warm, elevate her lower extremities, and transport without delay. D. Assist her ventilations with a bag-mask device, place one sterile dressing into her vagina, perform a rapid secondary assessment, and transport.
C. Administer high-flow oxygen, place a sterile pad over her vagina, keep her warm, elevate her lower extremities, and transport without delay.
Whenever possible, a female sexual assault victim should be: A. Given the option of being treated by a female EMT. B. Encouraged to take a shower and changer her clothes. C. Thoroughly assessed, even if no signs of injury exist. D. Asked to provide a brief description of the perpetrator.
A. Given the option of being treated by a female EMT.
It is not uncommon for young females who experience their first menstrual period to: A. Have a falsely positive home pregnancy test result. B. Lose up to 500 mL of blood within the first 24 hours. C. Experience abdominal cramping, which may be misinterpreted. D. Become so emotionally distraught that they may contemplate suicide.
C. Experience abdominal cramping, which may be misinterpreted.
When caring for a woman who is experiencing a gynecologic emergency, the EMT's main focus should be to: A. Determine the underlying cause of her problem. B. Ask questions related to her gynecologic history. C. Maintain her ABCs and transport without delay. D. Keep assessment and treatment to a minimum.
C. Maintain her ABCs and transport without delay.
Colic is pain caused by: A. Irritation of the parietal peritoneum. B. Distention or contraction of hollow organs. C. Blunt trauma to the solid abdominal organs. D. Vigorous palpation of the abdominal quadrants.
B. Distention or contraction of hollow organs.
Pain felt at a location other than its origin is called: A. Referred pain. B. Radiating pain. C. Visceral pain. D. Remote pain.
A. Referred pain.
The parietal peritoneum lines the: A. Retroperitoneal space. B. Lungs and chest cavity. C. Walls of the abdominal cavity. D. Surface of the abdominal organs.
C. Walls of the abdominal cavity.
The MOST common and significant complication associated with an acute abdomen is: A. Peritonitis. B. High fever. C. Severe pain. D. Internal bleeding.
A. Peritonitis.
Which of the following statements regarding the acute abdomen is MOST correct? A. The most common cause of an acute abdomen is inflammation of the gallbladder and liver. B. The parietal peritoneum is typically the first abdominal layer that becomes inflamed or irritated. C. The initial pain associated with an acute abdomen tends to be vague and poorly localized. D. An acute abdomen almost always occurs as the result of blunt trauma to solid abdominal organs.
C. The initial pain associated with an acute abdomen tends to be vague and poorly localized.
Erosion of the stomach or duodenum secondary to overactivity of digestive juices results in: A. Ileus. B. An ulcer. C. Appendicitis. D. Cholecystitis.
B. An ulcer.
Pain that is localized to the lower back and/or lower abdominal quadrants is MOST suggestive of: A. Acute pancreatitis. B. An aortic aneurysm. C. A kidney infection. D. Acute appendicitis.
B. An aortic aneurysm.
Which of the following organs lies in the retroperitoneal space? A. Liver. B. Spleen. C. Pancreas. D. Gallbladder.
C. Pancreas.
Which of the following conditions is more common in women than in men? A. Cystitis. B. Hepatitis. C. Pancreatitis. D. Cholecystitis.
A. Cystitis.
Pelvic inflammatory disease (PID) is an infection of the: A. Testicles. B. Gallbladder. C. Urinary Bladder. D. Fallopian Tubes.
D. Fallopian Tubes.
An incarcerated hernia is one that: A. Spontaneously reduces without any surgical intervention. B. Can be pushed back in the body cavity to which it belongs. C. Loses its blood supply due to compression by local tissues. D. Cannot be pushed back within its respective body cavity.
D. Cannot be pushed back within its respective body cavity.
In the presence of ileum, the only way the stomach can empty itself is by: A. Diarrhea. B. Vomiting. C. Muscular Contraction. D. Spontaneous Rupture.
B. Vomiting.
Which of the following is a nonspecific complaint that is commonly associated with an acute abdomen? A. High fever. B. Hematemesis. C. Loss of appetite. D. Lower quadrant pain.
C. Loss of appetite.
Peritonitis may result in shock because: A. Intra-abdominal hemorrhage is typically present. B. Fluid shifts from the bloodstream into body tissues. C. Abdominal distention impairs cardiac contractions. D. Severe pain causes systemic dilation of the vasculature.
B. Fluid shifts from the bloodstream into body tissues.
Which of the following statements regarding an ectopic pregnancy is correct? A. Signs of an ectopic pregnancy usually present within the first 6 to 8 weeks of pregnancy. B. The abdominal cavity is the most common location for abnormal implantation of a fertilized egg. C. An ectopic pregnancy almost always presents with acute signs and symptoms in the third trimester. D. Pain from a ruptured ectopic pregnancy is usually severe but the bleeding is typically rather minimal.
A. Signs of an ectopic pregnancy usually present within the first 6 to 8 weeks of pregnancy.
Common signs and symptoms of acute abdomen include all of the following, EXCEPT: A. Tachycardia. B. Hypertension. C. Nausea and Vomiting. D. Abdominal Guarding.
B. Hypertension.
When asking a patient if his or her abdominal pain "moves anywhere," you are establishing the presence or absence of: A. Referred Pain. B. Provoking Pain. C. Palliating Pain. D. Radiating Pain.
D. Radiating Pain.
Patients with acute abdominal pain should not be given anything by mouth because: A. It will create referred pain and obscure the diagnosis. B. Food will rapidly travel through the digestive system. C. A full stomach may make emergency surgery dangerous. D. Digestion prevents accurate auscultation of bowel sounds.
C. A full stomach may make emergency surgery dangerous.
When assessing a patient with right lower quadrant pain, you should first palpate the: A. Left Upper Quadrant. B. Left Lower Quadrant. C. Right Upper Quadrant. D. Right Lower Quadrant.
A. Left Upper Quadrant.
Which of the following is a late sign of shock in a patient with acute abdominal pain? A. Hypotension B. Tachycardia C. Tachypnea D. Diaphoresis
A. Hypotension
A 35-year-old mildly obese woman is complaining of localized pain in the right upper quadrant with referred pain to the right shoulder. The MOST likely cause of her pain is: A. Acute Cystitis. B. Cholecystitis. C. Appendicitis. D. Pancreatitis.
B. Cholecystitis.
A 59-year-old male presents with a sudden onset of severe lower back pain. He is conscious and alert, but very restless and diaphoretic. Your assessment reveals a pulsating mass to the left of his umbilicus. You should: A. Vigorously palpate the abdomen to establish pain severity. B. Administer oxygen and prepare for immediate transport. C. Place the patient in a sitting position and transport at once. D. Request a paramedic unit to give the patient pain medication.
B. Administer oxygen and prepare for immediate transport.
You are caring for a 25-year-old female with acute left lower quadrant pain. She is conscious and alert and in obvious pain. Her blood pressure is 110/70 mm Hg, pulse is 100 beats/min and regular, and respirations are 24 and unlabored. In addition to administering oxygen, you should: A. Place her in Trendelenburg's position. B. Request permission to give her Motrin. C. Give her small sips of water to drink. D. Place her in a comfortable position.
D. Place her in a comfortable position.
A 47-year-old male presents with severe abdominal pain of 3 hours' duration. His abdomen is distended and guarded. Your MOST important consideration for this patient should be to: A. Transport him in a supine position. B. Be alert for signs and symptoms of shock. C. Assess his BP to determine perfusion adequacy. D. Determine the exact location and cause of his pain.
B. Be alert for signs and symptoms of shock.
An important aspect in the treatment of a patient with severe abdominal pain is to: A. Provide emotional support en route to the hospital. B. Administer analgesic medications to alleviate pain. C. Encourage the patient to remain in a supine position. D. Give 100% oxygen only if signs of shock are present.
A. Provide emotional support en route to the hospital.
A 66-year-old female complains of diffuse abdominal pain and nausea. She has a history of hypertension and occasional depression. When caring for this patient, you should be MOST concerned with: A. The possibility that she may vomit. B. Determining the origin of her pain. C. The emotional effects caused by the pain. D. An acute rise in her systolic blood pressure.
A. The possibility that she may vomit.
The MOST appropriate treatment for a patient with severe abdominal pain and signs of shock includes: A. Administering oxygen via a nasal cannula. B. Giving oral fluids to maintain perfusion. C. Transporting the patient without delay. D. Positioning the patient on his or her side.
C. Transporting the patient without delay.
You are transporting a 49-year-old male with "tearing" abdominal pain. You are approximately 30 miles away from the closest hospital. During your ongoing assessment, you determine that the patient's condition has deteriorated significantly. You should: A. Begin ventilatory assistance with a BVM device. B. Immediately perform a detailed physical examination. C. Continue transporting and alert the receiving hospital. D. Consider requesting a rendezvous with an ALS unit.
D. Consider requesting a rendezvous with an ALS unit.
A young female presents with costovertebral angle tenderness. She is conscious and alert with stable vital signs. Which of the following organs is MOST likely causing her pain? A. Liver. B. Kidney. C. Pancreas. D. Gallbladder.
B. Kidney.
You are dispatched to an apartment complex for a young male with abdominal pain. Your priority upon arriving at the scene should be to: A. Quickly gain access to the patient. B. Notify the dispatcher of your arrival. C. Assess the scene for potential hazards. D. Place a paramedic ambulance on standby.
C. Assess the scene for potential hazards.
The term primigravida refers to a woman who: A. Has never been pregnant. B. Has had only one live birth. C. Is pregnant for the first time. D. Has had more than one live baby.
C. Is pregnant for the first time.
After the fetus has descended into the pelvis at the end of the third trimester, many mothers experience: A. Midback pain. B. A bloated feeling. C. An urge to push. D. Easier breathing.
D. Easier breathing.
A mother who is pregnant with her first baby is typically in the first stage of labor for approximately: A. 4 hours. B. 8 hours. C. 10 hours. D. 16 hours.
D. 16 hours.
When preparing a pregnant patient for delivery, you should position her: A. In a supine position with her legs spread. B. On her left side with the right leg elevated. C. In a sitting position with her hips elevated 12″. D. On a firm surface with her hips elevated 2″ to 4″.
D. On a firm surface with her hips elevated 2″ to 4″.
If a newborn's heart rate is less than 60 beats/min following delivery, you should: A. Flick the soles of its feet. B. Begin chest compressions. C. Resuction the mouth only. D. Provide ventilations for 30 seconds.
D. Provide ventilations for 30 seconds.
A newborn infant will usually begin breathing spontaneously within _______ seconds following birth. A. 3 to 5. B. 5 to 10. C. 15 to 30. D. 30 to 60.
C. 15 to 30.
In contrast to a full-term infant, a premature infant: A. Has an even proportionately larger head. B. Is often covered with excess vernix material. C. Is one who is born before 38 weeks' gestation. D. Retains heat better because of excess body hair.
A. Has an even proportionately larger head.
The presence of thick meconium in the amniotic fluid indicates: A. An expected finding in full-term infants. B. That the baby's airway may be obstructed. C. That the fetus is at least 4 weeks premature. D. That full newborn resuscitation will be needed.
B. That the baby's airway may be obstructed.
Spina bifida is a developmental defect in which A. An excessive amount of cerebrospinal fluid damages the spinal cord. B. Nerve fibers that arise from the spinal cord do not function properly. C. The spinal column is severely deformed, resulting in permanent paralysis. D. A portion of the spinal cord or meninges protrudes outside of the vertebrae.
D. A portion of the spinal cord or meninges protrudes outside of the vertebrae.
Which of the following statements regarding the placenta is correct? A. The placenta allows oxygen, carbon dioxide, and other products to transfer between the mother and fetus but does not allow blood to mix between the mother and fetus. B. The placental barrier consists of two layers of cells and allows the mother's blood that contains high concentrations of oxygen to directly mix with the blood of the fetus. C. The placenta, also referred to as the afterbirth, provides oxygen and nutrients to the fetus and is expelled from the vagina about 30 minutes before the baby is born. D. The placenta allows for the transfer of oxygen and carbon dioxide between the mother and fetus but prevents most medications from passing between the mother and fetus.
A. The placenta allows oxygen, carbon dioxide, and other products to transfer between the mother and fetus but does not allow blood to mix between the mother and fetus.
The term "bloody show" is defined as: A. The small amount of pink-tinged mucus that is discharged from the vagina after expulsion of the mucous plug. B. Mild vaginal bleeding that occurs within the first 30 minutes after the onset of the second stage of the labor process. C. The normal amount of vaginal bleeding that occurs within the first 24 hours following delivery of the baby and placenta. D. Any volume of blood that is expelled from the vagina after the amniotic sac has ruptured and contractions have begun.
A. The small amount of pink-tinged mucus that is discharged from the vagina after expulsion of the mucous plug.
If a pregnant patient requires spinal immobilization, you should secure her to the backboard and then: A. Tilt the board 30° to the right to prevent hypotension. B. Elevate the right side of the board with rolled towels or blankets. C. Raise the foot of the board 12″ in order to maintain blood pressure. D. Elevate the head of the board 6″ to prevent breathing impairment.
B. Elevate the right side of the board with rolled towels or blankets.
During delivery of the baby's head, you should suction the mouth before the nose because: A. Suctioning the nose first may cause the baby to gasp and aspirate fluid. B. It is easier to suction larger volumes of fluid from the baby's oropharynx. C. Babies are primarily mouth breathers and do not breathe through their nose. D. The mucosa of the nose is fragile and is easily damaged by vigorous suctioning.
A. Suctioning the nose first may cause the baby to gasp and aspirate fluid.
From what internal female organ is the fetus expelled during delivery? A. Vagina. B. Uterus. C. Cervix. D. Perineum.
B. Uterus.
Braxton-Hicks contractions are characterized by: A. Regular contractions of progressively increasing intensity. B. Alleviation of pain with movement or changing positions. C. Pink or red bloody show in conjunction with the contractions. D. A rupture of the amniotic sac just before the contractions begin. The umbilical cord: A. Separates from the placenta shortly after birth. B. Carries blood away from the baby via the artery. C. Carries oxygen to the baby via the umbilical vein. D. Contains two veins and one large umbilical artery.
B. Alleviation of pain with movement or changing positions. ________________________________________ C. Carries oxygen to the baby via the umbilical vein.
The umbilical cord: A. Separates from the placenta shortly after birth. B. Carries blood away from the baby via the artery. C. Carries oxygen to the baby via the umbilical vein. D. Contains two veins and one large umbilical artery.
C. Carries oxygen to the baby via the umbilical vein.
The onset of labor begins with: A. Thinning of the uterus. B. Full dilation of the cervix. C. Increased fetal movement. D. Contractions of the uterus.
D. Contractions of the uterus.
Signs and symptoms of preeclampsia include: A. Headache and Edema. B. Marked Hypoglycemia. C. Dyspnea and Bradycardia. D. Dysuria and Constipation.
A. Headache and Edema.
Which of the following statements regarding twins is correct? A. Twins are typically larger than single infants. B. Identical twins are typically of different gender. C. Most twins are born within 45 minutes of each other. D. Fraternal twins have two cords coming from one placenta.
C. Most twins are born within 45 minutes of each other.
After a baby is born, it is important to: A. Ensure that it is thoroughly dried and warmed. B. Position it so that its head is higher than its body. C. Cool the infant to stimulate effective breathing. D. Immediately clamp and cut the umbilical cord.
A. Ensure that it is thoroughly dried and warmed.
Abruptio placenta occurs when: A. The placenta prematurely separates from the uterine wall. B. A tear in the placenta causes severe internal hemorrhage. C. The placenta affixes itself to the outer layer of the uterus. D. The placenta develops over and covers the cervical opening.
A. The placenta prematurely separates from the uterine wall.
The ONLY indications for placing your gloved fingers in the vagina during delivery are: A. Breech presentation and prolapsed umbilical cord. B. Limb presentation and severe vaginal hemorrhage. C. Vertex presentation and delivery of the placenta. D. Nuchal cord and presentation of an arm or leg.
A. Breech presentation and prolapsed umbilical cord.
Which of the following is an indication of imminent birth? A. Rupture of the amniotic sac. B. Crowning of the baby's head. C. Irregular contractions lasting 10 minutes. D. Expulsion of the mucus plug from the vagina.
B. Crowning of the baby's head.
Following delivery of a full-term baby, you have properly cared for the baby and have clamped and cut the umbilical cord. During transport, you note that the mother is experiencing moderate vaginal bleeding. You should: A. Elevate her legs 6″ to 8″ and cover her with a blanket. B. Firmly massage the uterine funds with a circular motion. C. Carefully insert a sterile trauma dressing into her vagina. D. Place her legs together and position her on her left side.
B. Firmly massage the uterine funds with a circular motion.
Which of the following statements regarding a breech presentation is MOST correct? A. A breech presentation occurs when the buttocks are the presenting part. B. There is minimal risk of trauma to the infant with a breech presentation. C. It is impossible to deliver a breech presentation in the prehospital setting. D. Breech deliveries occur rapidly, so the EMT should deliver at the scene.
A. A breech presentation occurs when the buttocks are the presenting part.
A precipitous labor and delivery is MOST common in women who: A. Have gestational diabetes. B. Are younger than 30 years of age. C. Have delivered a baby before. D. Are pregnant for the first time.
C. Have delivered a baby before.
A 33-year-old woman who is 36 weeks pregnant is experiencing scant vaginal bleeding. During transport, you note that she suddenly becomes diaphoretic, tachycardic, and hypotensive. In addition to administering 100% oxygen, you should: A. Place her in a left lateral recumbent position. B. Position her supine and elevate her legs 12″. C. Carefully place sterile gauze into her vagina. D. Assist her ventilations with a bag-mask device.
A. Place her in a left lateral recumbent position.
By the 20th week of pregnancy, the uterus is typically at or above the level of the mother's: A. Belly Button. B. Pubic Bone. C. Xiphoid process. D. Superior Diaphragm.
A. Belly Button.
During your visual inspection of a 19-year-old woman in labor, you see the baby's head crowning at the vaginal opening. What should you do? A. Apply gentle pressure to the baby's head as it delivers. B. Tell the mother not to push and transport her immediately. C. Place your fingers in the vagina to assess for a nuchal cord. D. Maintain firm pressure to the head until it completely delivers.
A. Apply gentle pressure to the baby's head as it delivers.
Which of the following questions is of LEAST pertinence when determining whether a mother will deliver her baby within the next few minutes? A. "When are you due?" B. "Is this your first baby?" C. "Have you had a sonogram?" D. "Do you feel the urge to push?"
C. "Have you had a sonogram?"
While examining a woman in labor, you see the umbilical cord protruding from the vagina. You should: A. Carefully push the cord back into the vagina. B. Push the infant's head away from the cord. C. Cover the umbilical cord with a dry dressing. D. Gently pull on the cord to facilitate delivery.
B. Push the infant's head away from the cord.
When the mother is experiencing a contraction, you should instruct her to: A. Hold her breath. B. Push for 30 seconds. C. Take quick short breaths. D. Rest and breathe deeply.
C. Take quick short breaths.
Upon delivery of the baby's head, you note that its face is encased in the unruptured amniotic sac. You should: A. Give the mother 100% oxygen and transport at once. B. Puncture the sac and suction the baby's mouth and nose. C. Leave the amniotic sac intact until arrival at the hospital. D. Note the color of the amniotic fluid before breaking the sac.
B. Puncture the sac and suction the baby's mouth and nose.
The vagina and the neck of the uterus comprise the: A. Womb. B. Cervix. C. Fundus. D. Birth Canal.
D. Birth Canal.
Upon delivery of an infant's head, you note that the umbilical cord is wrapped around its neck. You should: A. Immediately clamp and cut the umbilical cord. B. Give 100% oxygen to the mother and transport at once. C. Provide free-flow oxygen to the infant and transport. D. Attempt to slip the cord gently over the infant's head.
D. Attempt to slip the cord gently over the infant's head.
Which of the following is NOT a component of the Apgar score? A. Pulse. B. Activity. C. Grimace. D. Body Size.
D. Body Size.
A history of pelvic inflammatory disease or tubal ligations increases a woman's risk for: A. Preeclampsia. B. Placenta previa. C. Gestational diabetes. D. An ectopic pregnancy.
D. An ectopic pregnancy.
Which of the following statements regarding gestational diabetes is correct? A. Gestational diabetes results in permanent diabetes mellitus after delivery. B. The onset of gestational diabetes typically manifests with hypoglycemia. C. In some cases, women with gestational diabetes require insulin injections. D. Diet and exercise are typically ineffective in controlling gestational diabetes.
C. In some cases, women with gestational diabetes require insulin injections.
You have just delivered a baby boy. His body is pink, but his hands and feet are blue. His heart rate is approximately 110 beats/min and his respirations are rapid and irregular. He has a weak cry when stimulated and resists attempts to straighten his legs. His Apgar score is: A. 6. B. 7. C. 8. D. 9.
C. 8.
You have just delivered a premature baby. Your assessment reveals that he is breathing adequately; however, his heart rate is 90 beats/min. You should: A. Keep him warm and provide ventilatory assistance. B. Begin chest compressions and reassess in 30 seconds. C. Clamp and cut the umbilical cord and keep him warm. D. Assess his skin color and give free-flow oxygen as needed.
A. Keep him warm and provide ventilatory assistance.
Which of the following processes occurs during ovulation? A. Certain female hormone levels decrease significantly in quantity. B. The endometrium sheds its lining and is expelled from the vagina. C. The inner lining of the uterus thickens in preparation for implantation. D. Numerous follicles mature and release eggs into the fallopian tubes.
C. The inner lining of the uterus thickens in preparation for implantation.
You are assessing a 25-year-old woman who is 39 weeks pregnant. She is experiencing regular contractions that are approximately 3 minutes apart and states that her amniotic sac broke 2 hours ago. After taking the standard precautions, you should: A. Apply 100% oxygen. B. Place her on her left side. C. Assess her for crowning. D. Transport her immediately.
C. Assess her for crowning.
Your 22-year-old patient is in active labor. Upon visual inspection, you note that the infant's leg is protruding from the vagina. Appropriate management of this situation includes: A. Placing the mother supine with her head down and pelvis elevated. B. Gently pulling on the infant's leg in an attempt to facilitate delivery. C. Placing the mother in a recumbent position and rapidly transporting. D. Carefully attempting to push the infant's leg off of the umbilical cord.
A. Placing the mother supine with her head down and pelvis elevated.
Eclampsia is MOST accurately defined as: A. High levels of protein in the patient's urine. B. Hypertension in the 20th week of pregnancy. C. Seizures that result from severe hypertension. D. A blood pressure greater than 140/90 mm Hg.
C. Seizures that result from severe hypertension.
A nuchal cord is defined as an umbilical cord that: A. Has separated from the placenta. B. Is wrapped around the baby's neck. C. Is lacerated due to a traumatic delivery. D. Has abnormally developed blood vessels.
B. Is wrapped around the baby's neck.
An abortion occurs when the fetus and placenta deliver before: A. 20 weeks. B. 24 weeks. C. 26 weeks. D. 28 weeks.
A. 20 weeks.
During delivery, it is MOST important to position your partner at the mother's head because: A. The mother may become nauseated and vomit. B. The mother needs to be apprised of the situation. C. She may need emotional support during the delivery. D. Mothers often need assisted ventilation during delivery.
A. The mother may become nauseated and vomit.
A pregnant trauma patient may lose a significant amount of blood before showing signs of shock because: A. Pregnant patients can dramatically increase their heart rate. B. Pregnancy causes vasodilation and a lower blood pressure. C. Pregnant patients have an overall increase in blood volume. D. Blood is shunted to the uterus and fetus during major trauma.
C. Pregnant patients have an overall increase in blood volume.
Most medical models base a pregnant woman's due date: A. Two weeks after her last menstrual cycle. B. On the last day of her last menstrual cycle. C. Two weeks before her last menstrual cycle. D. On the first day of her last menstrual cycle.
D. On the first day of her last menstrual cycle.
The amniotic fluid serves to: A. Transfer oxygen to the fetus. B. Insulate and protect the fetus. C. Remove viruses from the fetus. D. Assist in fetal development.
B. Insulate and protect the fetus.
Following delivery of a pulseless and apneic infant who has a foul odor, skin sloughing, and diffuse blistering, you should: A. Begin full resuscitation and transport. B. Report the case to the medical examiner. C. Provide emotional support to the mother. D. Dry the infant off to stimulate breathing.
C. Provide emotional support to the mother.
Which of the following is a normal physiologic change that occurs in the mother's respiratory system during pregnancy? A. Decreased respiratory rate and increased minute volume. B. Increased respiratory rate and decreased respiratory reserve. C. Increased respiratory reserve and decreased oxygen demand. D. Increased respiratory depth and decreased respiratory rate.
B. Increased respiratory rate and decreased respiratory reserve.
You have just delivered a full-term infant. His respirations are rapid and irregular, and he has a strong cry. What should you do next? A. Allow the mother to hold her baby. B. Clamp and cut the umbilical cord. C. Assess the brachial or umbilical pulse. D. Begin assisting the newborn's breathing.
C. Assess the brachial or umbilical pulse.
Supine hypotensive syndrome occurs when: A. A supine position kinks the ascending aorta. B. The pregnant uterus compresses the inferior vena cava. C. The superior vena cava is compressed by the uterus. D. Blood pressure decreases as a result of hypovolemia.
B. The pregnant uterus compresses the inferior vena cava.
If a baby is born at 7:52, the second Apgar score should be calculated at: A. 7:53. B. 7:57. C. 7:59. D. 8:00.
B. 7:57.
An infant is considered to be premature if it: A. Is born before 38 weeks' gestation or weighs less than 6 lb. B. Weighs less than 5.5 lb or is born before 37 weeks' gestation. C. Is born before 40 weeks' gestation or weighs less than 7 lb. D. Weighs less than 5 lb or is born before 36 weeks' gestation.
D. Weighs less than 5 lb or is born before 36 weeks' gestation.
When determining the frequency of contractions, you should time the contractions from the: A. End of one to the start of the next. B. Start of one to the end of the next. C. End of one to the end of the next. D. Start of one to the start of the next.
D. Start of one to the start of the next.
Common interventions used to stimulate spontaneous respirations in the newborn include all of the following, EXCEPT: A. Suctioning of the upper airway. B. Thorough drying with a towel. C. Positive-pressure ventilations. D. Some form of tactile stimulation.
C. Positive-pressure ventilations.
The leading cause of maternal death during the first trimester of pregnancy is: A. Massive brain damage secondary to a prolonged seizure. B. Unrecognized or untreated supine hypotensive syndrome. C. Internal bleeding caused by a ruptured ectopic pregnancy. D. Blunt trauma to the abdomen during a motor vehicle crash.
C. Internal bleeding caused by a ruptured ectopic pregnancy.
The third stage of labor begins when the: A. Placenta is fully delivered. B. Cervix is completely dilated. C. Umbilical cord has been clamped. D. Baby is expelled from the vagina.
D. Baby is expelled from the vagina.
Which of the following statements regarding the placenta is correct? A. The placenta allows for the transfer of oxygen and carbon dioxide between the mother and fetus but prevents most medications from passing between the mother and fetus. B. The placenta allows oxygen, carbon dioxide, and other products to transfer between the mother and fetus but does not allow blood to mix between the mother and fetus. C. The placental barrier consists of two layers of cells and allows the mother's blood that contains high concentrations of oxygen to directly mix with the blood of the fetus. D. The placenta, also referred to as the afterbirth, provides oxygen and nutrients to the fetus and is expelled from the vagina about 30 minutes before the baby is born.
B. The placenta allows oxygen, carbon dioxide, and other products to transfer between the mother and fetus but does not allow blood to mix between the mother and fetus.
A mother who is pregnant with her first baby is typically in the first stage of labor for approximately: A. 10 hours. B. 8 hours. C. 16 hours. D. 4 hours.
C. 16 hours.
After the fetus has descended into the pelvis at the end of the third trimester, many mothers experience: A. A bloated feeling. B. Easier breathing. C. Midback pain. D. An urge to push.
B. Easier breathing.
Signs and symptoms of preeclampsia include: A. Dysuria and constipation. B. Marked hypoglycemia. C. Dyspnea and bradycardia. D. Headache and edema.
D. Headache and edema.
Abruptio placenta occurs when: A. The placenta prematurely separates from the uterine wall. B. A tear in the placenta causes severe internal hemorrhage. C. The placenta affixes itself to the outer layer of the uterus. D. The placenta develops over and covers the cervical opening.
A. The placenta prematurely separates from the uterine wall.
When preparing a pregnant patient for delivery, you should position her: A. In a supine position with her legs spread. B. On her left side with the right leg elevated. C. In a sitting position with her hips elevated 12². D. On a firm surface with her hips elevated 2'' to 4''.
D. On a firm surface with her hips elevated 2" to 4".
When the mother is experiencing a contraction, you should instruct her to: A. Rest and breathe deeply. B. Push for 30 seconds. C. Take quick short breaths. D. Hold her breath.
C. Take quick short breaths.
During delivery of the baby's head, you should suction the mouth before the nose because: A. Suctioning the nose first may cause the baby to gasp and aspirate fluid. B. It is easier to suction larger volumes of fluid from the baby's oropharynx. C. Babies are primarily mouth breathers and do not breathe through their nose. D. The mucosa of the nose is fragile and is easily damaged by vigorous suctioning.
A. Suctioning the nose first may cause the baby to gasp and aspirate fluid.
A newborn infant will usually begin breathing spontaneously within _______ seconds following birth. A. 15 to 30. B. 3 to 5. C. 5 to 10. D. 30 to 60.
A. 15 to 30.
The ONLY indications for placing your gloved fingers in the vagina during delivery are: A. Vertex presentation and delivery of the placenta. B. Nuchal cord and presentation of an arm or leg. C. Breech presentation and prolapsed umbilical cord. D. Limb presentation and severe vaginal hemorrhage.
C. Breech presentation and prolapsed umbilical cord.
The fetus develops inside a fluid-filled, bag-like membrane called the amniotic sac, and contains: A. 200 to 250 mL of fluid. B. 250 to 359 mL of fluid. C. 350 to 500 mL of fluid. D. 500 to 1,000 mL of fluid.
D. 500 to 1,000 mL of fluid.
As the time for delivery nears, certain complications can occur. Preeclampsia, or pregnancy-induced hypertension, is a condition that can develop after the: A. 20th week of gestation. B. 24th week of gestation. C. 26th week of gestation. D. 30th week of gestation.
A. 20th week of gestation.
Prompt hospital treatment should be provided for a pregnant woman with vaginal bleeding. Which of the following should not be done in this emergency? A. Place a sanitary pad or trauma dressing in the vagina. B. Place her on her left side during transport. C. Give her high-flow oxygen. D. Save any tissue that may be passed from the vagina.
A. Place a sanitary pad or trauma dressing in the vagina.
An unruptured amniotic sac may contain a foul-smelling, greenish color fluid. This information must be: A. Included in your narrative report only. B. Given to the parents to pass along to their pediatrician. C. Relayed in your report to medical control. D. Ignored; this is normal in 50% of births.
C. Relayed in your report to medical control.
As soon as the head is delivered, use the index finger of your other hand to feel whether the umbilical cord is wrapped around the neck. This condition is commonly called: A. A nuchal cord. B. A blue cord. C. A tightly wound cord. D. A suffocative cord.
A. A nuchal cord.
Post delivery care includes drying the baby off, wrapping it immediately in a blanket or towel, and placing it on one side with: A. The head slightly lower than the rest of the body. B. The head slightly higher than the rest of the body. C. Its face down. D. A high flow of oxygen.
A. The head slightly lower than the rest of the body.
Once the infant is born, the umbilical cord is of no further use to the mother or infant. Post delivery care of the umbilical cord is important because: A. The tissue of the cord is very tough and must be cut quickly. B. Infection is easily transmitted through the cord to the baby. C. It could wrap around the infant's neck. D. It will quickly detach itself from the placenta.
B. Infection is easily transmitted through the cord to the baby.
To slow bleeding, you can massage the mother's abdomen in a firm, circular motion after the baby is delivered, stimulating the uterus to contract. You can also: A. Administer two tablespoons of oxytocin, if the mother has it available. B. Raise the upper half of the mother's body. C. Place the infant on the mother's breast to nurse. D. Place one or two sanitary napkins into the vagina.
C. Place the infant on the mother's breast to nurse.
Excessive bleeding occurs when the bleeding exceeds: A. 350 mL. B. 500 mL. C. 750 mL. D. 900 mL.
B. 500 mL.
A full-term fetus grows and develops inside the uterus for approximately _________ weeks. A. 30. B. 32. C. 36. D. 40.
D. 40.
Which of the following does not occur during the second stage of labor? A. The fetus enters the birth canal. B. Lightening occurs. C. The mother may feel pressure on her rectum. D. The perineum will begin to bulge.
B. Lightening occurs.
Which of the following is not measured using the Apgar score? A. Grimace or Irritability. B. Pulse. C. Weight. D. Activity or Muscle Tone.
C. Weight.
In what situation is it acceptable to place your sterile gloved fingers into the vagina? A. Breech delivery. B. Limb presentation. C. Prolapsed cord. D. Both A and C.
D. Both A and C.
Common interventions used to stimulate spontaneous respirations in the newborn include all of the following, EXCEPT: A. Suctioning the upper airway. B. Thorough drying with a towel. C. Positive-pressure ventilations. D. Some form of tactile stimulation.
C. Positive-pressure ventilations.
Which of the following newborns requires positive-pressure ventilations and chest compressions? A. Central cyanosis; respirations, fast; pulse, 100 beats/min B. Central cyanosis; respirations, slow; pulse, 50 beats/min C. Peripheral cyanosis; respirations, fast; pulse, 90 beats/min D. Peripheral cyanosis; respirations, absent; pulse, 80 beats/min
B. Central cyanosis; respirations, slow; pulse, 50 beats/min.
Vigorous suctioning of a newborn's airway is indicated if: A. There is meconium in the amniotic fluid. B. Positive-pressure ventilations are indicated. C. The newborn presents with labored breathing. D. His or her heart rate is less than 60 beats/min.
A. There is meconium in the amniotic fluid.
Which of the following is the lowest portion of the birth canal? A. Cervix. B. Uterus. C. Vagina. D. Perineum.
C. Vagina.
Uterine contractions that expel the fetus and placenta are generally referred to as A. Labor. B. The braxton-Hicks phase. C. The dilation phase. D. The fetal expulsion phase.
A. Labor.
The final stage of labor is generally described as the A. Dilation stage. B. Expulsion stage. C. Placental stage. D. Fundal stage.
C. Placental stage.
A woman who is gravid III A. Is pregnant for the fourth time. B. Has delivered three times. C. Is pregnant for the third time. D. Has delivered for the fourth time.
C. Is pregnant for the third time.
Postmaturity syndrom A. Occurs when the gestation of the fetus extends beyond 32 weeks. B. Occurs when the gestation of the fetus extends beyond 36 weeks. C. Causes improved placental blood flow and nutrient delivery to the fetus. D. Causes a hardened fetal skull leading to a more difficult delivery.
D. Causes a hardened fetal skull leading to a more difficult delivery.
Your supine, third-trimester pregnant patient is noted to have a lowered systolic blood pressure. You suspect that this may be due to which of the following? A. Supine hypotensive syndrome B. Lateral emergent syndrom C. Prone hypertensive syndrom D. Supine hyperglycemic syndrom
A. Supine hypotensive syndrome
Management specific to the situation noted in question 6 should include A. Placing the patient in the trendelenburg position. B. Beginning chest compressions. C. Positioning the patient on her side. D. Applying the AED (automated external defibrillator).
C. Positioning the patient on her side.
Your 30-year-old female patient is 4 months pregnant. She complains of cramping, abdominal pain, and bright red vaginal bleeding. Your treatment should include A. Packing the vagina with sanitary pads to control bleeding. B. Elevation of the patient's head to prevent aspiration. C. The administration of oxygen at 6 lpm. D. Providing general management for shock.
D. Providing general management for shock.
Your 20-year-old patient is 8 months pregnant. Prior to your arrival, she had a seizure that lasted about 2 minutes. You find her to have an altered mental status with a patent airway and adequate respirations and perfusion. Care of this patient should include A. Administration of low-flow oxygen by non rebreather mask. B. Withholding positive pressure ventilation. C. Transporting the patient in a supine position. D. Minimization of noise, light, and movement to prevent further seizures.
D. Minimization of noise, light, and movement to prevent further seizures.
Which of the following is a sign of an imminent delivery? A. Contractions that occur every 2 minutes or closer and last 60-90 seconds. B. Contractions that occur ever 2 minutes and last 30-60 seconds. C. Contractions that occur every 4 minutes and last 30-60 seconds. D. Contractions that occur every 4 minutes and last 60-90 seconds.
A. Contractions that occur every 2 minutes or closer and last 60-90 seconds.
Your patient has just delivered a healthy baby boy. Following the delivery of the placenta, her vaginal bleeding seems to increase which of the following best describes what you should do to provide emergency care for this patient? A. Massage the uterus, and position your patient on her side. B. Administer oxygen , and firmly massage the uterus. C. Massage the uterus, and pack the vagina to control bleeding. D. Administer oxygen, and pack the vagina with sanitary pads.
B. Administer oxygen , and firmly massage the uterus.
Which of the following is a sign of an abnormal delivery emergency? A. Fetal presentation of the head B. Colorless amniotic fluid C. Labor before the 46th week of pregnancy D. Recurrence of contractions after the infant is born
D. Recurrence of contractions after the infant is born
Management of a breech birth presentation includes A. Positioning the mother in a supine head-down, pelvis elevated position. B. Administering low-flow oxygen by non rebreather mask. C. Placing a hand in the vagina to delay delivery. D. Delaying transport for safe delivery on the scene.
D. Delaying transport for safe delivery on the scene.
Which of the following best describes an abnormal delivery situation where there may be compression of the cord against the walls of the vagina and the bony pelvis by the pressure of the infant's head or buttocks? A. Crowing. B. Limb presentation. C. Breech. D. Prolapsed cord.
D. Prolapsed cord.
Meconium staining of the amniotic fluid should be managed by A. Immediate transport to the hospital for evaluation by a physician. B. Administration of oxygen to the mothers to resolve fetal distress. C. Suctioning mouth and then nose before the infant takes the first breath. D. Stimulating the infants to cough to clear meconium from the airway.
C. Suctioning mouth and then nose before the infant takes the first breath.
In addition to the usual care for a newborn, care of the premature infant requires A. Vigorous suctioning to maintain the airway. B. Direct administration of supplemental oxygen by a face mask. C. Immediate transport for resuscitation. D. Vigilant attention to prevent heat loss or contamination.
D. Vigilant attention to prevent heat loss or contamination.
The Apgar scoring assessment of the newborn A. Ranges from 0 to 10. B. Includes assessment of the newborn's affect. C. Ranges form 3 to 10. D. Is preformed at 1 and 2 minutes following birth.
A. Ranges from 0 to 10.
Which of the following is a sign of a severely depressed newborn? A. Respiratory rate over 50per minute. B. Apgar score of 7-10 points. C. Cyanotic body (core and extremities). D. Heart rate over 120 per minute or under 80 per minute.
C. Cyanotic body (core and extremities).
You have just assisted with the delivery of a baby girl. She has shallow, gasping respirations and a heart rate that is less than 90 beats per minute. Which of the following best describes your emergency care for this patient? A. Administer "blow-by" oxygen and monitor pulse for 60 seconds. B. Assist ventilations with a bag-valve mask and reassess in 30 seconds. C. Administer high-concentration oxygen with a non rebreather mask. D. Assist ventilations with a bag-valve mask and begin chest compressions.
B. Assist ventilations with a bag-valve mask and reassess in 30 seconds.
Meconium staining of the amniotic fluid should be managed by: A. Immediate transport to the hospital for evaluation by a physician. B. Administration of oxygen to the mothers to resolve fetal distress. C. Suctioning mouth and then nose before the infant takes the first breath. D. Stimulating the infants to cough to clear meconium from the airway.
C. Suctioning mouth and then nose before the infant takes the first breath.
The Apgar score is initially preformed at _________ after birth and repeated at ___________ after birth. A. 30 seconds, 3 minutes. B. 60 Seconds, 5 minutes. C. 90 seconds, 6 minutes. D. 30 seconds, 4 minutes.
B. 60 Seconds, 5 minutes.
Irregular contractions that vary in intensity and duration are called: A. Braxton-hicks contractions. B. Normal pre delivery contractions. C. Braddock-McKoy contractions. D. Post delivery contractions.
A. Braxton-hicks contractions.
Placenta pre via results from: A. Uterine trauma and subsequent placental separation from the uterine wall. B. Rupture of the blood vessels under the placenta and subsequent separation. C. An abnormal implantation of the placenta. D. Hypertension and separation of the placenta from the uterine wall.
C. An abnormal implantation of the placenta.
A predisposing factor for the development of placenta pre via is A. Less than two deliveries. B. Less than 20 years of age. C. No history of vaginal bleeding. D. Bleeding following intercourse.
D. Bleeding following intercourse.
The hallmark sign of placenta pre via is A. Second-trimester painless vaginal bleeding B. Third-trimester painless vaginal bleeding. C. Third-trimester painless vaginal bleeding. D. Third-trimester painless dark red vaginal bleeding.
C. Third-trimester painless vaginal bleeding.
Abruptio placentae results from: A. Uterine trauma and subsequent placental separation from the uterine wall. B. Rupture of blood vessels under the placenta and subsequent separation. C. An abnormal implantation of the placenta. D. Hypertension and separation of the placenta from the uterine wall.
C. An abnormal implantation of the placenta.
Predisposing factors for an abruptio placentae are A. History of fewer than tow deliveries B. An abnormally long umbilical cord. C. Fever and use of psychotropic drugs. D. Smoking and hypertension.
D. Smoking and hypertension.
The hallmark sign of abrupt placentae is A. Vaginal bleeding that is associated with constant abdominal pain. B. Lower abdominal pain without vaginal bleeding. C. Lower back pain and cramping. D. Signs and symptoms of hypovolemic shock.
A. Vaginal bleeding that is associated with constant abdominal pain.
Preeclampsia A. Is a common condition that affects one in five women. B. Occurs most frequently in the first trimester of pregnancy. C. Frequently affects pregnant women over the age of 40 D. Chiefly affects women with a history of hypertension or diabetes.
D. Chiefly affects women with a history of hypertension or diabetes.
The primary clinical signs of preeclampsia are A. Vomiting and pulmonary edema. B. Hypertension and swelling in the extremities. C. Abdominal pain and severe headache. D. Reactive airway disease and pulmonary edema.
B. Hypertension and swelling in the extremities.
The third leading cause of maternal death that is most common in women 25-35 years of age is A. Abruptio Placentae. B. Placenta Previa. C. Preeclampsia/ Eclampsia. D. Ectopic Pregnancy.
D. Ectopic Pregnancy.
Factors that predispose an individual to develop an ectopic pregnancy include A. Smoking. B. Cardiac disease. C. Pelvic inflammatory disease. D. Inactivity.
C. Pelvic inflammatory disease.
A sign or symptom of an ectopic pregnancy emergency is: A. An intense urge to urinate. B. Sharp, knifelike lower abdominal pain localize to one side. C. A bluish discoloration around the umbilicus immediately following rupture. D. Bilateral lower abdominal pain that is described as a tearing pain.
B. Sharp, knifelike lower abdominal pain localize to one side.
During your visual inspection of a 19-year-old woman in labor, you see the baby's head crowning at the vaginal opening. What should you do? A. Apply gentle pressure to the baby's head as it delivers. B. Tell the mother not to push and transport her immediately. C. Place your fingers in the vagina to assess for a nuchal cord. D. Maintain firm pressure to the head until it completely delivers.
A. Apply gentle pressure to the baby's head as it delivers.
Following delivery of a full-term baby, you have properly cared for the baby and have clamped and cut the umbilical cord. During transport, you note that the mother is experiencing moderate vaginal bleeding. You should: A. Elevate her legs 6" to 8" and cover her with a blanket. B. Firmly massage the uterine fundus with a circular motion. C. Carefully insert a sterile trauma dressing into her vagina. D. Place her legs together and position her on her left side.
B. Firmly massage the uterine fundus with a circular motion.
You have just delivered a baby boy. His body is pink, but his hands and feet are blue. His heart rate is approximately 110 beats/min and his respirations are rapid and irregular. He has a weak cry when stimulated and resists attempts to straighten his legs. His Apgar score is: A. 6. B. 7. C. 8. D. 9.
C. 8.
Your 22-year-old patient is in active labor. Upon visual inspection, you note that the infant's leg is protruding from the vagina. Appropriate management of this situation includes: A. Placing the mother supine with her head down and pelvis elevated. B. Gently pulling on the infant's leg in an attempt to facilitate delivery. C. Placing the mother in a recumbent position and rapidly transporting. D. Carefully attempting to push the infant's leg off of the umbilical cord.
A. Placing the mother supine with her head down and pelvis elevated.
Following delivery of a pulseless and apneic infant who has a foul odor, skin sloughing, and diffuse blistering, you should: A. Begin full resuscitation and transport. B. Report the case to the medical examiner. C. Provide emotional support to the mother. D. Dry the infant off to stimulate breathing.
C. Provide emotional support to the mother.
A 33-year-old woman who is 36 weeks pregnant is experiencing scant vaginal bleeding. During transport, you note that she suddenly becomes diaphoretic, tachycardic, and hypotensive. In addition to administering 100% oxygen, you should: A. Place her in a left lateral recumbent position. B. Position her supine and elevate her legs 12. C. Carefully place sterile gauze into her vagina. D. Assist her ventilations with a bag-mask device.
A. Place her in a left lateral recumbent position.
You are assessing a 25-year-old woman who is 39 weeks pregnant. She is experiencing regular contractions that are approximately 3 minutes apart and states that her amniotic sac broke 2 hours ago. After taking the standard precautions, you should: A. Apply 100% oxygen. B. Place her on her left side. C. Assess her for crowning. D. Transport her immediately.
C. Assess her for crowning.
Upon delivery of the baby's head, you note that its face is encased in the unruptured amniotic sac. You should: A. Give the mother 100% oxygen and transport at once. B. Puncture the sac and suction the baby's mouth and nose. C. Leave the amniotic sac intact until arrival at the hospital. D. Note the color of the amniotic fluid before breaking the sac.
B. Puncture the sac and suction the baby's mouth and nose.
While examining a woman in labor, you see the umbilical cord protruding from the vagina. You should: A. Carefully push the cord back into the vagina. B. Push the infant's head away from the cord. C. Cover the umbilical cord with a dry dressing. D. Gently pull on the cord to facilitate delivery.
B. Push the infant's head away from the cord.
You have just delivered a premature baby. Your assessment reveals that he is breathing adequately; however, his heart rate is 90 beats/min. You should: A. Keep him warm and provide ventilatory assistance. B. Begin chest compressions and reassess in 30 seconds. C. Clamp and cut the umbilical cord and keep him warm. D. Assess his skin color and give free-flow oxygen as needed.
A. Keep him warm and provide ventilatory assistance.
Fetal complications associated with drug- or alcohol-addicted mothers include all of the following, EXCEPT: A. Profound Tachycardia. B. Premature Delivery. C. Low Birth Weight. D. Respiratory Depression.
A. Profound Tachycardia
Which of the following statements regarding gestational diabetes is correct: A. Diet and exercise are typically ineffective in controlling gestational diabetes. B. The onset of gestational diabetes typically manifests with hypoglycemia. C. Gestational diabetes results in permanent diabetes mellitus after delivery. D. In some cases, women with gestational diabetes require insulin injections.
D. In some cases, women with gestational diabetes require insulin injections.
Most medical models base a pregnant woman's due date: A. Two weeks after her last menstrual cycle. B. Two weeks before her last menstrual cycle. C. On the last day of her last menstrual cycle. D. On the first day of her last menstrual cycle.
D. On the first day of her last menstrual cycle.
Which of the following is NOT a component of the APGAR score? A. Activity. B. Grimace. C. Pulse. D. Body Size.
D. Body Size.
Common interventions used to stimulate spontaneous respirations in the newborn include all of the following, EXCEPT: A. Through drying with a towel. B. Suctioning of the upper airway. C. Some form of tactile stimulation. D. Positive-pressure ventilations.
D. Positive-pressure ventilations.
Which of the following questions is of LEAST pertinence when determining whether a mother will deliver her baby within the next few minutes? A. "Have you had a sonogram?" B. "Is this your first baby?" C. "When are you due?" D. "Do you feel the urge to push?"
A. "Have you had a sonogram?"
Eclampsia is MOST accurately defined as: A. A blood pressure greater than 140/90 mm Hg. B. Seizures that result from severe hypertension. C. High levels of protein in the patient's urine. D. Hypertension in the 20th week of pregnancy.
B. Seizures that result from severe hypertension.
Following delivery of a full-term baby, you have properly cared for the baby and have clamped and cut the umbilical cord. During transport, you note that the mother is experiencing moderate vaginal bleeding. You should: A. Elevate her legs 6″ to 8″ and cover her with a blanket. B. Place her legs together and position her on her left side. C. Carefully insert a sterile trauma dressing into her vagina. D. Firmly massage the uterine fundus with a circular motion.
D. Firmly massage the uterine fundus with a circular motion.
Rough handling of a hypothermic patient with a pulse may cause: A. Ventricular fibrillation. B. Profound bradycardia. C. Pulseless electrical activity. D. Ventricular tachycardia.
A. Ventricular fibrillation.
Drowning is MOST accurately defined as: A. Temporary survival after submersion in water. B. Water in the lungs following submersion in water. C. Death beyond 24 hours after submersion in water. D. Death from suffocation after submersion in water.
D. Death from suffocation after submersion in water.
The EMT must assume that any unwitnessed water-related incident is accompanied by: A. Cold water immersion. B. Possible spinal injury. C. An air embolism. D. Alcohol intoxication.
B. Possible spinal injury.
Common interventions used to stimulate spontaneous respirations in the newborn include all of the following, EXCEPT: A. Suctioning of the upper airway. B. Positive-pressure ventilations. C. Thorough drying with a towel. D. Some form of tactile stimulation.
B. Positive-pressure ventilations.
A 48-year-old male was stung on the leg by a jellyfish while swimming in the ocean. He is conscious and alert, but complains of intense pain at the wound site. Specific treatment for this patient includes: A. Applying a chemical ice pack to the wound and encouraging movement. B. Pulling the nematocysts out with tweezers and bandaging the wound. C. Irrigating the wound with vinegar and immersing his leg in hot water. D. Immersing his leg in fresh cold water and scraping away the stingers.
C. Irrigating the wound with vinegar and immersing his leg in hot water.
A 20-year-old male was pulled from cold water by his friends. The length of his submersion is not known and was not witnessed. You perform a primary assessment and determine that the patient is apneic and has a slow, weak pulse. You should: A. Ventilate with a bag-mask device, apply a cervical collar, remove his wet clothing, and transport rapidly. B. Apply 100% oxygen via a nonrebreathing mask, immobilize his spine, keep him warm, and transport rapidly. C. Provide rescue breathing, remove wet clothing, immobilize his spine, keep him warm, and transport carefully. D. Suction his airway for 30 seconds, provide rescue breathing, keep him warm, and transport at once.
C. Provide rescue breathing, remove wet clothing, immobilize his spine, keep him warm, and transport carefully.
Most of the serious injuries associated with scuba diving are caused by: A. Water temperature lower than 70°F (21°C). B. Too rapid of an ascent. C. Alcohol consumption. D. Too rapid of a descent.
B. Too rapid of an ascent.
While drinking beer with his friends near a creek, a 31-year-old male was bitten on the leg by an unidentified snake. The patient is conscious and alert and in no apparent distress. Your assessment of his leg reveals two small puncture marks with minimal pain and swelling. In addition to administering oxygen and providing reassurance, further care for this patient should include: A. Transporting only with close, continuous monitoring. B. Supine positioning, splinting the leg, and transporting. C. Elevating the lower extremities and giving antivenin. D. Applying ice to the wound and transporting quickly.
B. Supine positioning, splinting the leg, and transporting.
A person's ability to shiver is lost when his or her body temperature falls below: A. 92°F (33°C). B. 95°F (35°C). C. 90°F (32°C). D. 94°F (34°C).
C. 90°F (32°C).
Which of the following statements regarding gestational diabetes is correct? A. In some cases, women with gestational diabetes require insulin injections. B. The onset of gestational diabetes typically manifests with hypoglycemia. C. Diet and exercise are typically ineffective in controlling gestational diabetes. D. Gestational diabetes results in permanent diabetes mellitus after delivery.
A. In some cases, women with gestational diabetes require insulin injections.
Common signs and symptoms of heat exhaustion include all of the following, EXCEPT: A. Headache. B. Hot, dry skin. C. Tachycardia. D. Nausea.
B. Hot, dry skin.
You and your partner respond to a park where several people were reportedly struck by lightning. When you arrive, you find three patients. The first patient is lying supine on the ground; he is unresponsive and does not appear to be breathing. The second patient is ambulatory, appears confused, and is holding his arm against his chest. The third patient is sitting on the ground holding the sides of his head. After calling for backup, you should: A. Recognize that the patients who are conscious are at high risk for developing cardiac arrest and quickly assess them for potentially life-threatening injuries. B. Open the unresponsive patient's airway with the jaw-thrust maneuver, assess his ABCs, and begin CPR and attach the AED as soon as possible. C. Immediately begin CPR on the unresponsive patient, but cease resuscitation efforts if there is no response after 5 minutes of treatment. D. Focus your initial treatment efforts on the patients who are conscious because the unresponsive patient is likely in irreversible cardiac arrest.
B. Open the unresponsive patient's airway with the jaw-thrust maneuver, assess his ABCs, and begin CPR and attach the AED as soon as possible.
A nuchal cord is defined as an umbilical cord that: A. Has abnormally developed blood vessels. B. Is wrapped around the baby's neck. C. Has separated from the placenta. D. Is lacerated due to a traumatic delivery.
B. Is wrapped around the baby's neck.
You have just delivered a premature baby. Your assessment reveals that he is breathing adequately; however, his heart rate is 90 beats/min. You should: A. Assess his skin color and give free-flow oxygen as needed. B. Begin chest compressions and reassess in 30 seconds. C. Clamp and cut the umbilical cord and keep him warm. D. Keep him warm and provide ventilatory assistance.
D. Keep him warm and provide ventilatory assistance.
The onset of labor begins with: A. Contractions of the uterus. B. Full dilation of the cervix. C. Thinning of the uterus. D. Increased fetal movement.
A. Contractions of the uterus.
Your assessment of a 23-year-old female reveals a core body temperature of 93.4°F (34°C). She is conscious, answers your questions appropriately, is shivering, and complains of nausea. Her skin is cold and pale, her muscles appear rigid, and her respirations are rapid. In addition to monitoring her ABCs, administering oxygen, and turning up the heat in the back of the ambulance, you should: A. Apply the AED in case she develops cardiopulmonary arrest, cover her with layers of blankets, and transport carefully. B. Cover her with warm blankets and let her move about on the stretcher in order to generate body heat and increase her temperature. C. Place heat packs to her groin, axillae, and behind her neck; cover her with warm blankets; and avoid rough handling. D. Sit her up and give her small sips of warm water to drink, place heat packs to her axillae and groin, and cover her with blankets.
C. Place heat packs to her groin, axillae, and behind her neck; cover her with warm blankets; and avoid rough handling.
The term primigravida refers to a woman who: A. Has had only one live birth. B. Is pregnant for the first time. C. Has had more than one live baby. D. Has never been pregnant.
B. Is pregnant for the first time.
A 33-year-old woman who is 36 weeks pregnant is experiencing scant vaginal bleeding. During transport, you note that she suddenly becomes diaphoretic, tachycardic, and hypotensive. In addition to administering 100% oxygen, you should: A. Carefully place sterile gauze into her vagina. B. Assist her ventilations with a bag-mask device. C. Position her supine and elevate her legs 12″. D. Place her in a left lateral recumbent position.
D. Place her in a left lateral recumbent position.
If a newborn's heart rate is less than 60 beats/min following delivery, you should: A. Begin chest compressions. B. Provide ventilations for 30 seconds. C. Resuction the mouth only. D. Flick the soles of its feet.
B. Provide ventilations for 30 seconds.
If a pregnant patient requires spinal immobilization, you should secure her to the backboard and then: A. Elevate the head of the board 6″ to prevent breathing impairment. B. Elevate the right side of the board with rolled towels or blankets. C. Tilt the board 30° to the right to prevent hypotension. D. Raise the foot of the board 12″ in order to maintain blood pressure.
B. Elevate the right side of the board with rolled towels or blankets.
You are dispatched to a residence for a young female who is sick. The patient complains of a rash to her lower extremities and truncal area. Your assessment reveals a small, painful blister on her inner thigh. As your partner is taking the patient's vital signs, she states that she and her family returned from a camping trip two days ago. On the basis of this patient's presentation, you should suspect: A. Lyme disease. B. Exposure to poison ivy. C. Rocky Mountain spotted fever. D. An allergic reaction.
A. Lyme disease.
The two MOST efficient ways for the body to eliminate excess heat are: A. Sweating and dilation of skin blood vessels. B. Respiration and bradycardia. C. Hyperventilation and tachycardia. D. Perspiration and tachycardia.
A. Sweating and dilation of skin blood vessels.
Upon delivery of an infant's head, you note that the umbilical cord is wrapped around its neck. You should: A. Immediately clamp and cut the umbilical cord. B. Attempt to slip the cord gently over the infant's head. C. provide free-flow oxygen to the infant and transport. D. Give 100% oxygen to the mother and transport at once.
B. Attempt to slip the cord gently over the infant's head.
You have just delivered a baby boy. His body is pink, but his hands and feet are blue. His heart rate is approximately 110 beats/min and his respirations are rapid and irregular. He has a weak cry when stimulated and resists attempts to straighten his legs. His Apgar score is: A. 9. B. 6. C. 8. D. 7.
C. 8.
When determining the frequency of contractions, you should time the contractions from the: A. Start of one to the start of the next. B. End of one to the start of the next. C. Start of one to the end of the next. D. End of one to the end of the next.
A. Start of one to the start of the next.
Breath-holding syncope is caused by a decreased stimulus to breathe and occurs when: A. A diver holds his or her breath for a long period of time. B. A diver holds his or her breath during a staged ascent. C. A swimmer hyperventilates prior to entering the water. D. A swimmer breathes shallowly before entering the water.
C. A swimmer hyperventilates prior to entering the water.
Hypothermia can worsen internal bleeding secondary to: A. A decreased heart rate. B. Severe muscular rigidity. C. Cardiac arrhythmias. D. Blood clotting abnormalities.
D. Blood clotting abnormalities.
You receive a call to a residence for a sick patient. Upon your arrival, you find the patient, a 53-year-old diabetic male, lying down on his front porch. His wife tells you that he had been mowing the lawn in the heat for the past 3 hours. The patient is confused and has hot, moist skin. His pulse is weak and thready, and his blood pressure is 90/50 mm Hg. In addition to administering 100% oxygen, you should: A. Place him in a sitting position and have him drink 1 L of water. B. Administer one tube of oral glucose and reassess his mental status. C. Perform a rapid assessment and look for signs of trauma. D. Load him into the ambulance and begin rapid cooling interventions.
D. Load him into the ambulance and begin rapid cooling interventions.
Which of the following conditions would be the LEAST likely to increase a person's risk of hypothermia? A. Hyperglycemia. B. Spinal cord injury. C. Severe infection. D. Hypoperfusion.
A. Hyperglycemia.
Braxton-Hicks contractions are characterized by: A. Alleviation of pain with movement or changing positions. B. Regular contractions of progressively increasing intensity. C. Pink or red bloody show in conjunction with the contractions. D. A rupture of the amniotic sac just before the contractions begin.
A. Alleviation of pain with movement or changing positions.
A dysbarism injury refers to the signs and symptoms related to changes in: A. Rapid descent. B. Rapid ascent. C. Decompression. D. Barometric pressure.
D. Barometric pressure.
During delivery of the baby's head, you should suction the mouth before the nose because: A. Babies are primarily mouth breathers and do not breathe through their nose. B. It is easier to suction larger volumes of fluid from the baby's oropharynx. C. The mucosa of the nose is fragile and is easily damaged by vigorous suctioning. D. Suctioning the nose first may cause the baby to gasp and aspirate fluid.
D. Suctioning the nose first may cause the baby to gasp and aspirate fluid.
Geriatric patients are at a higher risk for heatstroke because: A. Their ability to shiver is reduced. B. Most geriatric patients are obese. C. Their ability to sweat is enhanced. D. Circulation to the skin is reduced.
D. Circulation to the skin is reduced.
While examining a woman in labor, you see the umbilical cord protruding from the vagina. You should: A. Gently pull on the cord to facilitate delivery. B. Carefully push the cord back into the vagina. C. Cover the umbilical cord with a dry dressing. D. Push the infant's head away from the cord.
D. Push the infant's head away from the cord.
In contrast to the brown recluse spider, the black widow spider: A. Has a bite that is typically painless until a blister develops. B. Has a bite that usually produces local pain but no systemic signs or symptoms. C. Is large and has a red-orange hourglass mark on its abdomen. D. Is very small and has a violin-shaped marking on its back.
C. Is large and has a red-orange hourglass mark on its abdomen.
Abruptio placenta occurs when: A. The placenta develops over and covers the cervical opening. B. The placenta affixes itself to the outer layer of the uterus. C. A tear in the placenta causes severe internal hemorrhage. D. The placenta prematurely separates from the uterine wall.
D. The placenta prematurely separates from the uterine wall.
The amniotic fluid serves to: A. Assist in fetal development. B. Insulate and protect the fetus. C. Transfer oxygen to the fetus. D. Remove viruses from the fetus.
B. Insulate and protect the fetus.
Which of the following statements regarding a breech presentation is MOST correct? A. Breech deliveries occur rapidly, so the EMT should deliver at the scene. B. It is impossible to deliver a breech presentation in the prehospital setting. C. A breech presentation occurs when the buttocks are the presenting part. D. There is minimal risk of trauma to the infant with a breech presentation.
C. A breech presentation occurs when the buttocks are the presenting part.
When a warm hand is immersed in water that is 70°F, heat is transferred from the hand to the water through a process called: A. Radiation. B. Conduction. C. Convection. D. Evaporation.
B. Conduction.
The body's natural cooling mechanism, in which sweat is converted to a gas, is called: A. Radiation. B. Convection. C. Conduction. D. Evaporation.
D. Evaporation.
Heat loss from the body through respiration occurs when: A. Warm air is exhaled into the atmosphere. B. The core body temperature is above 98°F. C. Cool air is inhaled and displaces warm air. D. Air temperature is above body temperature.
A. Warm air is exhaled into the atmosphere.
Hypothermia occurs when the core body temperature falls below: A. 98°F. B. 95°F. C. 90°F. D. 88°F.
B. 95°F.
The transfer of heat to circulating air, such as when cool air moves across the body's surface, is called: A. Radiation. B. Conduction. C. Convection. D. Evaporation.
C. Convection.
Shivering is a mechanism in which the body generates heat by: A. Increasing the metabolic rate. B. Decreasing the use of oxygen. C. Decreasing the metabolic rate. D. Retaining excess carbon dioxide.
A. Increasing the metabolic rate.
Covering a patient's ________ will significantly minimize radiation heat loss. A. Head. B. Chest. C. Abdomen. D. Extremities.
A. Head.
The body's natural protective mechanisms against heat loss are: A. Shivering and vasodilation. B. Vasodilation and respiration. C. Respiration and vasoconstriction. D. Vasoconstriction and shivering.
D. Vasoconstriction and shivering.
Which of the following conditions would be the LEAST likely to increase a person's risk of hypothermia? A. Hypoperfusion. B. Severe infection. C. Hyperglycemia. D. Spinal cord injury.
C. Hyperglycemia.
A patient with a core body temperature of 92°F will MOST likely experience: A. Bradycardia. B. Rapid Breathing. C. Muscle Stiffness. D. Loss of Consciousness.
B. Rapid Breathing.
A person's ability to shiver is lost when his or her body temperature falls below: A. 90°F. B. 92°F. C. 95°F. D. 97°F.
A. 90°F.
Compared to adults, infants and children are at higher risk for hypothermia for all of the following reasons, EXCEPT: A. A decreased ability to shiver. B. A relatively small surface area. C. A smaller amount of body fat. D. A smaller overall muscle mass.
B. A relatively small surface area.
Which of the following would be the LEAST likely to occur in a patient with a core body temperature of between 89°F and 92°F? A. Bradypnea. B. Confusion. C. Stiff muscles. D. Tachycardia.
D. Tachycardia.
To assess a patient's general temperature, pull back on your glove and place the back of your hand on his or her skin at the: A. Neck. B. Chest. C. Abdomen. D. Forehead.
C. Abdomen.
When assessing a hypothermic patient, you should palpate for a carotid pulse for approximately __________ seconds before determining that he or she is pulseless. A. 10 to 20. B. 20 to 30. C. 30 to 45. D. 60 to 90.
C. 30 to 45.
Hypothermia can worsen internal bleeding secondary to: A. Cardiac arrhythmias. B. A decreased heart rate. C. Severe muscular rigidity. D. Blood-clotting abnormalities.
D. Blood-clotting abnormalities.
Rough handling of a hypothermic patient with a pulse may cause: A. Profound bradycardia. B. Ventricular fibrillation. C. Ventricular tachycardia. D. Pulseless electrical activity.
B. Ventricular fibrillation.
To obtain the MOST accurate reading of a patient's core body temperature, you should place a special hypothermia thermometer: A. Into the patient's rectum. B. Under the patient's tongue. C. Behind the patient's knee. D. Under the patient's armpit.
A. Into the patient's rectum.
All of the following terms are used to describe a cold body part that is not frozen, EXCEPT: A. Frostnip. B. Frostbite. C. Chilblains. D. Trench Foot.
B. Frostbite.
Patients with generalized hypothermia are at an increased risk of a local cold injury because: A. Blood is shunted away from the extremities to the body's core. B. Peripheral vasodilation brings warm blood to the skin's surface. C. The major muscles of the body become rigid during hypothermia. D. The patient is usually unable to escape the cold ambient temperature.
A. Blood is shunted away from the extremities to the body's core.
A frostbitten foot can be identified by the presence of: A. Gross deformity. B. Soft, smooth skin. C. Mottling and blisters. D. Blanching of the skin.
C. Mottling and blisters.
The two MOST efficient ways for the body to eliminate excess heat are: A. Respiration and Bradycardia. B. Perspiration and Tachycardia. C. Sweating and Vessel Dilation. D. Hyperventilation and Tachycardia.
C. Sweating and Vessel Dilation.
High air temperature reduces the body's ability to lose heat by: A. Radiation. B. Convection. C. Conduction. D. Evaporation.
A. Radiation.
High humidity reduces the body's ability to lose heat through: A. Radiation. B. Convection. C. Conduction. D. Evaporation.
D. Evaporation.
Hyperthermia is MOST accurately defined as a condition in which: A. The core body temperature exceeds 98.6°F. B. The body is exposed to more heat than it can lose. C. Heat evaporates a significant amount of body water. D. The body eliminates more heat than it can generate.
B. The body is exposed to more heat than it can lose.
When the body loses sweat, it also loses: A. Plasma. B. Nutrients. C. Erythrocytes. D. Electrolytes.
D. Electrolytes.
Geriatric patients, newborns, and infants are especially prone to hyperthermia because they: A. Have relatively smaller heads. B. Have less body fat. C. Exhibit poor thermoregulation. D. Have smaller body surface areas.
C. Exhibit poor thermoregulation.
Heat cramps are MOST likely the result of: A. A loss of water and electrolytes. B. Complete thermoregulatory failure. C. Carbon dioxide buildup in the muscles. D. Stress on the muscles from excess exertion.
A. A loss of water and electrolytes.
Common signs and symptoms of heat exhaustion include all of the following, EXCEPT: A. Nausea. B. Headache. C. Tachycardia. D. Hot, dry skin.
D. Hot, dry skin.
In order for sweating to be an effective cooling mechanism: A. Several layers of clothing must be worn. B. It must be able to evaporate from the body. C. The relative humidity must be above 90%. D. The body must produce at least 1L per hour.
B. It must be able to evaporate from the body.
Heatstroke occurs when: A. A person's core body temperature rises above 103°F. B. The body's heat-eliminating mechanisms are overwhelmed. C. A person becomes dehydrated secondary to excess water loss. D. The ambient temperature exceeds 90°F and the humidity is high.
B. The body's heat-eliminating mechanisms are overwhelmed.
Signs of late heatstroke include: A. Hot, moist skin. B. Nausea and vomiting. C. A weak, rapid pulse. D. A change in behavior.
C. A weak, rapid pulse.
Geriatric patients are at a higher risk for heatstroke because: A. Circulation to the skin is reduced. B. Most geriatric patients are obese. C. Their ability to sweat is enhanced. D. Their ability to shiver is reduced.
A. Circulation to the skin is reduced.
Which of the following medications increases a person's risk of a heat-related emergency? A. Motrin. B. Tylenol. C. Aspirin. D. Diuretics.
D. Diuretics.
Drowning is MOST accurately defined as: A. Temporary survival after submersion in water. B. Death from suffocation after submersion in water. C. Water in the lungs following submersion in water. D. Death beyond 24 hours after submersion in water.
B. Death from suffocation after submersion in water.
Which of the following statements regarding drowning is MOST correct? A. Hypoxia in the drowning victim initially occurs due to water in the lungs. B. Artificial ventilations can easily be performed in patients with a laryngospasm. C. Large amounts of water enter the lungs in a small number of drowning victims. D. Laryngospasm following submersion in water makes rescue breathing difficult.
D. Laryngospasm following submersion in water makes rescue breathing difficult.
The EMT-B must assume that any unwitnessed water-related incident is accompanied by: A. An air embolism. B. Alcohol intoxication. C. Possible spinal injury. D. Cold water immersion.
C. Possible spinal injury.
The diving reflex may allow a person to survive extended periods of submersion in cold water secondary to: A. Bradycardia and a slowing of the metabolic rate. B. Laryngospasm that protects the lungs from water. C. Tachycardia and a lowering of the blood pressure. D. Increases in the metabolic rate and oxygen demand.
A. Bradycardia and a slowing of the metabolic rate.
Most of the serious injuries associated with scuba diving are caused by: A. Water less than 70°F. B. Too rapid of a descent. C. Alcohol consumption. D. Too rapid of an ascent.
D. Too rapid of an ascent.
An air embolism associated with diving occurs when: A. The diver hyperventilates prior to entering the water. B. The diver holds his or her breath during a rapid ascent. C. The alveoli completely collapse due to high pressure. D. High water pressure forces air into the mediastinum.
B. The diver holds his or her breath during a rapid ascent.
Signs and symptoms of an air embolism include all of the following, EXCEPT: A. Dysphasia. B. Pale skin. C. Dizziness. D. Joint pain.
B. Pale skin.
The MOST prominent symptom of decompression sickness is: A. Tightness in the chest. B. Difficulty with vision. C. Dizziness and nausea. D. Abdominal or joint pain.
D. Abdominal or joint pain.
Breath-holding syncope is caused by a decreased stimulus to breathe and occurs when: A. A swimmer breathes shallowly before entering the water. B. A diver holds his or her breath during a staged ascent. C. A swimmer hyperventilates prior to entering the water. D. A diver holds his or her breath for a long period of time.
C. A swimmer hyperventilates prior to entering the water.
Burns associated with lightning strikes are typically: A. Superficial. B. Third-degree. C. Full-thickness. D. Partial-thickness.
A. Superficial.
Which of the following statements regarding lightning strikes is MOST correct? A. Lighting often results in a brief period of systole that resolves spontaneously. B. The tissue damage pathway caused by lightning usually occurs through the skin. C. Victims who are struck by lightning often experience severe full-thickness burns. D. Cervical spine fractures are the most common cause of lightning-related deaths.
A. Lighting often results in a brief period of asystole that resolves spontaneously.
The venom of a black widow spider is toxic to the: A. Renal system. B. Nervous system. C. Respiratory system. D. Cardiovascular system.
B. Nervous system.
The venom of a brown recluse spider is cytotoxic, meaning that it: A. Suppresses the respiratory drive. B. Destroys the body's red blood cells. C. Weakens the structure of the bones. D. Causes severe local tissue damage.
D. Causes severe local tissue damage.
In contrast to the brown recluse spider, the black widow spider: A. Is very small and has a violin-shaped marking on its back. B. Is large and has a red-orange hourglass mark on its abdomen. C. Produces a bite that is typically painless until a blister develops. D. Usually produces local pain but no systemic signs or symptoms.
B. Is large and has a red-orange hourglass mark on its abdomen.
All of the following snakes are pit vipers, EXCEPT for the: A. Copperhead. B. Rattlesnake. C. Coral Snake. D. Cottonmouth.
C. Coral Snake.
Which of the following is an early sign of pit viper envenomation? A. Local swelling and ecchymosis. B. General weakness and diaphoresis. C. Syncope and bleeding at distal sites. D. Signs and symptoms of hypo perfusion.
A. Local swelling and ecchymosis.
A 30-year-old male was rescued after being lost in the woods for approximately 18 hours. The outside temperature is 30°F. He is immediately placed in the warmed ambulance, where you perform an initial assessment. He is unconscious, pale, and apneic. After initiating artificial ventilations, you should: A. Apply an AED and assess his cardiac rhythm. B. Assess for a carotid pulse for up to 45 seconds. C. Begin chest compressions and transport at once. D. Apply chemical heat packs to his groin and axillae.
B. Assess for a carotid pulse for up to 45 seconds.
You are dispatched to a local high school track and field event for a 16-year-old male who fainted. The outside temperature is approximately 95°F with high humidity. Upon your arrival, the patient is conscious, alert, and complains of nausea and a headache. His skin is cool, clammy, and pale. You should: A. Give him a liquid salt solution to drink. B. Administer 100% supplemental oxygen. C. Apply chemical ice packs to his axillae. D. Move him into the cooled ambulance.
D. Move him into the cooled ambulance.
You receive a call to a residence for a "sick patient." Upon your arrival, you find the patient, a 53-year-old diabetic male, lying down on his front porch. His wife tells you that he had been mowing the lawn in the heat for the past 3 hours. The patient is confused and has hot, moist skin. His pulse is weak and thready and his BP is 90/50 mm Hg. In addition to administering 100% oxygen, you should: A. Perform a rapid medical assessment and look for signs of trauma. B. Place him in a sitting position and have him drink 1 liter of water. C. Load him into the ambulance and begin rapid cooling interventions. D. Administer one tube of oral glucose and reassess his mental status.
C. Load him into the ambulance and begin rapid cooling interventions.
You and your partner are standing by at a large social event at a river resort when a frantic woman tells you that she found a young male floating face-down in the water. Nobody claims to have witnessed the event. After you and your partner enter the water and reach the patient, you should: A. Move him as a unit to a supine position. B. Begin ventilations with a barrier device. C. Immediately secure him to a longboard. D. Perform a jaw-thrust to open his airway.
A. Move him as a unit to a supine position.
You are assessing a 33-year-old male who complains of severe abdominal pain, weakness, and nausea. He tells you that he was gathering wood to build a fire when he felt a sudden, sharp pain on the back of his hand. Your assessment reveals that the patient's abdomen is rigid and painful to palpation. You should suspect: A. A black widow spider bite. B. Envenomation from a pit viper. C. A brown recluse spider bite. D. Rocky mountain spotted fever.
A. A black widow spider bite.
Approximately 12 hours after scuba diving with her friends, a 29-year-old female presents with pain in her elbows and knees. She is conscious and alert and is breathing with adequate tidal volume. When asked, she states that she may have ascended too rapidly during her dive, but didn't experience any symptoms until now. When treating this patient, you should: A. Position her supine with her legs elevated 6" and her head down. B. Place her in a left lateral recumbent position with her head down. C. Administer supplemental oxygen via a nasal cannula at 2 to 4 L/min. D. Provide ventilatory assistance with a BVM device and 100% oxygen.
B. Place her in a left lateral recumbent position with her head down.
While drinking beer with his friends near a creek, a 31-year-old male was bitten on the leg by an unidentified snake. The patient is conscious and alert and in no apparent distress. Your assessment of his leg reveals two small puncture marks with minimal pain and swelling. In addition to administering oxygen and providing reassurance, further care for this patient should include: A. Applying ice to the wound and transporting quickly. B. Transporting only with close, continuous monitoring. C. Elevating the lower extremities and giving antivenin. D. Supine positioning, splinting the leg, and transporting.
D. Supine positioning, splinting the leg, and transporting.
A 20-year-old male was pulled from cold water by his friends. The length of his submersion is not known and was not witnessed. You perform an initial assessment and determine that the patient is apneic and has a slow, weak pulse. You should: A. Suction his airway for 30 seconds, provide rescue breathing, keep him warm, and transport at once. B. Ventilate with a BVM device, apply a cervical collar, remove his wet clothing, and transport rapidly. C. Provide rescue breathing, remove wet clothing, immobilize his spine, keep him warm, and transport carefully. D. Apply 100% oxygen via a nonrebreathing mask, immobilize his spine, keep him warm, and transport rapidly.
C. Provide rescue breathing, remove wet clothing, immobilize his spine, keep him warm, and transport carefully.
You are dispatched to a residence for a young female who is "sick." The patient complains of a rash to her lower extremities and truncal area. Your assessment reveals a small, painful blister on her inner thigh. As your partner is taking the patient's vital signs, she states that she and her family returned from a camping trip 2 days ago. On the basis of this patient's presentation, you should suspect: A. Lyme disease. B. An allergic reaction. C. Exposure to poison ivy. D. Rocky mountain spotted fever.
A. Lyme disease.
A 48-year-old male was stung on the leg by a jellyfish while swimming in the ocean. He is conscious and alert, but complains of intense pain at the wound site. Specific treatment for this patient includes: A. Irrigating the wound with vinegar and immersing his leg in hot water. B. Pulling the nematocysts out with tweezers and bandaging the wound. C. Immersing his leg in fresh cold water and scraping away the stingers. D. Applying a chemical ice pack to the wound and encouraging movement.
A. Irrigating the wound with vinegar and immersing his leg in hot water.
Which of the following statements regarding drowning is correct? A. Hypoxia in the drowning victim initially occurs due to water in the lungs. B. Laryngospasm following submersion in water makes rescue breathing difficult. C. Artificial ventilations can easily be performed in patients with a laryngospasm. D. Large amounts of water enter the lungs in a small number of drowning victims.
B. Laryngospasm following submersion in water makes rescue breathing difficult.
Which of the following would be the LEAST likely to occur in a patient with a core body temperature of between 89°F (32°C) and 92°F (33°C)? A. Bradypnea. B. Stiff Muscles. C. Confusion. D. Tachycardia.
D. Tachycardia.
Which of the following conditions would be the LEAST likely to increase a person's risk of hypothermia? A. Hyperglycemia. B. Severe Infection. C. Hypoperfusion. D. Spinal Cord Injury.
A. Hyperglycemia.
All of the following snakes are pit vipers, EXCEPT for the: A. Copperhead. B. Coral Snake. C. Cottonmouth. D. Rattlesnake.
B) Coral Snake.
Which of the following MOST accurately describes hyperthermia? A. The body is exposed to more heat than it can lose. B. Heat evaporates a significant amount of body water. C. The core body temperature exceeds 99.5°F (37°C). D. The body eliminates more heat than it can generate.
A. The body is exposed to more heat than it can lose.
All of the following terms are used to describe a cold body part that is not frozen, EXCEPT: A. Chilblains. B. Frostnip. C. Frostbite. D. Trench Foot.
C. Frostbite.
Compared to adults, infants and children are at higher risk for hypothermia for all of the following reasons, EXCEPT: A. A decreased ability to shiver. B. A relatively small surface area. C. A smaller amount of body fat. D. A smaller overall muscle mass.
B. A relatively small surface area.
Which of the following is an early sign of pit viper envenomation? A. Local swelling and ecchymosis. B. Signs and symptoms of hypo perfusion. C. General weakness and diaphoresis. D. Syncope and bleeding at distal sites.
A. Local swelling and ecchymosis.
___________ causes body heat to be lost as warm air in the lungs is exhaled into the atmosphere and cooler air is inhaled. A. Convection. B. Conduction. C. Radiation. D. Respiration.
D. Respiration.
Evaporation, the conversion of a liquid to a gas, is a process that requires: A. Energy. B. Circulating Air. C. A warmer ambient temperature. D. High Humidity.
A. Energy.
The rate and amount of heat loss by the body can be modified by all of the following except: A. Increasing heat production. B. Moving to an area where heat loss is decreased. C. Wearing insulated clothing. D. Increasing fluid intake.
D. Increasing fluid intake.
The characteristic appearance of blue lips and/or fingertips seen in hypothermia is the result of: A. Lack of oxygen in venous blood. B. Frostbite. C. Blood vessels constricting. D. Bruising.
C. Blood vessels constricting
Signs and symptoms of severe systemic hypothermia include all of the following except: A. Weak pulse. B. Coma. C. Shivering. D. Very slow respirations.
C. Shivering.
Hypothermia is more common among all of the following except: A. Older individuals. B. Long-distance athletes. C. Infants and children. D. Those who are already ill.
B. Long-distance athletes.
To assess a patient's general temperature, pull back your glove and place the back of your hand on the patient's: A. Abdomen, underneath clothing. B. Forehead. C. Forearm, on the inside of the wrist. D. Neck, at the area where you check the carotid pulse.
A. Abdomen, underneath clothing.
Never assume that a(n) __________, pulseless patient is dead. A. Apneic. B. Cyanotic. C. Cold. D. Hypothermic.
C. Cold.
Management of hypothermia in the field consists of all of the following except: A. Applying heat packs to the groin, axillary, and cervical regions B. Removing wet clothing C. Preventing further heat loss D. Massaging the cold extremities.
D. Massaging the cold extremities.
All of the following conditions refer to when exposed parts of the body become very cold, but not frozen, except: A. Frostnip. B. Trench Foot. C. Immersion Foot. D. Frostbite.
D. Frostbite.
When the body is exposed to more heat energy than it loses, __________ result(s). A. Hyperthermia. B. Heat cramps. C. Heat exhaustion. D. Heatstroke.
A. Hyperthermia.
Contributing factors to the development of heat illnesses include all of the following except: A. High air temperature. B. Vigorous exercise. C. High humidity. D. Increased fluid intake.
D. Increased fluid intake.
It is important to remain hydrated while on duty. Drink at least ________ of water per day, and more when exertion or heat is involved. A. 8 glasses. B. 1 liter. C. 2 liters. D. 3 liters.
D. 3 liters.
Which of the following statements about heat cramps is false? A. They only occur when it is hot outdoors. B. They may be seen in well-conditioned athletes. C. The exact cause of heat cramps is not well understood. D. Dehydration may play a role in the development of heat cramps.
A. They only occur when it is hot outdoors.
Signs and symptoms of heat exhaustion and associated hypovolemia include all of the following except: A. Cold, clammy skin with ashen pallor. B. Dizziness, weakness, or faintness. C. Normal vital signs. D. Normal thirst.
D. Normal thirst.
Most spinal injuries in diving incidents affect the: A. Cervical spine. B. Thoracic spine. C. Lumbar spine. D. Sacrum/coccyx.
A. Cervical spine.
Often, the first sign of heatstroke is: A. A change in behavior. B. An increase in pulse rate. C. An increase in respirations. D. Hot, dry, flushed skin.
A. A change in behavior.
The least common but most serious illness caused by heat exposure, occurring when the body is subjected to more heat than it can handle and normal mechanisms for getting rid of the excess heat are overwhelmed, is: A. Hyperthermia. B. Heat cramps. C. Heat exhaustion. D. Heatstroke.
D. Heatstroke.
_________ is the body's reaction to an irritation of water entering the lower respiratory tract. A. Bronchoconstriction. B. Laryngospasm. C. Esophageal Spasms. D. Swelling in the oropharynx.
B. Laryngospasm.
Treatment of drowning and/or near drowning begins with: A. Opening the airway. B. Ventilation with 100% oxygen via bag-mask device. C. Suctioning the lungs to remove the water. D. Rescue and removal from the water.
D. Rescue and removal from the water.
In a diving emergency, ___________ occurs when bubbles of gas, especially nitrogen, obstruct the blood vessels. A. Compression sickness. B. Decompression sickness. C. Pulmonary sickness. D. Nitrogen toxicity.
B. Decompression sickness.
If the near drowning victim has evidence of upper airway obstruction by foreign matter, which of the following would not be considered a method for clearing it? A. Remove the obstruction manually. B. Apply suction. C. Place the patient in the recovery position to allow drainage. D. Use abdominal thrusts.
C. Place the patient in the recovery position to allow drainage.
You should never give up on resuscitating a cold-water drowning victim because: A. When the patient is submerged in water colder than body temperature, heat is maintained in the body. B. The resulting hypothermia can protect vital organs from the lack of oxygen. C. The resulting hypothermia raises the metabolic rate. D. All of the above.
B. The resulting hypothermia can protect vital organs from the lack of oxygen.
The three phases of a dive, in the order they occur, are: A. Ascent descent, and bottom. B. Descent, bottom, and ascent. C. Orientation, bottom and ascent. D. Descent, orientation, and ascent.
B. Descent, bottom, and ascent.
Areas usually affected by descent problems include: A. The lungs. B. The skin. C. The joints. D. Vision.
A. The lungs.
Potential problems associated with rupture of the lungs include all of the following except: A. Air Emboli. B. Pneumomediastinum. C. Pneumothorax. D. Hemopneumothorax.
D. Hemopneumothorax.
The organs most severely affected by air embolism are the: A. Brain and spinal cord. B. Brain and heart. C. Heart and lungs. D. Brain and lungs.
A. Brain and spinal cord.
Black widow spiders may be found in: A. New Hampshire. B. Woodpiles. C. Georgia. D. All of the above.
D. All of the above.
Coral snake venom is a powerful toxin that causes __________ of the nervous system. A. Paralysis. B. Hyperactivity. C. Hypoactivity. D. Hemiparesis.
A. Paralysis.
Rocky Mountain spotted fever and Lyme disease are both spread through the tick's: A. Saliva. B. Blood. C. Hormones. D. Excrement.
A. Saliva.
Signs of envenomation by a pit viper include all of the following except: A. Swelling. B. Chest pain. C. Ecchymosis. D. Severe burning pain at the site of the injury.
B. Chest pain.
Removal of a tick should be accomplished by: A. Suffocating it with gasoline. B. Burning it with a lighted match to cause it to release its grip. C. Using fine tweezers to pull it straight out of the skin. D. Suffocating it with Vasoline.
C. Using fine tweezers to pull it straight out of the skin.
Which of the following statements regarding the brown recluse spider is false? A. It is larger than the black widow spider. B. It lives mostly in the southern and central parts of the country. C. Venom is not neurotoxic. D. Bites rarely cause systemic signs and symptoms.
A. It is larger than the black widow spider.
Treatment of a snake bite from a pit viper includes: A. Calming the patient. B. Providing BLS as needed if the patient shows no sign of envenomation. C. Marking the skin with a pen over the swollen are to note whether swelling is spreading. D. All of the above.
D. All of the above.
At 1400 in July, the weather is 105 degrees F and very humid. You have been called for a "man down" at the park. As you arrive, you recognize him as an alcoholic who has been a "frequent flyer" with your service. It looks like he had been sitting under a tree when he fell over, unconscious. As you assess the patient, he has cold, clammy skin and a dry tongue. You suspect that: A. He is well-hydrated B. He has suffered heat exhaustion C. He is hypothermic D. He has heatstroke
B. He has suffered heat exhaustion.
At 1400 in July, the weather is 105 degrees F and very humid. You have been called for a "man down" at the park. As you arrive, you recognize him as an alcoholic who has been a "frequent flyer" with your service. It looks like he had been sitting under a tree when he fell over, unconscious. As you look closer, you note that he is shivering and his respirations are 20 breaths/min. You begin to have a stronger suspicion that he is now getting: A. Hyperthermic. B. Hypothermic. C. Drunk. D. Heatstroke.
B. Hypothermic.
At 1400 in July, the weather is 105 degrees F and very humid. You have been called for a "man down" at the park. As you arrive, you recognize him as an alcoholic who has been a "frequent flyer" with your service. It looks like he had been sitting under a tree when he fell over, unconscious. The direct transfer of heat from his body to the cold ground is called: A. Conduction. B. Convection. C. Radiation. D. Evaporation.
A. Conduction.
At 1400 in July, the weather is 105 degrees F and very humid. You have been called for a "man down" at the park. As you arrive, you recognize him as an alcoholic who has been a "frequent flyer" with your service. It looks like he had been sitting under a tree when he fell over, unconscious. You pull back on your glove and place the back of your hand on his skin at the abdomen. and skin feels cool. Again, you suspect A. Hyperthermia. B. Hypothermia. C. That he is drunk. D. Heatstroke.
B. Hypothermia.
At 1400 in July, the weather is 105 degrees F and very humid. You have been called for a "man down" at the park. As you arrive, you recognize him as an alcoholic who has been a "frequent flyer" with your service. It looks like he had been sitting under a tree when he fell over, unconscious. How will you treat this patient? A. Prevent conduction heat loss. B. Prevent convection heat loss. C. Remove the patient from the environment. D. All of the above.
D. All of the above
Small infant have a poor ability to thermoregulate and are unable to shiver to control heat loss until about the age of: A. 4-6 months. B. 6-12 months. C. 12-18 months. D. 18-24 months.
C. 12-18 months.
Most heat stroke cases occur when the temperature is around _________ and the humidity is 80%. A. 80 degrees F. B. 90 degrees F. C. 100 degrees F. D. 110 degrees F.
A. 80 degrees F.
Most frostbitten parts are: A. Soft and moist. B. Hard and waxy. C. Soft and waxy. D. Hard and moist.
B. Hard and waxy.
If a patient has a cold skin temperature, he or she likely is: A. Hypothermic. B. Hyperthermic. C. Hypovolemic. D. Hypoglycemic.
A. Hypothermic.
If a patient has a hot skin temperature, he or she likely is: A. Hypothermic. B. Hyperthermic. C. Hypoglycemic. D. Hypervolemic.
B. Hyperthermic.
When treating multiple victims of lightning strikes, who should you concentrate your efforts on first? A. Conscious patients. B. Unconscious patients in respiratory or cardiac arrest. C. All unconscious patients. D. None of the above.
B. Unconscious patients in respiratory or cardiac arrest.
What is the best method of inactivating a jellyfish sting? A. Urinating on it. B. Flushing the site with cold water. C. Applying vinegar. D. Applying an ice pack.
C. Applying vinegar.
Which of the following MOST accurately defines an allergic reaction? A. An exaggerated immune system response to any substance B. Destruction of the immune system by an external substance C. A release of erythrocytes in response to a foreign substance D. A direct negative effect on the body by an external substance
A. An exaggerated immune system response to any substance
Chemicals that are responsible for the signs and symptoms of an allergic reaction to a bee sting include: A. The bee venom itself. B. Adrenaline and histamines. C. Leukocytes and epinephrine. D. Histamines and leukotrienes.
D. Histamines and leukotrienes.
The foreign substance responsible for causing an allergic reaction is called a/an: A. Allergen. B. Antibody. C. Histamine. D. Leukotriene.
A. Allergen.
Anaphylaxis is MOST accurately defined as a/an: A. Moderate allergic reaction that primarily affects the vasculature. B. Extreme allergic reaction that may affect multiple body systems. C. Severe allergic reaction that typically resolves without treatment. D. Allergic reaction that causes bronchodilation and vasoconstriction.
B. Extreme allergic reaction that may affect multiple body systems.
Which of the following patients would MOST likely have a delayed onset of an allergic reaction? A. A 21-year-old female who inhaled pollen. B. A 30-year-old male who was stung by a bee. C. A 45-year-old male who ingested penicillin. D. A 50-year-old male who was exposed to latex.
C. A 45-year-old male who ingested penicillin.
The two MOST common signs of anaphylaxis are: A. Wheezing and diffuse urticaria. B. Watery eyes and localized itching. C. Expiratory stridor and tachycardia. D. Hypertension and swollen hands.
A. Wheezing and diffuse urticaria.
Urticaria is the medical term for: A. Hives. B. Burning. C. Swelling. D. A wheal.
A. Hives.
Because the stinger of a honeybee remains in the wound following a sting: A. The stinger can easily be removed with tweezers. B. The body's immune system deactivates the venom. C. Toxicity of the venom decreases within 10 minutes. D. It can continue to inject venom for up to 20 minutes.
D. It can continue to inject venom for up to 20 minutes.
Most patients who die of anaphylaxis do so within the first: A. 5 minutes following exposure. B. 30 minutes following exposure. C. 60 minutes following exposure. D. 90 minutes following exposure.
B. 30 minutes following exposure.
What part of the scene size-up would MOST likely provide clues regarding the source of a patient's allergic reaction? A. General activities occurring at the scene. B. The patient's general physical appearance. C. The environment in which the patient is found. D. The time of season in which the exposure occurred.
C. The environment in which the patient is found.
The stinger from a honeybee should be: A. Left in place and covered. B. Scraped away from the skin. C. Squeezed with tweezers and removed. D. Irrigated with copious amounts of water.
B. Scraped away from the skin.
Which of the following adventitious sounds indicates swelling of the upper airway? A. Rales. B. Stridor. C. Rhonchi. D.Wheezing.
B. Stridor.
A raised, swollen, well-defined area on the skin that is the result of an insect bite or sting is called: A. Hives. B. Purpura. C. Urticaria. D. A wheal.
D. A wheal.
Which of the following statements regarding fire ants is correct? A. Fire ants often bite a person repeatedly. B. Fire ant bites rarely cause anaphylaxis. C. Fire ant bites typically occur on the face. D. Most people are allergic to fire ant toxin.
A. Fire ants often bite a person repeatedly.
Which of the following negative effects of anaphylaxis will be the MOST rapidly fatal if not treated immediately? A. Diffuse urticaria. B. Severe hypotension. C. Upper airway swelling. D. Systemic vasodilation.
C. Upper airway swelling.
Which of the following medications blocks the release of histamines? A. Adrenaline. B. Epinephrine. C. Diphenhydramine. D. Acetaminophen.
C. Diphenhydramine.
Common signs and symptoms of an allergic reaction include all of the following, EXCEPT: A. Abdominal pain. B. Drying of the eyes. C. Flushing of the skin. D. Persistent dry cough.
B. Drying of the eyes.
Which of the following physiologic actions does epinephrine produce when given for an allergic reaction? A. Bronchodilation and vasodilation. B. Vasoconstriction and bronchodilation. C. Bronchoconstriction and vasoconstriction. D. Inhibition of further histamine release.
B. Vasoconstriction and bronchodilation.
Common side effects of epinephrine include all of the following, EXCEPT: A. Drowsiness. B. Tachycardia. C. Headache. D. Dizziness.
A. Drowsiness.
The adult epinephrine auto-injector contains ___ milligrams and the pediatric auto-injector contains ___ milligrams. A. 0.1, 0.01. B. 0.01, 0.1. C. 0.03, 0.3. D. 0.3, 0.15.
D. 0.3, 0.15.
Epinephrine is indicated for patients with an allergic reaction when: A. Wheezing and hypotension are present. B. The patient is anxious and tachycardic. C. A paramedic is present at the scene. D. The reaction produces urticaria only.
A. Wheezing and hypotension are present.
When using an auto-injector to give epinephrine, the primary injection site is the: A. Medial part of the buttocks. B. Lateral portion of the arm. C. Lateral portion of the thigh. D. Medial portion of the thigh.
C. Lateral portion of the thigh.
When administering epinephrine via auto-injector, you should hold the injector in place for: A. 5 seconds. B. 10 seconds. C. 15 seconds. D. 20 seconds.
B. 10 seconds.
Immediately after giving an epinephrine injection you should: A. Properly dispose of the syringe. B. Record the time and dose given. C. Reassess the patient's vital signs. D. Notify medical control of your action.
A. Properly dispose of the syringe.
The effects of epinephrine are typically observed within __________ following administration. A. 30 seconds. B. 1 minute. C. 30 minutes. D. 1 hour.
B. 1 minute.
A 38-year-old female was bitten by fire ants while at the park with her kids. Your initial assessment reveals that she is semiconscious, has profoundly labored breathing with reduced tidal volume, and has a rapid, thready pulse. She has a red rash on her entire body and her face appears swollen. You should: A. Administer 0.3 mg of epinephrine 1:1,000. B. Assist her ventilations with 100% oxygen. C. Apply oxygen and perform a rapid assessment. D. Place her supine with her legs elevated 6" to 12".
B. Assist her ventilations with 100% oxygen.
A 40-year-old male was stung by a bee and is having obvious signs of a severe allergic reaction. He tells you that he has an AnaKit in his car, but has never used it before. As your partner applies oxygen to the patient, you contact medical control. The physician will MOST likely advise you to: A. Administer two antihistamine tablets. B. Inject 1.0 mg of epinephrine 1:1,000 SC. C. Transport at once with close monitoring. D. Inject 0.3 mL of epinephrine 1:1,000 SC.
D. Inject 0.3 mL of epinephrine 1:1,000 SC.
You have administered one dose of epinephrine to a 40-year-old female to treat an allergic reaction that she developed after being stung by a scorpion. Your reassessment reveals that she is still having difficulty breathing, has a decreasing mental status, and has a blood pressure of 80/50 mm Hg. You should: A. Monitor her en route to the hospital and call medical control if she worsens. B. Crush up an antihistamine tablet and place it in between her cheek and gum. C. Request permission from medical control to give another dose of epinephrine. D. Administer a nebulizer bronchodilator to improve the status of her breathing.
C. Request permission from medical control to give another dose of epinephrine.
While auscultating breath sounds of a patient who was stung multiple times by a yellow jacket, you hear bilateral wheezing over all lung fields. This indicates: A. Rapid swelling of the upper airway tissues. B. A significant amount of fluid in the alveoli. C. Constriction of the bronchioles in the lungs. D. Enlargement of the bronchioles in the lungs.
C. Constriction of the bronchioles in the lungs.
You have just administered subcutaneous epinephrine to a 22-year-old male from his AnaKit. After properly disposing of the syringe, you should: A. Have him chew and swallow the antihistamine tablets. B. Transport at once and carefully monitor his condition. C. Prepare another dose of epinephrine in case it is needed. D. Apply a chemical cold pack to the site of the sting or bite.
A. Have him chew and swallow the antihistamine tablets.
A 19-year-old female was stung multiple times on the legs by fire ants. She states that she is allergic to fire ants, but does not have epinephrine or an AnaKit. The patient is conscious and alert and complains of pain to the area of the bites. Her blood pressure is 122/70 mm Hg, pulse is 100 beats/min and strong, and respirations are 18 breaths/min and unlabored. You should: A. Position her legs well above the level of her heart. B. Administer oxygen and transport her to the hospital. C. Request a paramedic unit to administer epinephrine. D. Not transport her and tell her to see her physician.
B. Administer oxygen and transport her to the hospital.
A 50-year-old male was stung by a honeybee approximately 15 minutes ago. He presents with respiratory distress, facial swelling, and hypotension. After placing him on oxygen and administering his epinephrine via auto-injector, you note that his breathing has improved. Additionally, his facial swelling is resolving and his blood pressure is stable. Your next action should be to: A. Load him into the ambulance and transport to the hospital. B. Visualize his airway to assess for oropharyngeal swelling. C. Notify medical control of the situation and your treatment. D. Record the time and dose of the injection on the run form.
D. Record the time and dose of the injection on the run form.
A 37-year-old male is having a severe allergic reaction to penicillin. He does not have an epinephrine auto-injector or AnaKit and your protocols do not allow you to carry such drugs on the ambulance. How should you proceed with the treatment of this patient? A. Ask the patient if he has any Benadryl tablets that you can administer. B. Administer oxygen, transport at once, and request a paramedic rendezvous. C. Remain at the scene with the patient and request a paramedic ambulance. D. Maintain the patient's airway and immediately transport him to the hospital.
B. Administer oxygen, transport at once, and request a paramedic rendezvous.
After administering 0.3 mL of epinephrine 1:1,000 to a 22-year-old female with an allergic reaction, you note improvement in her breathing and dissipation of her hives. However, she is still anxious and tachycardic. You should: A. Contact medical control and obtain authorization to administer another 0.3 mL of epinephrine. B. Transport her rapidly as it is obvious that she is having a severe reaction to the epinephrine. C. Monitor her closely but recall that anxiety and tachycardia are side effects of epinephrine. D. Consider administering 0.15 mL of epinephrine to completely resolve her allergic reaction.
C. Monitor her closely but recall that anxiety and tachycardia are side effects of epinephrine.
You respond to the residence of a 55-year-old female with a possible allergic reaction to peanuts that she ate approximately 30 minutes ago. The patient is conscious and alert, but has diffuse urticaria and the feeling that she has a lump in her throat. As your partner applies oxygen to the patient, you should: A. Ask her if she has prescribed epinephrine. B. Obtain a complete set of baseline vital signs. C. Ascertain if she has a family history of allergies. D. Ask her when her last allergic reaction occurred.
A. Ask her if she has prescribed epinephrine.
A 48-year-old male is found unconscious in the garden by his wife. When you arrive at the scene and assess the man, you find that he is unresponsive, has labored and shallow breathing, and has hives over his entire trunk. You should: A. Perform a rapid head-to-toe assessment. B. Maintain his airway and assist his ventilations. C. Ask the wife if her husband has any known allergies. D. Apply the AED in the event that cardiac arrest occurs.
B. Maintain his airway and assist his ventilations.
Chemicals that are responsible for the signs and symptoms of an allergic reaction to a bee sting include: A. The bee venom itself. B. Histamines and leukotrienes. C. Leukocytes and epinephrine. D. Adrenaline and histamines.
B. Histamines and leukotrienes.
Urticaria is the medical term for: A. Burning. B. Swelling. C. Hives. D. A wheal
C. Hives.
Because the stinger of a honeybee remains in the wound following a sting: A. The body's immune system deactivates the venom. B. The toxicity of the venom decreases within 10 minutes. C. It can continue to inject venom for up to 20 minutes. D. The stinger can easily be removed with tweezers.
C. It can continue to inject venom for up to 20 minutes.
You are assessing a young male who was stung on the leg by a scorpion. He is conscious and alert, his breathing is regular and unlabored, and his blood pressure is 122/64 mm Hg. Assessment of his leg reveals a wheal surrounded by an area of redness. He states that he had a "bad reaction" the last time he was stung by a scorpion, and carries his own epinephrine auto-injector. You should: A. Apply high-flow oxygen, apply a chemical cold pack directly to the injection site, and transport at once. B. Apply high-flow oxygen, obtain approval from medical control to assist him with his epinephrine, and transport. C. Apply oxygen as needed, clean the area with soap and water or a mild antiseptic, and transport him to the hospital. D. Assess his ABCs and vital signs in 15 minutes and allow him to drive himself to the hospital if he remains stable.
C. Apply oxygen as needed, clean the area with soap and water or a mild antiseptic, and transport him to the hospital.
A 38-year-old female was bitten by fire ants while at the park with her kids. Your primary assessment reveals that she is semiconscious, has profoundly labored breathing, and has a rapid, thready pulse. She has a red rash on her entire body and her face is very swollen. You should: A. Assist her ventilations with 100% oxygen. B. Place her supine with her legs elevated 6² to 12². C. Administer 0.3 mg of epinephrine. D. Perform a rapid secondary assessment.
A. Assist her ventilations with 100% oxygen.
A 48-year-old male is found unconscious in the garden by his wife. When you arrive at the scene and assess the man, you find that he is unresponsive, has severely labored breathing, and has hives over his entire trunk. You should: A. Ask his wife if he has any known allergies. B. Maintain his airway and assist his ventilations. C. Perform a detailed secondary assessment. D. Apply the automated external defibrillator (AED) in the event that cardiac arrest occurs.
B. Maintain his airway and assist his ventilations.
Common signs and symptoms of an allergic reaction include all of the following, EXCEPT: A. Abdominal cramps. B. Drying of the eyes. C. Persistent dry cough. D. Flushing of the skin.
B. Drying of the eyes.
Which of the following physiologic actions does epinephrine produce when given for an allergic reaction? A. Vasoconstriction and bronchodilation. B. Blocking of further histamine release. C. Bronchoconstriction and vasoconstriction. D. Bronchodilation and vasodilation.
A. Vasoconstriction and bronchodilation.
You have administered one dose of epinephrine to a 40-year-old female to treat an allergic reaction that she developed after being stung by a scorpion. Your reassessment reveals that she is still having difficulty breathing, has a decreasing mental status, and has a blood pressure of 80/50 mm Hg. You should: A. Request permission from medical control to give another dose of epinephrine. B. Administer a nebulizer bronchodilator to improve the status of her breathing. C. Crush up an antihistamine tablet and place it in between her cheek and gum. D. Monitor her en route to the hospital and call medical control if she worsens.
A. Request permission from medical control to give another dose of epinephrine.
A 19-year-old female was stung multiple times on the legs by fire ants. She states that she is allergic to fire ants, but does not carry her own epinephrine. The patient is conscious and alert and complains of pain to the area of the bites. Her blood pressure is 122/70 mm Hg, her pulse is 100 beats/min and strong, and her respirations are 18 breaths/min and unlabored. You should: A. Advise her to see her physician as soon as possible. B. Request a paramedic unit to administer epinephrine. C. Position her legs well above the level of her heart. D. Administer oxygen and transport her to the hospital.
D. Administer oxygen and transport her to the hospital.
Another common name for envenomation is: A. Hives. B. Sting. C. Snake Bite. D. Wheezing.
B. Sting.
Injection of medications such as penicillin may cause an immediate and severe reaction. "Immediate" is described as: A. Within 1 minute. B. Within 5 minutes. C. Within 15 minutes. D. Within 30 minutes.
D. Within 30 minutes.
Eating certain foods, such as shell fish or nuts, may result in a relatively slow reaction that still can be quite severe. That relatively slow reaction time is: A. More than 15 minutes. B. More than 30 minutes. C. More than 45 minutes. D. More than 60 minutes.
B. More than 30 minutes.
Which of the following is not a safe way to remove the stinger and attached muscle injected by a honeybee into the patient? A. Use clean tweezers. B. Use a clean credit card to scrape it. C. Use the edge of a sharp, stiff object to scrape it. D. Use the side of a tongue depressor to scrape it.
A. Use clean tweezers.
If untreated, a severe anaphylactic reaction can proceed rapidly to death. More than two-thirds of patients who die, do so within: A. The first half hour. B. The first hour. C. Three hours. D. Twelve hours.
A. The first half hour.
Remember, the presence of hypoperfusion or respiratory distress indicates that the patient is having a severe enough allergic reaction that can lead to death. Hypoperfusion is: A. Stridor. B. Shock. C. Pallor. D. Nostril Flaring.
B. Shock.
Which of the following is not a common sign and symptom of allergic reactions in the respiratory system? A. An increase in pulse. B. Hoarseness. C. Tightness in the chest or throat. D. Wheezing and/or stridor.
A. An increase in pulse.
All of the following are common signs and symptoms of allergic reactions related to the circulatory system, except: A. Decrease in blood pressure. B. Increase in pulse rate. C. Sneezing, itchy runny nose. D. Loss of consciousness and coma.
C. Sneezing, itchy runny nose.
Any substance that can trigger the body's immune system and cause an allergic reaction is known as a(n): A. Allergen. B. Histamine. C. Toxin. D. Leukotriene.
A. Allergen.
Before administering epinephrine for injection from an allergic emergency kit, what is the first step? A. Get an order. B. Clean and wipe the thigh. C. Place the needle in proper position. D. Check the expiration date of the medication.
A. Get an order.
________ is a harsh, high-pitched inspiratory sound. A. Wheezing. B. Stridor. C. Grunting. D. Croup.
B. Stridor.
When administering an EpiPen auto injector, it is important to hold the injector in place for ________ seconds to ensure that all of the medication is injected. A. 5. B. 10. C. 15. D. 30.
B. 10.
Epinephrine can have an effect within: A. 30 seconds. B. 1 minute. C. 2 minutes. D. 3 minutes.
B. 1 minute.
Which of the following MOST accurately defines an allergic reaction? A. An exaggerated immune system response to any substance. B. Destruction of the immune system by an external substance. C. A release of erythrocytes in response to a foreign substance. D. A direct negative effect on the body by an external substance.
A. An exaggerated immune system response to any substance.
Chemicals that are responsible for the signs and symptoms of an allergic reaction to a bee sting include: A. The bee venom itself. B. Adrenaline and histamines. C. Leukocytes and epinephrine. D. Histamines and leukotrienes.
D. Histamines and leukotrienes.
Anaphylaxis is MOST accurately defined as a(n): A. Moderate allergic reaction that primarily affects the vasculature. B. Extreme allergic reaction that may affect multiple body systems. C. Severe allergic reaction that typically resolves without treatment. D. Allergic reaction that causes bronchodilation and vasoconstriction.
B. Extreme allergic reaction that may affect multiple body systems.
The two MOST common signs of anaphylaxis are: A. Wheezing and widespread urticaria. B. Watery eyes and localized itching. C. Expiratory stridor and tachycardia. D. Hypertension and swollen hands.
A. Wheezing and widespread urticaria.
Urticaria is the medical term for: A. Hives. B. Burning. C. Swelling. D. A wheal.
A. Hives.
The foreign substance responsible for causing an allergic reaction is called a(n): A. Allergen. B. Antibody. C. Histamine. D. Leukotriene.
A. Allergen.
Which of the following patients would MOST likely have a delayed onset of an allergic reaction? A. A 21-year-old female who inhaled pollen. B. A 30-year-old male who was stung by a bee. C. A 45-year-old male who ingested penicillin. D. A 50-year-old male who was exposed to latex.
C. A 45-year-old male who ingested penicillin.
Which of the following statements regarding fire ants is correct? A. Fire ants often bite a person repeatedly. B. Fire ant bites rarely cause anaphylaxis. C. Fire ant bites typically occur on the face. D. Most people are allergic to fire ant toxin.
A. Fire ants often bite a person repeatedly.
A raised, swollen, well-defined area on the skin that is the result of an insect bite or sting is called: A. A pustule. B. Purpura. C. Urticaria. D. A wheal.
D. A wheal.
Because the stinger of a honeybee remains in the wound following a sting: A. The stinger can easily be removed with tweezers. B.The body's immune system deactivates the venom. C. The toxicity of the venom decreases within 10 minutes. D. It can continue to inject venom for up to 20 minutes.
D. It can continue to inject venom for up to 20 minutes.
The stinger from a honeybee should be: A. Left in place and covered. B. Scraped away from the skin. C. Squeezed with tweezers and removed. D. Irrigated with copious amounts of water.
B. Scraped away from the skin.
Most patients who die of anaphylaxis do so within the first __________ following exposure. A. 5 minutes. B. 30 minutes. C. 60 minutes. D. 90 minutes.
B. 30 minutes.
You are assessing a young male who was stung on the leg by a scorpion. He is conscious and alert, his breathing is regular and unlabored, and his blood pressure is 122/64 mm Hg. Assessment of his leg reveals a wheal surrounded by an area of redness. He states that he had a "bad reaction" the last time he was stung by a scorpion, and carries his own epinephrine auto-injector. You should: A. Apply high-flow oxygen, apply a chemical cold pack directly to the injection site, and transport at once. B. Apply high-flow oxygen, obtain approval from medical control to assist him with his epinephrine, and transport. C. Assess his ABCs and vital signs in 15 minutes and allow him to drive himself to the hospital if he remains stable. D. Apply oxygen as needed, clean the area with soap and water or a mild antiseptic, and transport him to the hospital.
D. Apply oxygen as needed, clean the area with soap and water or a mild antiseptic, and transport him to the hospital.
Which of the following would MOST likely provide clues regarding the source of a patient's allergic reaction? A. The patient's family history. B. The patient's general physical appearance. C. The environment in which the patient is found. D. The time of season in which the exposure occurred.
C. The environment in which the patient is found.
A 73-year-old man presents with a generalized rash, which he thinks may have been caused by an antibiotic that he recently began taking. He has a history of coronary artery disease, hypertension, and emphysema. He is conscious and alert, his blood pressure is 144/94 mm Hg, and his pulse is 64 beats/min and regular. You auscultate his breath sounds and hear scattered wheezing, although he is not experiencing respiratory distress. In addition to administering oxygen, you should: A. Contact medical control if needed, transport the patient, and monitor him for signs of deterioration. B. Ask him if he has epinephrine and request approval from medical control to administer it to the patient. C. Avoid the use of epinephrine because of his cardiac history, even if his symptoms become severe. D. Begin transport and request to administer epinephrine if his systolic blood pressure falls below 110 mm Hg.
A. Contact medical control if needed, transport the patient, and monitor him for signs of deterioration.
A 37-year-old male is having a severe allergic reaction to penicillin. He does not have an epinephrine auto-injector and your protocols do not allow you to carry epinephrine on the ambulance. How should you proceed with the treatment of this patient? A. Ask the patient if he has any diphenhydramine (Benadryl) tablets that you can administer. B. Administer oxygen, transport at once, and request a paramedic intercept. C. Remain at the scene with the patient and request a paramedic ambulance. D. Quickly determine if there are any bystanders who may carry epinephrine.
B. Administer oxygen, transport at once, and request a paramedic intercept.
Which of the following negative effects of anaphylaxis will be the MOST rapidly fatal if not treated immediately? A. Diffuse urticaria. B. Severe hypotension. C. Upper airway swelling. D. Systemic vasodilation.
C. Upper airway swelling.
A 38-year-old female was bitten by fire ants while at the park with her kids. Your primary assessment reveals that she is semiconscious, has profoundly labored breathing, and has a rapid, thready pulse. She has a red rash on her entire body and her face is very swollen. You should: A. Perform a rapid secondary assessment. B. Assist her ventilations with 100% oxygen. C. Administer 0.3 mg of epinephrine. D. Place her supine with her legs elevated 6″ to 12″.
B. Assist her ventilations with 100% oxygen.
A 48-year-old male is found unconscious in the garden by his wife. When you arrive at the scene and assess the man, you find that he is unresponsive, has severely labored breathing, and has hives over his entire trunk. You should: A. Perform a detailed secondary assessment. B. Maintain his airway and assist his ventilations. C. Ask his wife if he has any known allergies. D. Apply the automated external defibrillator (AED) in the event that cardiac arrest occurs.
B. Maintain his airway and assist his ventilations.
You respond to the residence of a 55-year-old female with a possible allergic reaction to peanuts that she ate approximately 30 minutes ago. The patient is conscious and alert, but has diffuse urticaria and the feeling that she has a lump in her throat. As your partner applies oxygen to the patient, you should: A. Ask her if she has prescribed epinephrine. B. Obtain a complete set of baseline vital signs. C. Ascertain if she has a family history of allergies. D. Ask her when her last allergic reaction occurred.
A. Ask her if she has prescribed epinephrine.
Which of the following medications blocks the release of histamines? A. Albuterol (Ventolin). B. Epinephrine. C. Acetaminophen (Tylenol). D. Diphenhydramine (Benadryl).
D. Diphenhydramine (Benadryl).
Common signs and symptoms of an allergic reaction include all of the following, EXCEPT: A. Abdominal cramps. B. Drying of the eyes. C. Flushing of the skin. D. Persistent dry cough.
B. Drying of the eyes.
Which of the following sounds indicates swelling of the upper airway? A. Rales. B. Stridor. C. Rhonchi. D. Wheezing.
B. Stridor.
While auscultating breath sounds of a patient who was stung multiple times by a yellow jacket, you hear bilateral wheezing over all lung fields. This indicates: A. Rapid swelling of the upper airway tissues. B. A significant amount of fluid in the alveoli. C. Narrowing of the bronchioles in the lungs. D. Enlargement of the bronchioles in the lungs.
C. Narrowing of the bronchioles in the lungs.
Which of the following physiologic actions does epinephrine produce when given for an allergic reaction? A. Bronchodilation and Vasodilation. B. Vasoconstriction and Bronchodilation. C. Bronchoconstriction and Vasoconstriction. D. Blocking of further histamine release.
B. Vasoconstriction and Bronchodilation.
Epinephrine is indicated for patients with an allergic reaction when: A. Wheezing and hypotension are present. B. The patient is anxious and tachycardic. C. A paramedic is present at the scene. D. The reaction produces severe urticaria.
A. Wheezing and hypotension are present.
The adult epinephrine auto-injector delivers ______ mg of epinephrine, and the pediatric auto-injector delivers ______ mg. A. 0.1, 0.01. B. 0.01, 0.1. C. 0.03, 0.3. D. 0.3, 0.15.
D. 0.3, 0.15.
When using an auto-injector to give epinephrine, the primary injection site is the: A. Medial part of the buttocks. B. Lateral portion of the arm. C. Lateral portion of the thigh. D. Medial portion of the thigh.
C. Lateral portion of the thigh.
When administering epinephrine via auto-injector, you should hold the injector in place for: A. 5 seconds. B. 10 seconds. C. 15 seconds. D. 20 seconds.
B. 10 seconds.
Immediately after giving an epinephrine injection, you should: A. Properly dispose of the syringe. B. Record the time and dose given. C. Reassess the patient's vital signs. D. Notify medical control of your action.
A. Properly dispose of the syringe.
A 50-year-old male was stung by a honeybee approximately 15 minutes ago. He presents with respiratory distress, facial swelling, and hypotension. After placing him on oxygen and administering his epinephrine via auto-injector, you note that his breathing has improved. Additionally, his facial swelling is resolving and his blood pressure is stable. Your next action should be to: A. Reassess his breathing and blood pressure in 15 minutes. B. Visualize his airway to assess for oropharyngeal swelling. C. Notify medical control of the patient's response to your treatment. D. Record the time and dose of the injection, and transport promptly.
D. Record the time and dose of the injection, and transport promptly.
You have administered one dose of epinephrine to a 40-year-old female to treat an allergic reaction that she developed after being stung by a scorpion. Your reassessment reveals that she is still having difficulty breathing, has a decreasing mental status, and has a blood pressure of 80/50 mm Hg. You should: A. Monitor her en route to the hospital and call medical control if she worsens. B. Crush up an antihistamine tablet and place it in between her cheek and gum. C. Request permission from medical control to give another dose of epinephrine. D. Administer a nebulized bronchodilator to improve the status of her breathing.
C. Request permission from medical control to give another dose of epinephrine.
The effects of epinephrine are typically observed within _________ following administration. A. 30 seconds. B. 1 minute. C. 30 minutes. D. 1 hour.
B. 1 minute.
Common side effects of epinephrine include all of the following, EXCEPT: A. Drowsiness. B. Tachycardia. C. Headache. D. Dizziness.
A. Drowsiness.
A 19-year-old female was stung multiple times on the legs by fire ants. She states that she is allergic to fire ants, but does not carry her own epinephrine. The patient is conscious and alert and complains of pain to the area of the bites. Her blood pressure is 122/70 mm Hg, her pulse is 100 beats/min and strong, and her respirations are 18 breaths/min and unlabored. You should: A. Position her legs well above the level of her heart. B. Administer oxygen and transport her to the hospital. C. Request a paramedic unit to administer epinephrine. D. Advise her to see her physician as soon as possible.
B. Administer oxygen and transport her to the hospital.
After administering 0.3 mg of epinephrine via auto-injector to a 22-year-old female with an allergic reaction, you note improvement in her breathing and dissipation of her hives. However, she is still anxious and tachycardic. You should: A. Contact medical control and obtain authorization to administer another 0.3 mg of epinephrine. B. Transport her rapidly, as it is obvious that she is having a severe reaction to the epinephrine. C. Monitor her closely but recall that anxiety and tachycardia are side effects of epinephrine. D. Consider administering 0.15 mg of epinephrine to completely resolve her allergic reaction.
C. Monitor her closely but recall that anxiety and tachycardia are side effects of epinephrine.
Type I diabetes: A. Is typically treated with medications such as Diabenase. B. Is a condition in which no insulin is produced by the body. C. Typically occurs in patients between 50 and 70 years of age. D. Is defined as a blood sugar level that is less than 120 mg/dL.
B. Is a condition in which no insulin is produced by the body.
Insulin functions in the body by: A. Producing new glucose as needed. B. Enabling glucose to enter the cells. C. Increasing circulating blood glucose. D. Metabolizing glucose to make energy.
B. Enabling glucose to enter the cells.
Diabetes is MOST accurately defined as a/an: A. Disorder of carbohydrate metabolism. B. Abnormally high blood glucose level. C. Mass excretion of glucose by the kidneys. D. Lack of insulin production in the pancreas.
A. Disorder of carbohydrate metabolism.
Patients with type II diabetes usually control their disease with all of the following, EXCEPT: A. Diet and exercise. B. Tolbutamide (Orinase). C. Glyburide (Micronase). D. Supplemental insulin.
D. Supplemental insulin.
Which of the following statements regarding glucose is correct? A. Most cells will function normally without glucose. B. Blood glucose levels decrease in the absence of insulin. C. The brain requires glucose as much as it requires oxygen. D. The brain requires insulin to allow glucose to enter the cells.
C. The brain requires glucose as much as it requires oxygen.
Normal blood glucose levels, as measured by a glucometer, are: A. 60-80 mg/dL. B. 80-120 mg/dL. C. 130-150 mg/dL. D. 160-200 mg/dL.
B. 80-120 mg/dL.
Patients with uncontrolled diabetes experience polyuria because: A. They drink excess amounts of water due to dehydration. B. Excess glucose in the blood is excreted by the kidneys. C. Low blood glucose levels result in cellular dehydration. D. High blood sugar levels cause permanent kidney damage.
B. Excess glucose in the blood is excreted by the kidneys.
Excessive eating caused by cellular "hunger" is called: A. Polyphagia. B. Polydipsia. C. Dysphasia. D. Dyspepsia.
A. Polyphagia.
Metformin, a non-insulin medication, is another name for: A. Glucophage. B. Glucotrol. C. Micronase. D. Diabenase.
A. Glucophage.
Ketone production is the result of: A. Acidosis when blood glucose levels are low. B. Blood glucose levels higher than 120 mg/dL. C. Fat metabolization when glucose is unavailable. D. Rapid entry of glucose across the cell membrane.
C. Fat metabolization when glucose is unavailable.
Diabetic ketoacidosis occurs when: A. Blood glucose levels rapidly fall. B. The cells rapidly metabolize glucose. C. The pancreas produces excess insulin. D. Insulin is not available in the body.
D. Insulin is not available in the body.
Diabetic coma is a life-threatening condition that results in: A. Hypoglycemia, excess insulin, and dehydration. B. Hyperglycemia, ketoacidosis, and dehydration. C. Hypoglycemia, dehydration, and ketoacidosis. D. Hyperglycemia, excess insulin, and ketoacidosis.
B. Hyperglycemia, ketoacidosis, and dehydration.
Which of the following statements regarding diabetic coma is correct? A. Diabetic coma can be prevented by taking smaller insulin doses. B. Diabetic coma typically develops over a period of hours or days. C. Patients with low blood glucose levels are prone to diabetic coma. D. Diabetic coma rapidly progresses once hyperglycemia develops.
B. Diabetic coma typically develops over a period of hours or days.
Common signs and symptoms of diabetic coma include all of the following, EXCEPT: A. Warm, dry skin. B. Cool, clammy skin. C. Rapid, thready pulse. D. Acetone breath odor.
B. Cool, clammy skin.
Insulin shock will MOST likely develop if a patient: A. Takes too much of his or her prescribed insulin. B. Markedly overeats and misses an insulin dose. C. Eats a regular meal followed by mild exertion. D. Misses one or two prescribed insulin injections.
A. Takes too much of his or her prescribed insulin.
The signs and symptoms of insulin shock are the result of: A. Increased blood glucose levels. B. Fat metabolism within the cells. C. Decreased blood glucose levels. D. Prolonged and severe dehydration.
C. Decreased blood glucose levels.
Classic signs and symptoms of hypoglycemia include: A. Warm, dry skin; irritability; bradycardia; rapid respirations. B. Cool, clammy skin; weakness; tachycardia; rapid respirations. C. Warm, dry skin; hunger; abdominal pain; deep, slow respirations. D. Cold, clammy skin; bradycardia; hunger; deep, rapid respirations.
B. Cool, clammy skin; weakness; tachycardia; rapid respirations.
Assessment of a patient with hypoglycemia will MOST likely reveal: A. Sunken Eyes. B. Hyperactivity. C. Warm, dry skin. D. Bizarre Behavior.
D. Bizarre Behavior.
Insulin shock tends to develop more often and more severely in children because: A. They have larger glucose stores than adults do. B. They do not always eat correctly and on schedule. C. Their cells do not uptake glucose as fast as adults' do. D. Their low activity levels cause rapid glucose depletion.
B. They do not always eat correctly and on schedule.
Kussmaul respirations are an indication that the body is: A. Attempting to eliminate acids from the blood. B. Trying to generate energy by breathing deeply. C. Severely hypoxic and is eliminating excess CO2. D. Compensating for decreased blood glucose levels.
A. Attempting to eliminate acids from the blood.
In contrast to insulin shock, diabetic coma: A. Is rapidly reversible if oral glucose is given. B. Commonly results in excess water retention. C. Can only be corrected in the hospital setting. D. Is a rapidly developing metabolic disturbance.
C. Can only be corrected in the hospital setting.
When obtaining a SAMPLE history from a diabetic patient, it would be MOST important to determine: A. If he or she has had any recent illnesses or excessive stress. B. Approximately how much water the patient has drank today. C. If there is a family history of diabetes or related conditions. D. The name of the physician who prescribed his or her insulin.
A. If he or she has had any recent illnesses or excessive stress.
When assessing an unconscious diabetic patient, the primary visible difference between diabetic coma and insulin shock is the: A. Patient's mental status. B. Rate of the patient's pulse. C. Presence of a medic-alert tag. D. Rate and depth of breathing.
D. Rate and depth of breathing.
Glutose is a trade name for: A. Glucotrol. B. Oral glucose. C. Micronase. D. Glucophage.
B. Oral glucose.
To which of the following diabetic patients should you administer oral glucose? A. An unconscious 33-year-old male with cool, clammy skin. B. A conscious 37-year-old female with nausea and vomiting. C. A semiconscious 40-year-old female without a gag reflex. D. A confused 55-year-old male with tachycardia and pallor.
D. A confused 55-year-old male with tachycardia and pallor.
Proper procedure for administering oral glucose to a patient includes all of the following, EXCEPT: A. Assessing the patient's mental status. B. Ensuring the absence of a gag reflex. C. Checking the medication's expiration date. D. Requesting permission from medical control.
B. Ensuring the absence of a gag reflex.
Which of the following conditions is the diabetic patient at an increased risk of developing? A. Blindness. B. Depression. C. Alcoholism. D. Hepatitis B.
A. Blindness.
A 19-year-old male complains of "not feeling right." His insulin and a syringe are on a nearby table. The patient says he thinks he took his insulin and can't remember whether or not he ate. He is also unable to tell you the time or what day it is. In addition to administering oxygen, you should: A. Contact medical control and administer oral glucose. B. Assist him with his insulin injection and reassess him. C. Request a paramedic ambulance to administer IV glucose. D. Transport only with close, continuous monitoring en route.
A. Contact medical control and administer oral glucose.
A man finds his 59-year-old wife unconscious on the couch. He states that she takes medications for type II diabetes. He further tells you that his wife has been ill recently and has not eaten for the past 24 hours. Your assessment reveals that the patient is responsive to painful stimuli only. You should: A. Quickly assess for the presence of a medic-alert tag. B. Administer 100% oxygen via a nonrebreathing mask. C. Open and maintain her airway and assess breathing. D. Administer oral glucose between her cheek and gum.
C. Open and maintain her airway and assess breathing.
You respond to a movie theater for a 70-year-old male who is confused. His wife tells you he has type II diabetes but refuses to take his pills. Your assessment reveals that the patient is diaphoretic, tachycardic, and tachypneic. Initial management for this patient should include: A. Administering 1 to 2 tubes of oral glucose. B. Applying a nonrebreathing mask at 15 L/min. C. Assisting the patient with his diabetic medication. D. Performing a rapid exam and obtaining vital signs.
B. Applying a nonrebreathing mask at 15 L/min.
You respond to a residence for a patient who is "not acting right." As you approach the door, the patient, a 35-year-old male, begins shouting profanities at you and your partner while holding a baseball bat. The man is confused and diaphoretic, and is wearing a medic-alert bracelet. You should: A. Calm him down so you can assess him. B. Be assertive and talk the patient down. C. Contact medical control for instructions. D. Retreat at once and call law enforcement.
D. Retreat at once and call law enforcement.
You are treating a 20-year-old male with a history of diabetes. The patient states that he is not feeling well. His vital signs are stable; however, he is confused and his skin is cool and clammy. You attempt to obtain a blood glucose reading with your glucometer; however, it reads "error" after three attempts. After administering 100% oxygen, you should: A. Perform a detailed physical exam at the scene. B. Administer oral glucose as needed and transport. C. Attempt to obtain another blood glucose reading. D. Transport only with close, continuous monitoring.
B. Administer oral glucose as needed and transport.
A 28-year-old female patient is found to be responsive to verbal stimuli only. Her roommate states that she was recently diagnosed with type I diabetes and has had difficulty controlling her blood sugar level. She further tells you that the patient has been urinating excessively and has progressively worsened over the last 24 to 36 hours. On the basis of this patient's clinical presentation, you should suspect that she: A. Is significantly hyperglycemic. B. Is experiencing insulin shock. C. Has a urinary tract infection. D. Has overdosed on her insulin.
A. Is significantly hyperglycemic.
You are treating a 40-year-old male with a documented blood sugar reading of 300 mg/dL. The patient is semiconscious and breathing shallowly, and is receiving assisted ventilation from your partner. You should recognize that definitive treatment for this patient includes: A. Oxygen. B. Glucagon. C. Insulin. D. Dextrose.
C. Insulin.
A 29-year-old female presents with confusion and disorientation. Her respirations are rapid and shallow and her pulse is 120 beats/min and thready. She is markedly diaphoretic and has an oxygen saturation of 89%. You should: A. Transport immediately. B. Administer oral glucose. C. Provide ventilatory support. D. Treat her for hyperglycemia.
C. Provide ventilatory support.
A 37-year-old female with a history of diabetes presents with excessive urination and weakness of 2 days' duration. You apply 100% oxygen and assess her blood glucose level, which reads 320 mg/dL. If this patient's condition is not promptly treated, she will MOST likely develop: A. Severe insulin shock. B. Acidosis and dehydration. C. Complete renal failure. D. Hypoxia and overhydration.
B. Acidosis and dehydration.
A 75-year-old male with a history of insulin-dependent diabetes presents with chest pain and a general feeling of weakness. He tells you that he took his insulin today and ate a regular meal approximately 2 hours ago. You should treat this patient as though he is experiencing: A. Hypoglycemia. B. An acute stroke. C. Hyperglycemia. D. A heart attack.
D. A heart attack.
Diabetes is MOST accurately defined as a(n): A. Disorder of carbohydrate metabolism. B. Abnormally high blood glucose level. C. Mass excretion of glucose by the kidneys. D. Lack of insulin production in the pancreas.
A. Disorder of carbohydrate metabolism.
Patients with type 2 diabetes usually control their disease with all of the following, EXCEPT: A. Diet and exercise. B. Tolbutamide (Orinase). C. Glyburide (Micronase). D. Supplemental insulin.
D. Supplemental insulin.
Type 1 diabetes: A. Is typically treated with medications such as metformin (Glucophage). B. Is a condition in which no insulin is produced by the body. C. Typically occurs in patients between 50 and 70 years of age. D. Is defined as a blood sugar level that is less than 120 mg/dL.
B. Is a condition in which no insulin is produced by the body.
Glipizide, a non-insulin-type medication, is another name for: A. Glucophage. B. Glucotrol. C. Micronase. D. Diabinese.
B. Glucotrol.
Insulin functions in the body by: A. Producing new glucose as needed. B. Enabling glucose to enter the cells. C. Increasing circulating blood glucose. D. Metabolizing glucose to make energy.
B. Enabling glucose to enter the cells.
Which of the following statements regarding glucose is correct? A. Most cells will function normally without glucose. B. Blood glucose levels decrease in the absence of insulin. C. The brain requires glucose as much as it requires oxygen. D. The brain requires insulin to allow glucose to enter the cells.
C. The brain requires glucose as much as it requires oxygen.
Excessive eating caused by cellular "hunger" is called: A. Polyphagia. B. Polydipsia. C. Dysphasia. D. Dyspepsia.
A. Polyphagia.
Patients with uncontrolled diabetes experience polyuria because: A. They drink excess amounts of water due to dehydration. B. Excess glucose in the blood is excreted by the kidneys. C. Low blood glucose levels result in cellular dehydration. D. High blood sugar levels cause permanent kidney damage.
B. Excess glucose in the blood is excreted by the kidneys.
Ketone production is the result of: A. Acidosis when blood glucose levels are low. B. Blood glucose levels higher than 120 mg/dL. C. Fat metabolization when glucose is unavailable. D. Rapid entry of glucose across the cell membrane.
C. Fat metabolization when glucose is unavailable.
A 37-year-old female with a history of diabetes presents with excessive urination and weakness of 2 days' duration. You apply 100% oxygen and assess her blood glucose level, which reads 320 mg/dL. If this patient's condition is not promptly treated, she will MOST likely develop: A. Severe insulin shock. B. Acidosis and dehydration. C. Complete renal failure. D. Hypoxia and overhydration.
B. Acidosis and dehydration.
The normal blood glucose level, as measured by a glucometer, is between: A. 60 and 80 mg/dL. B. 80 and 120 mg/dL. C. 130 and 150 mg/dL. D. 160 and 200 mg/dL.
B. 80 and 120 mg/dL.
Kussmaul respirations are an indication that the body is: A. Attempting to eliminate acids from the blood. B. Trying to generate energy by breathing deeply. C. Severely hypoxic and is eliminating excess CO2. D. Compensating for decreased blood glucose levels.
A. Attempting to eliminate acids from the blood.
Diabetic ketoacidosis occurs when: A. Blood glucose levels rapidly fall. B. The cells rapidly metabolize glucose. C. The pancreas produces excess insulin. D. Insulin is not available in the body.
D. Insulin is not available in the body.
A 28-year-old female patient is found to be responsive to verbal stimuli only. Her roommate states that she was recently diagnosed with type 1 diabetes and has had difficulty controlling her blood sugar level. She further tells you that the patient has been urinating excessively and has progressively worsened over the last 24 to 36 hours. On the basis of this patient's clinical presentation, you should suspect that she: A. Is significantly hyperglycemic. B. Has a low blood glucose level. C. Has a urinary tract infection. D. Has overdosed on her insulin
A. Is significantly hyperglycemic.
Diabetic coma is a life-threatening condition that results from: A. Hypoglycemia, excess insulin, and dehydration. B. Hyperglycemia, ketoacidosis, and dehydration. C. Hypoglycemia, dehydration, and ketoacidosis. D. Hyperglycemia, excess insulin, and ketoacidosis.
B. Hyperglycemia, ketoacidosis, and dehydration.
Which of the following statements regarding diabetic coma is correct? A. Diabetic coma can be prevented by taking smaller insulin doses. B. Diabetic coma typically develops over a period of hours or days. C. Patients with low blood glucose levels are prone to diabetic coma. D. Diabetic coma rapidly progresses once hyperglycemia develops.
B. Diabetic coma typically develops over a period of hours or days.
Common signs and symptoms of diabetic coma include all of the following, EXCEPT: A. Warm, dry skin. B. Cool, clammy skin. C. Rapid, thready pulse. D. Acetone breath odor.
B. Cool, clammy skin.
The signs and symptoms of insulin shock are the result of: A. Increased blood glucose levels. B. Fat metabolism within the cells. C. Decreased blood glucose levels. D. Prolonged and severe dehydration.
C. Decreased blood glucose levels.
Insulin shock will MOST likely develop if a patient: A. Takes too much of his or her prescribed insulin. B. Markedly overeats and misses an insulin dose. C. Eats a regular meal followed by mild exertion. D. Misses one or two prescribed insulin injections.
A. Takes too much of his or her prescribed insulin.
Hypoglycemic crisis tends to develop more often and more severely in children because: A. They have larger glucose stores than adults do. B. They do not always eat correctly and on schedule. C. Their cells do not uptake glucose as fast as adults' do. D. Their low activity levels cause rapid glucose depletion.
B. They do not always eat correctly and on schedule.
Classic signs and symptoms of hypoglycemia include: A. Warm, dry skin; irritability; bradycardia; and rapid respirations. B. Cool, clammy skin; weakness; tachycardia; and rapid respirations. C. Warm, dry skin; hunger; abdominal pain; and deep, slow respirations. D. Cold, clammy skin; bradycardia; hunger; and deep, rapid respirations.
B. Cool, clammy skin; weakness; tachycardia; and rapid respirations.
Assessment of a patient with hypoglycemia will MOST likely reveal: A. Sunken Eyes. B. Hyperactivity. C. Warm, dry skin. D. Combativeness.
D. Combativeness.
In contrast to insulin shock, diabetic coma: A. Is rapidly reversible if oral glucose is given. B. Commonly results in excess water retention. C. Can only be corrected in the hospital setting. D. Is a rapidly developing metabolic disturbance.
C. Can only be corrected in the hospital setting.
A 75-year-old male with type 1 diabetes presents with chest pain and a general feeling of weakness. He tells you that he took his insulin today and ate a regular meal approximately 2 hours ago. You should treat this patient as though he is experiencing: A. Hypoglycemia. B. An acute stroke. C. Hyperglycemia. D. A heart attack.
D. A heart attack.
You respond to a residence for a patient who is "not acting right." As you approach the door, the patient, a 35-year-old male, begins shouting profanities at you and your partner while holding a baseball bat. The man is confused and diaphoretic, and is wearing a medical identification bracelet. You should: A. Calm him down so you can assess him. B. Be assertive and talk the patient down. C. Contact medical control for instructions. D. Retreat at once and call law enforcement.
D. Retreat at once and call law enforcement.
A 29-year-old female presents with confusion and disorientation. Her respirations are rapid and shallow and her pulse is 120 beats/min and thready. She is markedly diaphoretic and has an oxygen saturation of 89%. You should: A. Transport immediately. B. Administer oral glucose. C. Provide ventilatory support. D. Treat her for hyperglycemia.
C. Provide ventilatory support.
A man finds his 59-year-old wife unconscious on the couch. He states that she takes medications for type 2 diabetes. He further tells you that his wife has been ill recently and has not eaten for the past 24 hours. Your assessment reveals that the patient is unresponsive. You should: A. Quickly assess for the presence of a medical identification tag. B. Administer 100% oxygen via a nonrebreathing mask. C. Open and maintain her airway and assess breathing. D. Administer oral glucose between her cheek and gum.
C. Open and maintain her airway and assess breathing.
You respond to a movie theater for a 70-year-old male who is confused. His wife tells you he has type 2 diabetes but refuses to take his pills. Your assessment reveals that the patient is diaphoretic, tachycardic, and tachypneic. Initial management for this patient should include: A. Administering one to two tubes of oral glucose. B. Applying a nonrebreathing mask at 15 L/min. C. Assisting the patient with his diabetic medication. D. Performing a rapid exam and obtaining vital signs.
B. Applying a nonrebreathing mask at 15 L/min.
When obtaining a SAMPLE history from a patient with diabetes, it would be MOST important to determine: A. If he or she has had any recent illnesses or excessive stress. B. Approximately how much water the patient drank that day. C. If there is a family history of diabetes or related conditions. D. The name of the physician who prescribed his or her insulin.
A. If he or she has had any recent illnesses or excessive stress.
When assessing an unconscious diabetic patient, the primary visible difference between hyperglycemia and hypoglycemia is the: A. Patient's mental status. B. Rate of the patient's pulse. C. Presence of a medical identification tag. D. Rate and depth of breathing.
D. Rate and depth of breathing.
You are treating a 40-year-old male with a documented blood sugar reading of 300 mg/dL. The patient is semiconscious and breathing shallowly, and is receiving assisted ventilation from your partner. You should recognize that definitive treatment for this patient includes: A. Oxygen. B. Glucagon. C. Insulin. D. Dextrose.
C. Insulin.
A 19-year-old male complains of "not feeling right." His insulin and a syringe are on a nearby table. The patient says he thinks he took his insulin and cannot remember whether he ate. He is also unable to tell you the time or what day it is. The glucometer reads "error" after several attempts to assess his blood glucose level. In addition to administering oxygen, you should: A. Contact medical control and administer oral glucose. B. Assist him with his insulin injection and reassess him. C. Request a paramedic ambulance to administer IV glucose. D. Transport only with close, continuous monitoring en route.
A. Contact medical control and administer oral glucose.
Which of the following conditions is the diabetic patient at an increased risk of developing? A. Blindness. B. Depression. C. Alcoholism. D. Hepatitis B.
A. Blindness.
Glutose is a trade name for: A. Glucotrol. B. Oral glucose. C. Micronase. D. Glucophage.
B. Oral glucose.
To which of the following diabetic patients should you administer oral glucose? A. An unconscious 33-year-old male with cool, clammy skin. B. A conscious 37-year-old female with nausea and vomiting. C. A semiconscious 40-year-old female without a gag reflex. D. A confused 55-year-old male with tachycardia and pallor.
D. A confused 55-year-old male with tachycardia and pallor.
Proper procedure for administering oral glucose to a patient includes all of the following, EXCEPT: A. Assessing the patient's mental status. B. Ensuring the absence of a gag reflex. C. Checking the medication's expiration date. D. Requesting permission from medical control.
B. Ensuring the absence of a gag reflex.
Hemoglobin is: A. The fluid portion of the blood that transports cells throughout the body. B. Essential for the formation of clots, such as when vessel damage occurs. C. Found within the red blood cells and is responsible for carrying oxygen. D. A key component of the blood and is produced in response to an infection.
C. Found within the red blood cells and is responsible for carrying oxygen.
Which of the following statements regarding sickle cell disease is correct? A. Sickle cell disease is an inherited blood disorder that causes the blood to clot too quickly. B. In sickle cell disease, the red blood cells are abnormally shaped and are less able to carry oxygen. C. The red blood cells of patients with sickle cell disease are round and contain hemoglobin A. D. Because of their abnormal shape, red blood cells in patients with sickle cell disease are less apt to lodge in a blood vessel.
B. In sickle cell disease, the red blood cells are abnormally shaped and are less able to carry oxygen.
A 30-year-old female presents with severe acute pain to the left upper quadrant of her abdomen. During your assessment, she tells you that she has sickle cell disease. You should suspect that: A. Her spleen is enlarged because of red blood cell engorgement. B. The pain in her abdomen is referred pain from an enlarged liver. C. She has gastrointestinal bleeding due to large blood vessel rupture. D. She is experiencing an aplastic crisis and needs a blood transfusion.
A. Her spleen is enlarged because of red blood cell engorgement.
An acute accelerated drop in the hemoglobin level, which is caused by red blood cells breaking down at a faster rate than normal, occurs during a(n) __________ crisis. A. Aplastic. B. Hemolytic. C. Vaso-occlusive. D. Splenic sequestration.
B. Hemolytic.
Patients with thrombophilia are at an increased risk for: A. Various Cancers. B. Hemorrhagic Stroke. C. Acute Arterial Rupture. D. Pulmonary Embolism.
D. Pulmonary Embolism.
During your assessment of a 19-year-old male, you are told that he is being treated with factor VIII. This indicates that: A. He has a thrombosis. B. He has hemophilia B. C. He has hemophilia A. D. His blood clots too quickly.
C. He has hemophilia A.
A 42-year-old male is found unresponsive on his couch by a neighbor. During your assessment, you find no signs of trauma and the patient's blood glucose level is 75 mg/dL. His blood pressure is 168/98 mm Hg, his heart rate is 45 beats/min and bounding, and his respirations are 8 breaths/min and irregular. The patient is wearing a medical alert bracelet that states he has hemophilia. You should: A. Administer oxygen via a nonrebreathing mask, apply oral glucose in between his cheek and gum, and transport. B. Suspect that he has internal bleeding and is in shock, administer high-flow oxygen, and transport at once. C. Administer high-flow oxygen, perform a detailed secondary assessment at the scene, and transport promptly. D. Suspect that he has intracranial bleeding, assist his ventilations, and transport rapidly to an appropriate hospital.
D. Suspect that he has intracranial bleeding, assist his ventilations, and transport rapidly to an appropriate hospital.
Colic is pain caused by: A. Irritation of the parietal peritoneum. B. Distention or contraction of hollow organs. C. Blunt trauma to the solid abdominal organs. D. Vigorous palpation of the abdominal quadrants.
B. Distention or contraction of hollow organs.
Pain felt at a location other than its origin is called: A. Referred pain. B. Radiating pain. C. Visceral pain. D. Remote pain.
A. Referred pain.
The parietal peritoneum lines the: A. Retroperitoneal space. B. Lungs and chest cavity. C. Walls of the abdominal cavity. D. Surface of the abdominal organs.
C. Walls of the abdominal cavity.
The MOST common and significant complication associated with an acute abdomen is: A. Peritonitis. B. High Fever. C. Severe Pain. D. Internal Bleeding.
A. Peritonitis.
Which of the following statements regarding the acute abdomen is MOST correct? A. The most common cause of an acute abdomen is inflammation of the gallbladder and liver. B. The parietal peritoneum is typically the first abdominal layer that becomes inflamed or irritated. C. The initial pain associated with an acute abdomen tends to be vague and poorly localized. D. An acute abdomen almost always occurs as the result of blunt trauma to solid abdominal organs.
C. The initial pain associated with an acute abdomen tends to be vague and poorly localized.
Erosion of the stomach or duodenum secondary to overactivity of digestive juices results in: A. Ileus. B. An Ulcer. C. Appendicitis. D. Cholecystitis.
B. An Ulcer.
Pain that is localized to the lower back and/or lower abdominal quadrants is MOST suggestive of: A. Acute pancreatitis. B. An aortic aneurysm. C. A kidney infection. D. Acute appendicitis.
B. An aortic aneurysm.
Which of the following organs lies in the retroperitoneal space? A. Liver. B. Spleen. C. Pancreas. D. Gallbladder.
C. Pancreas.
Which of the following conditions is more common in women than in men? A. Cystitis. B. Hepatitis. C. Pancreatitis. D. Cholecystitis.
A. Cystitis.
Pelvic inflammatory disease (PID) is an infection of the: A. Testicles. B. Gallbladder. C. Urinary Bladder. D. Fallopian Tubes.
D. Fallopian Tubes.
An incarcerated hernia is one that: A. Spontaneously reduces without any surgical intervention. B. Can be pushed back in the body cavity to which it belongs. C. Loses its blood supply due to compression by local tissues. D. Cannot be pushed back within its respective body cavity.
D. Cannot be pushed back within its respective body cavity.
In the presence of ileum, the only way the stomach can empty itself is by: A. Diarrhea. B. Vomiting. C. Muscular Contraction. D. Spontaneous Rupture.
B. Vomiting.
Which of the following is a nonspecific complaint that is commonly associated with an acute abdomen? A. High fever. B. Hematemesis. C. Loss of appetite. D. Lower quadrant pain.
C. Loss of appetite.
Peritonitis may result in shock because: A. Intra-abdominal hemorrhage is typically present. B. Fluid shifts from the bloodstream into body tissues. C. Abdominal distention impairs cardiac contractions. D. Severe pain causes systemic dilation of the vasculature.
B. Fluid shifts from the bloodstream into body tissues.
Which of the following statements regarding an ectopic pregnancy is correct? A. Signs of an ectopic pregnancy usually present within the first 6 to 8 weeks of pregnancy. B. The abdominal cavity is the most common location for abnormal implantation of a fertilized egg. C. An ectopic pregnancy almost always presents with acute signs and symptoms in the third trimester. D. Pain from a ruptured ectopic pregnancy is usually severe but the bleeding is typically rather minimal.
A. Signs of an ectopic pregnancy usually present within the first 6 to 8 weeks of pregnancy.
Common signs and symptoms of acute abdomen include all of the following, EXCEPT: A. Tachycardia. B. Hypertension. C. Nausea and Vomiting. D. Abdominal Guarding.
B. Hypertension.
When asking a patient if his or her abdominal pain "moves anywhere," you are establishing the presence or absence of: A. Referred pain. B. Provoking pain. C. Palliating pain. D. Radiating pain.
D. Radiating pain.
Patients with acute abdominal pain should not be given anything by mouth because: A. It will create referred pain and obscure the diagnosis. B. Food will rapidly travel through the digestive system. C. A full stomach may make emergency surgery dangerous. D. Digestion prevents accurate auscultation of bowel sounds.
C. A full stomach may make emergency surgery dangerous.
When assessing a patient with right lower quadrant pain, you should first palpate the: A. Left upper quadrant. B. Left lower quadrant. C. Right upper quadrant. D. Right lower quadrant.
A. Left upper quadrant.
Which of the following is a late sign of shock in a patient with acute abdominal pain? A. Hypotension. B. Tachycardia. C. Tachypnea. D. Diaphoresis.
A. Hypotension.
A 35-year-old mildly obese woman is complaining of localized pain in the right upper quadrant with referred pain to the right shoulder. The MOST likely cause of her pain is: A. Acute cystitis. B. Cholecystitis. C. Appendicitis. D. Pancreatitis.
B. Cholecystitis.
A 59-year-old male presents with a sudden onset of severe lower back pain. He is conscious and alert, but very restless and diaphoretic. Your assessment reveals a pulsating mass to the left of his umbilicus. You should: A. Vigorously palpate the abdomen to establish pain severity. B. Administer oxygen and prepare for immediate transport. C. Place the patient in a sitting position and transport at once. D. Request a paramedic unit to give the patient pain medication.
B. Administer oxygen and prepare for immediate transport.
You are caring for a 25-year-old female with acute left lower quadrant pain. She is conscious and alert and in obvious pain. Her blood pressure is 110/70 mm Hg, pulse is 100 beats/min and regular, and respirations are 24 and unlabored. In addition to administering oxygen, you should: A. Place her in Trendelenburg's position. B. Request permission to give her Motrin. C. Give her small sips of water to drink. D. Place her in a comfortable position.
D. Place her in a comfortable position.
A 47-year-old male presents with severe abdominal pain of 3 hours' duration. His abdomen is distended and guarded. Your MOST important consideration for this patient should be to: A. Transport him in a supine position. B. Be alert for signs and symptoms of shock. C. Assess his BP to determine perfusion adequacy. D. Determine the exact location and cause of his pain.
B. Be alert for signs and symptoms of shock.
An important aspect in the treatment of a patient with severe abdominal pain is to: A. Provide emotional support en route to the hospital. B. Administer analgesic medications to alleviate pain. C. Encourage the patient to remain in a supine position. D. Give 100% oxygen only if signs of shock are present.
A. Provide emotional support en route to the hospital.
A 66-year-old female complains of diffuse abdominal pain and nausea. She has a history of hypertension and occasional depression. When caring for this patient, you should be MOST concerned with: A. The possibility that she may vomit. B. Determining the origin of her pain. C. The emotional effects caused by the pain. D. An acute rise in her systolic blood pressure.
A. The possibility that she may vomit.
The MOST appropriate treatment for a patient with severe abdominal pain and signs of shock includes: A. Administering oxygen via a nasal cannula. B. Giving oral fluids to maintain perfusion. C. Transporting the patient without delay. D. Positioning the patient on his or her side.
C. Transporting the patient without delay.
You are transporting a 49-year-old male with "tearing" abdominal pain. You are approximately 30 miles away from the closest hospital. During your ongoing assessment, you determine that the patient's condition has deteriorated significantly. You should: A. Begin ventilatory assistance with a BVM device. B. Immediately perform a detailed physical examination. C. Continue transporting and alert the receiving hospital. D. Consider requesting a rendezvous with an ALS unit.
D. Consider requesting a rendezvous with an ALS unit.
A young female presents with costovertebral angle tenderness. She is conscious and alert with stable vital signs. Which of the following organs is MOST likely causing her pain? A. Liver. B. Kidney. C. Pancreas. D. Gallbladder.
B. Kidney.
You are dispatched to an apartment complex for a young male with abdominal pain. Your priority upon arriving at the scene should be to: A. Quickly gain access to the patient. B. Notify the dispatcher of your arrival. C. Assess the scene for potential hazards. D. Place a paramedic ambulance on standby.
C. Assess the scene for potential hazards.
Solid abdominal organs include the: A. Stomach and small intestine. B. Spleen, kidneys, and pancreas. C. Gallbladder and large intestine. D. Urinary bladder, colon, and ureters.
B. Spleen, kidneys, and pancreas.
Which of the following organs would MOST likely bleed profusely if injured? A. Liver. B. Stomach. C. Appendix. D. Gallbladder.
A. Liver.
Which of the following organs lies in the retroperitoneal space? A. Liver. B. Spleen. C. Pancreas. D. Gallbladder.
C. Pancreas.
Injury to a hollow abdominal organ would MOST likely result in: A. Pain secondary to blood in the peritoneum. B. Profound shock due to severe internal bleeding. C. Impairment in the blood's clotting abilities. D. Leakage of contents into the abdominal cavity.
D. Leakage of contents into the abdominal cavity.
Functions of the liver include: A. Storage of bile, which is produced in the gallbladder. B. Production of substances necessary for blood clotting. C. Production of hormones that regulate blood sugar levels. D. Release of amylase, which breaks down starches into sugar.
B. Production of substances necessary for blood clotting.
Which of the following organs assists in the filtration of blood, serves as a blood reservoir, and produces antibodies? A. Liver. B. Kidney. C. Spleen. D. Pancreas.
C. Spleen.
The kidneys help to regulate blood pressure by: A. Retaining key electrolytes, such as potassium. B. Eliminating toxic waste products from the body. C. Removing sodium, and thus water, from the body. D. Accommodating a large amount of blood volume.
C. Removing sodium, and thus water, from the body.
The MOST common and significant complication associated with an acute abdomen is: A. Peritonitis. B. High Fever. C. Severe Pain. D. Internal Bleeding.
A. Peritonitis.
The parietal peritoneum lines the: A. Retroperitoneal space. B. Lungs and chest cavity. C. Walls of the abdominal cavity. D. Surface of the abdominal organs.
C. Walls of the abdominal cavity.
In the presence of ileum, the only way the stomach can empty itself is by: A. Diarrhea. B. Vomiting. C. Muscular Contraction. D. Spontaneous Rupture.
B. Vomiting.
Peritonitis may result in shock because: A. Intra-abdominal hemorrhage is typically present. B. Fluid shifts from the bloodstream into body tissues. C. Abdominal distention impairs cardiac contractions. D. Severe pain causes systemic dilation of the vasculature.
B. Fluid shifts from the bloodstream into body tissues.
In contrast to the parietal peritoneum, the visceral peritoneum: A. Is supplied by the same nerves from the spinal cord that supply the skin of the abdomen. B. Lines the walls of the abdominal cavity and is stimulated when the solid abdominal organs contract. C. Is less likely to become inflamed or infected because it lines the abdominal organs themselves. D. Is supplied by nerves of the autonomic nervous system, which are less able to localize pain or sensation.
D. Is supplied by nerves of the autonomic nervous system, which are less able to localize pain or sensation.
A 35-year-old mildly obese woman is complaining of localized pain in the right upper quadrant with referred pain to the right shoulder. The MOST likely cause of her pain is: A. Acute cystitis. B. Acute cholecystitis. C. Appendicitis. D. Pancreatitis.
B. Acute cholecystitis.
Pain that may be perceived at a distant point on the surface of the body, such as the back or shoulder, is called: A. Referred Pain. B. Radiating Pain. C. Visceral Pain. D. Remote Pain.
A. Referred Pain.
Pain that is localized to the lower back and/or lower abdominal quadrants is MOST suggestive of: A. Acute pancreatitis. B. An aortic aneurysm. C. A kidney infection. D. Acute appendicitis.
B. An aortic aneurysm.
A young female presents with costovertebral angle tenderness. She is conscious and alert with stable vital signs. Which of the following organs is MOST likely causing her pain? A. Liver. B. Kidney. C. Pancreas. D. Gallbladder.
B. Kidney.
Which of the following statements regarding the acute abdomen is correct? A. The most common cause of an acute abdomen is inflammation of the gallbladder and liver. B. The parietal peritoneum is typically the first abdominal layer that becomes inflamed or irritated. C. The initial pain associated with an acute abdomen tends to be vague and poorly localized. D. An acute abdomen almost always occurs as the result of blunt trauma to solid abdominal organs.
C. The initial pain associated with an acute abdomen tends to be vague and poorly localized.
Which of the following is characteristic of peptic ulcer disease (PUD)? A. The passage of bright red blood in the stool or coughing up blood. B. Symptom relief after taking non steroidal anti-inflammatory drugs. C. Sharp pain that is typically located in both lower abdominal quadrants. D. Burning or pain in the stomach that subsides immediately after eating.
D. Burning or pain in the stomach that subsides immediately after eating.
Erosion of the protective layer of the stomach or duodenum secondary to overactivity of digestive juices results in: A. Ileus. B. An Ulcer. C. Appendicitis. D. Cholecystitis.
B. An Ulcer.
Pain that radiates to the right lower quadrant from the umbilical area, nausea and vomiting, and anorexia are MOST indicative of: A. Pancreatitis. B. Appendicitis. C. Cholecystitis. D. Gastroenteritis.
B. Appendicitis.
Esophageal varicose MOST commonly occur in patients who: A. Drink a lot of alcohol. B. Have severe diabetes. C. Have a history of esophagitis. D. Have weak immune systems.
A. Drink a lot of alcohol.
Which of the following statements regarding gastrointestinal bleeding is correct? A. In the majority of cases, bleeding within the gastrointestinal tract occurs acutely and is severe. B. Bleeding within the gastrointestinal tract is a symptom of another disease, not a disease itself. C. Lower gastrointestinal bleeding results from conditions such as Mallory-Weiss syndrome. D. Chronic bleeding within the gastrointestinal tract is usually more severe than bleeding that occurs acutely.
B. Bleeding within the gastrointestinal tract is a symptom of another disease, not a disease itself.
Which of the following conditions is more common in women than in men? A. Cystitis. B. Hepatitis. C. Pancreatitis. D. Cholecystitis.
A. Cystitis.
A strangulated hernia is one that: A. Spontaneously reduces without any surgical intervention. B. Can be pushed back into the body cavity to which it belongs. C. Is reducible if surgical intervention occurs within 2 hours. D. Loses its blood supply due to compression by local tissues.
D. Loses its blood supply due to compression by local tissues.
Chronic renal failure is a condition that: A. Can be reversed with prompt treatment. B. Occurs from conditions such as dehydration. C. Is often caused by hypertension or diabetes. D. Causes dehydration from excessive urination.
C. Is often caused by hypertension or diabetes.
You are dispatched to an apartment complex for a young male with abdominal pain. Your priority upon arriving at the scene should be to: A. Quickly gain access to the patient. B. Notify the dispatcher of your arrival. C. Assess the scene for potential hazards. D. Place a paramedic ambulance on standby.
C. Assess the scene for potential hazards.
Most patients with abdominal pain prefer to: A. Lie on their side with their knees drawn into the abdomen. B. Sit in a semi-Fowler position with their knees slightly bent. C. Lie in a supine position with their knees in a flexed position. D. Sit fully upright because it helps relax the abdominal muscles.
A. Lie on their side with their knees drawn into the abdomen.
Patients with acute abdominal pain should not be given anything to eat or drink because: A. It will create referred pain and obscure the diagnosis. B. Food will rapidly travel through the digestive system. C. Substances in the stomach increase the risk of aspiration. D. Digestion prevents accurate auscultation of bowel sounds.
C. Substances in the stomach increase the risk of aspiration.
A 30-year-old woman with a history of alcoholism presents with severe upper abdominal pain and is vomiting large amounts of bright red blood. Her skin is cool, pale, and clammy; her heart rate is 120 beats/min and weak; and her blood pressure is 70/50 mm Hg. Your MOST immediate action should be to: A. Protect her airway from aspiration. B. Keep her supine and elevate her legs. C. Rapidly transport her to the hospital. D. Give her high-flow supplemental oxygen.
A. Protect her airway from aspiration.
When assessing a patient with abdominal pain, you should: A. Palpate the abdomen in a clockwise direction beginning with the quadrant after the one the patient indicates is painful. B. Ask the patient to point to the area of pain or tenderness and assess for rebound tenderness over that specific area. C. Visually assess the painful area of the abdomen, but avoid palpation because this could worsen his or her condition. D. Observe for abdominal guarding, which is characterized by sudden relaxation of the abdominal muscles when palpated.
A. Palpate the abdomen in a clockwise direction beginning with the quadrant after the one the patient indicates is painful.
Elderly patients with abdominal problems may not exhibit the same pain response as younger patients because of: A. Chronic dementia, which inhibits communication. B. Interactions of the numerous medications they take. C. Progressive deterioration of abdominal organ function. D. Age-related deterioration of their sensory systems.
D. Age-related deterioration of their sensory systems.
An important aspect in the treatment of a patient with severe abdominal pain is to: A. Provide emotional support en route to the hospital. B. Administer analgesic medications to alleviate pain. C. Encourage the patient to remain in a supine position. D. Give 100% oxygen only if signs of shock are present.
A. Provide emotional support en route to the hospital.
A 47-year-old male presents with severe abdominal pain of 3 hours' duration. His abdomen is distended and guarded. Your MOST important consideration for this patient should be to: A. Transport him in a supine position. B. Be alert for signs and symptoms of shock. C. Assess his blood pressure to determine perfusion adequacy. D. Determine the exact location and cause of his pain.
B. Be alert for signs and symptoms of shock.
The MOST important treatment for a patient with severe abdominal pain and signs of shock includes: A. Administering high-flow oxygen. B. Giving oral fluids to maintain perfusion. C. Transporting the patient without delay. D. Positioning the patient on his or her side.
C. Transporting the patient without delay.
A 59-year-old male presents with a sudden onset of severe lower back pain. He is conscious and alert, but very restless and diaphoretic. Your assessment reveals a pulsating mass to the left of his umbilicus. You should: A. Vigorously palpate the abdomen to establish pain severity. B. Administer oxygen and prepare for immediate transport. C. Place the patient in a sitting position and transport at once. D. Request a paramedic unit to give the patient pain medication.
B. Administer oxygen and prepare for immediate transport.
You are transporting a 49-year-old male with "tearing" abdominal pain. You are approximately 30 miles away from the closest hospital. During your reassessment, you determine that the patient's condition has deteriorated significantly. You should: A. Assist his ventilations with a bag-mask device. B. Immediately perform a rapid physical examination. C. Continue transporting and alert the receiving hospital. D. Consider requesting a rendezvous with an ALS unit.
D. Consider requesting a rendezvous with an ALS unit.
Which of the following statements regarding dialysis is correct? A. Acute hypertension is a common adverse effect of dialysis. B. Hemodialysis is effective but carries a high risk of peritonitis. C. Patients who miss a dialysis treatment often present with weakness. D. The purpose of dialysis is to help the kidneys retain salt and water.
C. Patients who miss a dialysis treatment often present with weakness.
You respond to the residence of a 70-year-old male who complains of weakness and severe shortness of breath. His wife tells you that he is a dialysis patient, but has missed his last two treatments. After applying high-flow oxygen, you auscultate his lungs and hear diffuse rhonchi. The patient is conscious, but appears confused. His blood pressure is 98/54 mm Hg, his pulse rate is 120 beats/min and irregular, and his respirations are 24 breaths/min and labored. You should: A. Leave him in a sitting position, keep him warm, and prepare for immediate transport. B. Place him in a supine position, elevate his lower extremities, and transport at once. C. Treat for shock and request a paramedic unit to respond to the scene and assist you. D. Perform a detailed secondary assessment and then transport him to a dialysis center.
A. Leave him in a sitting position, keep him warm, and prepare for immediate transport.
71. Peritonitis may result in shock because: A. Severe pain causes systemic dilation of the vasculature. B. Abdominal distention impairs cardiac contractions. C. Fluid shifts from the bloodstream into body tissues. D. Intra-abdominal hemorrhage is typically present.
C. Fluid shifts from the bloodstream into body tissues.
Patients with acute abdominal pain should not be given anything to eat or drink because: A. Substances in the stomach increase the risk of aspiration. B. It will create referred pain and obscure the diagnosis. C. Food will rapidly travel through the digestive system. D. Digestion prevents accurate auscultation of bowel sounds.
A. Substances in the stomach increase the risk of aspiration.
An important aspect in the treatment of a patient with severe abdominal pain is to: A. Encourage the patient to remain in a supine position. B. Administer analgesic medications to alleviate pain. C. Provide emotional support en route to the hospital. D. Give 100% oxygen only if signs of shock are present.
C. Provide emotional support en route to the hospital.
Peritonitis may result in shock because: A. Severe pain causes systemic dilation of the vasculature. B. Abdominal distention impairs cardiac contractions. C. Fluid shifts from the bloodstream into body tissues. D. Intra-abdominal hemorrhage is typically present.
C. Fluid shifts from the bloodstream into body tissues.
Chronic renal failure is a condition that: A. Is often caused by hypertension or diabetes. B. Occurs from conditions such as dehydration. C. Causes dehydration from excessive urination. D. Can be reversed with prompt treatment.
A. Is often caused by hypertension or diabetes.
A 47-year-old male presents with severe abdominal pain of 3 hours' duration. His abdomen is distended and guarded. Your MOST important consideration for this patient should be to: A. Transport him in a supine position. B. Determine the exact location and cause of his pain. C. Be alert for signs and symptoms of shock. D. Assess his blood pressure to determine perfusion adequacy.
C. Be alert for signs and symptoms of shock.
Pain that radiates to the right lower quadrant from the umbilical area, nausea and vomiting, and anorexia are MOST indicative of: A. Pancreatitis. B. Appendicitis. C. Cholecystitis. D. Gastroenteritis.
B. Appendicitis.
A strangulated hernia is one that: A. Loses its blood supply due to compression by local tissues. B. Spontaneously reduces without any surgical intervention. C. Is reducible if surgical intervention occurs within 2 hours. D. Can be pushed back into the body cavity to which it belongs.
A. Loses its blood supply due to compression by local tissues.
A 59-year-old male presents with a sudden onset of severe lower back pain. He is conscious and alert, but very restless and diaphoretic. Your assessment reveals a pulsating mass to the left of his umbilicus. You should: A. Administer oxygen and prepare for immediate transport. B. Request a paramedic unit to give the patient pain medication. C. Place the patient in a sitting position and transport at once. D. Vigorously palpate the abdomen to establish pain severity.
A. Administer oxygen and prepare for immediate transport.
A strangulated hernia is one that: A. Loses its blood supply due to compression by local tissues. B. Spontaneously reduces without any surgical intervention. C. Is reducible if surgical intervention occurs within 2 hours. D. Can be pushed back into the body cavity to which it belongs.
A. Loses its blood supply due to compression by local tissues.
A 30-year-old woman with a history of alcoholism presents with severe upper abdominal pain and is vomiting large amounts of bright red blood. Her skin is cool, pale, and clammy; her heart rate is 120 beats/min and weak; and her blood pressure is 70/50 mm Hg. Your MOST immediate action should be to: A. Keep her supine and elevate her legs. B. Rapidly transport her to the hospital. C. Protect her airway from aspiration. D. Give her high-flow supplemental oxygen.
C. Protect her airway from aspiration.
Most patients with abdominal pain prefer to: A. Lie on their side with their knees drawn into the abdomen. B. Lie in a supine position with their knees in a flexed position. C. Sit fully upright because it helps relax the abdominal muscles. D. Sit in a semi-Fowler position with their knees slightly bent.
A. Lie on their side with their knees drawn into the abdomen.
Functions of the liver include: A. Production of hormones that regulate blood sugar levels. B. Storage of bile, which is produced in the gallbladder. C. Production of substances necessary for blood clotting. D. Release of amylase, which breaks down starches into sugar.
C. Production of substances necessary for blood clotting.
A 35-year-old mildly obese woman is complaining of localized pain in the right upper quadrant with referred pain to the right shoulder. The MOST likely cause of her pain is: A. Acute Cholecystitis. B. Appendicitis. C. Acute Cystitis. D. Pancreatitis.
A. Acute Cholecystitis.
Esophageal varicose MOST commonly occur in patients who: A. Drink a lot of alcohol. B. Have severe diabetes. C. Have weak immune systems. D. Have a history of esophagitis.
A. Drink a lot of alcohol.
Pain that is localized to the lower back and/or lower abdominal quadrants is MOST suggestive of: A. Acute appendicitis. B. A kidney infection. C. Acute pancreatitis. D. An aortic aneurysm.
D. An aortic aneurysm.
Erosion of the protective layer of the stomach or duodenum secondary to overactivity of digestive juices results in: A. Cholecystitis. B. An Ulcer. C. Ileus. D. Appendicitis.
B. An Ulcer.
The kidneys help to regulate blood pressure by: A. Removing sodium, and thus water, from the body. B. Accommodating a large amount of blood volume. C. Eliminating toxic waste products from the body. D. Retaining key electrolytes, such as potassium.
A. Removing sodium, and thus water, from the body.
The MOST common and significant complication associated with an acute abdomen is: A. Peritonitis. B. Severe pain. C. Internal bleeding. D. High fever.
A. Peritonitis.
Which of the following statements regarding the acute abdomen is correct? A. The parietal peritoneum is typically the first abdominal layer that becomes inflamed or irritated. B. An acute abdomen almost always occurs as the result of blunt trauma to solid abdominal organs. C. The most common cause of an acute abdomen is inflammation of the gallbladder and liver. D. The initial pain associated with an acute abdomen tends to be vague and poorly localized.
D. The initial pain associated with an acute abdomen tends to be vague and poorly localized.
Erosion of the protective layer of the stomach or duodenum secondary to overactivity of digestive juices results in: A. Ileus. B. Appendicitis. C. Cholecystitis. D. An ulcer.
D. An ulcer.
Pain that radiates to the right lower quadrant from the umbilical area, nausea and vomiting, and anorexia are MOST indicative of: A. Cholecystitis. B. Gastroenteritis. C. Pancreatitis. D. Appendicitis.
D. Appendicitis.
Esophageal varicose MOST commonly occur in patients who: A. Drink a lot of alcohol. B. Have weak immune systems. C. Have severe diabetes. D. Have a history of esophagitis.
A. Drink a lot of alcohol.
You are dispatched to an apartment complex for a young male with abdominal pain. Your priority upon arriving at the scene should be to: A. Quickly gain access to the patient. B. Place a paramedic ambulance on standby. C. Assess the scene for potential hazards. D. Notify the dispatcher of your arrival.
C. Assess the scene for potential hazards.
A 30-year-old woman with a history of alcoholism presents with severe upper abdominal pain and is vomiting large amounts of bright red blood. Her skin is cool, pale, and clammy; her heart rate is 120 beats/min and weak; and her blood pressure is 70/50 mm Hg. Your MOST immediate action should be to: A. Protect her airway from aspiration. B. Give her high-flow supplemental oxygen. C. Keep her supine and elevate her legs. D. Rapidly transport her to the hospital.
A. Protect her airway from aspiration.
Patients with acute abdominal pain should not be given anything to eat or drink because: A. Digestion prevents accurate auscultation of bowel sounds. B. Substances in the stomach increase the risk of aspiration. C. Food will rapidly travel through the digestive system. D. It will create referred pain and obscure the diagnosis.
B. Substances in the stomach increase the risk of aspiration.
When assessing a patient with abdominal pain, you should: A. Observe for abdominal guarding, which is characterized by sudden relaxation of the abdominal muscles when palpated. B. Palpate the abdomen in a clockwise direction beginning with the quadrant after the one the patient indicates is painful. C. Visually assess the painful area of the abdomen, but avoid palpation because this could worsen his or her condition. D. Ask the patient to point to the area of pain or tenderness and assess for rebound tenderness over that specific area.
B. Palpate the abdomen in a clockwise direction beginning with the quadrant after the one the patient indicates is painful.
Remember that the patient is in pain and is probably anxious; he or she requires all your skills of rapid assessment and: A. Rapid transport. B. Quick communication to medical control. C. Emotional support. D. Advanced life support care.
C. Emotional support.
Diverticulitis occurs in which quadrant? A. Left upper quadrant B. Left lower quadrant C. Right upper quadrant D. Right lower quadrant
B. Left lower quadrant
An aortic aneurysm will occur in which area of the body? A. Lower back and both left quadrants. B. Lower back and lower quadrants. C. Upper back and lower quadrants. D. Upper back and upper quadrants.
B. Lower back and lower quadrants.
When forming a general impression of the patient during the initial assessment, which important "scale" information should be included? A. PAT scale. B. rule of nines scale. C. SARS scale. D. AVPU scale.
D. AVPU scale.
Use OPQRST to ask the patient what makes the pain better or worse. The "S" stands for: A. How sick the patient feels. B. The sudden onset of pain. C. Secondary survey questions. D. The severity of pain.
D. The severity of pain.
During the baseline vital sign check, the last vital sign to change is: A. Respiratory Function. B. Blood Pressure. C. Unconsciousness. D. Pulse Rate.
B. Blood Pressure.
Remember, the condition of a patient with an acute abdomen can change: A. Rapidly from stable to unstable. B. Slowly, even if transport is delayed. C. Quickly if you do not administer an IV. D. None of the above.
A. Rapidly from stable to unstable.
During the focused physical exam, the first step listed to assess the abdomen is: A. Palpate all four quadrants. B. Determine if the patient has a history of acute abdomen. C. Determine whether the abdomen is tender when palpated. D. Explain to the patient what you are about to do.
A. Palpate all four quadrants.
Where will you find localization of pain for a patient with a kidney stone? A. Radiating to the genitalia. B. Involving the spleen. C. In the chest. D. In the lower back.
A. Radiating to the genitalia
A bladder infection called cystitis is more common than kidney infection, especially in women. Patients with cystitis usually have: A. Back Pain. B. Groin Pain. C. Lower Abdominal Pain. D. Upper Abdominal Pain.
B. Groin Pain.
When the waste product that is normally excreted into the urine remains in the blood, this can lead to which of the following conditions? A. Uremia. B. Kidney Stones. C. Cystitis. D. All of the above.
A. Uremia.
The type of pain where the patient experiences tenderness when direct pressure is applied but severe pain when that pressure is released is called: A. Rebound Tenderness. B. Guarding. C. Strangulation. D. Aa hernia.
A. Rebound Tenderness.
In which condition does the junction between the esophagus and the stomach tear, causing severe bleeding and potentially death? A. Gastroenteritis. B. Diverticulitis. C. Mallory-Weiss syndrome. D. Hernia.
C. Mallory-Weiss syndrome.
The three major parts of the brain are the: A. Cerebellum, medulla, and occiput. B. Brain stem, midbrain, and spinal cord. C. Midbrain, cerebellum, and spinal cord. D. Cerebrum, cerebellum, and brain stem.
D. Cerebrum, cerebellum, and brain stem.
The most basic functions of the body, such as breathing, blood pressure, and swallowing, are controlled by the: A. Brain Stem. B. Cerebrum. C. Cerebellum. D. Cerebral Cortex.
A. Brain Stem.
The anterior aspect of the cerebrum controls: A. Touch. B. Emotion. C. Movement. D. Vision.
B. Emotion.
Muscle control and body coordination are controlled by the: A. Cerebrum. B. Cerebellum. C. Brain stem. D. Cerebral cortex.
B. Cerebellum.
The left cerebral hemisphere controls: A. The right side of the face. B. Heart rate and pupil reaction. C. The right side of the body. D. Breathing and blood pressure.
C. The right side of the body.
The spinal cord exits the cranium through the: A. Foramen Magnum. B. Vertebral Foramen. C. Foramen Lamina. D. Cauda Equina.
A. Foramen Magnum.
Which of the following conditions would MOST likely affect the entire brain? A. Blocked cerebral artery in the frontal lobe. B. Reduced blood supply to the left hemisphere. C. Ruptured cerebral artery in the occipital lobe. D. Respiratory failure or cardiopulmonary arrest.
D. Respiratory failure or cardiopulmonary arrest.
Interruption of cerebral blood flow may result from all of the following, EXCEPT: A. A thrombus. B. An embolism. C. Cerebral vasodilation. D. An acute arterial rupture.
C. Cerebral vasodilation.
The MOST significant risk factor for a hemorrhagic stroke is: A. Severe stress. B. Hypertension. C. Heavy exertion. D. Diabetes mellitus.
B. Hypertension.
An area of swelling or enlargement in a weakened arterial wall is called: A. A thrombus. B. An aneurysm. C. An embolism. D. Atherosclerosis.
B. An aneurysm.
Which of the following clinical signs would be MOST suggestive of a ruptured cerebral artery? A. Unilateral hemiparesis. B. Confusion and weakness. C. Nasal discharge of blood. D. Sudden, severe headache.
D. Sudden, severe headache.
Which of the following MOST accurately describes the cause of an ischemic stroke? A. Blockage of a cerebral artery B. Acute atherosclerotic disease C. Rupture of a cerebral artery D. Narrowing of a carotid artery
A. Blockage of a cerebral artery
A transient ischemic attack (TIA) occurs when: A. Medications are given to dissolve a cerebral blood clot. B. A small cerebral artery ruptures and causes minimal damage. C. The normal body processes destroy a clot in a cerebral artery. D. Signs and symptoms resolve spontaneously within 48 hours.
C. The normal body processes destroy a clot in a cerebral artery.
A patient who is experiencing receptive aphasia is: A. Unable to speak clearly or understand questions. B. Most likely experiencing a right-hemispheric stroke. C. Able to speak clearly but has difficulty understanding. D. Able to understand questions but cannot speak clearly.
C. Able to speak clearly but has difficulty understanding.
A patient whose speech is slurred and difficult to understand is experiencing: A. Aphasia. B. Dysarthria. C. Dysphagia. D. Paraplegia.
B. Dysarthria.
Which of the following conditions would be the LEAST likely to mimic the signs and symptoms of a stroke? A. Hypoglycemia. B. Hypovolemia. C. Postictal state. D. Intracranial bleeding.
B. Hypovolemia.
Successful treatment of a stroke depends on whether or not: A. Surgical intervention is performed to remove obstructive clots. B. Medications are given to restore the function of infarcted cells. C. The stroke occurs within the left or right hemisphere of the brain. D. Clot-buster therapy is given within 2 to 3 hours of symptom onset.
D. Clot-buster therapy is given within 2 to 3 hours of symptom onset.
When obtaining medical history information from the family of a suspected stroke patient, it is MOST important to determine: A. When the patient last appeared normal. B. If there is a family history of a stroke. C. If the patient has been hospitalized before. D. The patient's overall medication compliance.
A. When the patient last appeared normal.
When transporting a stable stroke patient with unilateral hemiparalysis, it is best to place the patient in a: A. Recumbent position with the paralyzed side up. B. Recumbent position with the paralyzed side down. C. Sitting position with the head at a 45° to 90° angle. D. Supine position with the legs elevated 6" to 12".
B. Recumbent position with the paralyzed side down.
Which of the following is NOT an assessment parameter included in the Cincinnati Stroke Scale? A. Speech. B. Arm drift. C. Memory. D. Facial droop.
C. Memory.
When assessing arm movement of a patient with a suspected stroke, you should: A. Observe for arm movement for approximately 5 minutes. B. Expect to see one arm slowly drift down to the patient's side. C. Ask the patient to close his or her eyes during the assessment. D. Ask the patient to hold his or her arms up with the palms down.
C. Ask the patient to close his or her eyes during the assessment.
What Glasgow Coma Scale (GCS) score would you assign to a patient who responds to painful stimuli, uses inappropriate words, and flexes his or her arms in response to pain? A. 6. B. 8. C. 9. D. 10.
B. 8.
A patient who is possibly experiencing a stroke is NOT eligible for thrombolytic (fibrinolytic) therapy if he or she: A. Has had a prior heart attack. B. Is older than 60 years of age. C. Has a GCS that is less than 8. D. Has bleeding within the brain.
D. Has bleeding within the brain.
A generalized seizure is characterized by: A. Severe twitching of all the body's muscles. B. A blank stare and brief lapse of consciousness. C. Unconsciousness for greater than 30 minutes. D. A core body temperature of greater than 103°F.
A. Severe twitching of all the body's muscles.
An absence seizure is also referred to as a: A. Petit mal seizure. B. Grand mal seizure. C. Total body seizure. D. Generalized motor seizure.
A. Petit mal seizure.
Which of the following MOST accurately describes the postictal state that follows a seizure? A. Hyperventilation and hypersalivation. B. Rapidly improving level of consciousness. C. Dazed, confused, or combative appearance. D. Gradually decreasing level of consciousness.
C. Dazed, confused, or combative appearance.
Which of the following medications is NOT used to treat patients with a history of seizures? A. Dilantin. B. Dilaudid. C. Tegretol. D. Phenobarbital.
B. Dilaudid.
Which of the following conditions is NOT a common cause of seizures? A. Poisoning or overdose. B. Severe hypovolemia. C. Acute hypoglycemia. D. Acute alcohol withdrawal.
B. Severe hypovolemia.
Status epileptics is characterized by: A. Generalized seizures that last less than 5 minutes. B. An absence seizure that is not preceded by an aura. C. Profound tachycardia and total muscle flaccidity. D. Prolonged seizures without a return of consciousness.
D. Prolonged seizures without a return of consciousness.
Febrile seizures: A. Often result in permanent brain damage. B. Are also referred to as petit map seizures. C. Are usually benign but should be evaluated. D. Occur when a child's fever progressively rises.
C. Are usually benign but should be evaluated.
Which of the following is a metabolic cause of a seizure? A. Poisoning. B. Head trauma. C. Brain tumor. D. Massive stroke.
A. Poisoning.
A patient without a history of seizures experiences a sudden convulsion. The LEAST likely cause of this seizure is: A. Epilepsy. B. Brain Tumor. C. Serious Infection. D. Intracranial Bleeding.
A. Epilepsy.
Which of the following MOST accurately describes a focal seizure? A. A seizure that begins in one extremity. B. A blank stare without muscle contractions. C. A seizure that is not preceded by an aura. D. Generalized seizures without incontinence.
A. A seizure that begins in one extremity.
A patient with an altered mental status is: A. Completely unresponsive to all forms of stimuli. B. Typically alert but is confused as to preceding events. C. Usually able to be aroused with a painful stimulus. D. Not thinking clearly or is incapable of being aroused.
D. Not thinking clearly or is incapable of being aroused.
Which of the following conditions would MOST likely mimic the signs and symptoms of a stroke? A. High fever. B. Hypoglycemia. C. Acute poisoning. D. Alcohol intoxication.
B. Hypoglycemia.
The principle clinical difference between a stroke and hypoglycemia is that patients with hypoglycemia: A. Always take oral medications to maintain normal blood glucose levels. B. Do not present with slurred speech or weakness to one side of the body. C. Are typically alert and attempt to communicate with healthcare providers. D. Usually have an altered mental status or decreased level of consciousness.
D. Usually have an altered mental status or decreased level of consciousness.
When caring for a child with documented hypoglycemia, you should be MOST alert for: A. A seizure. B. An acute stroke. C. Respiratory distress. D. A febrile convulsion.
A. A seizure.
Individuals with chronic alcoholism are predisposed to intracranial bleeding and hypoglycemia secondary to abnormalities in the: A. Brain. B. Kidneys. C. Liver. D. Pancreas.
C. Liver.
The mental status of a patient who has experienced a typical seizure: A. Progressively worsens over a period of a few hours. B. Is easily differentiated from that of acute hypoglycemia. C. Is likely to improve over a period of 5 to 30 minutes. D. Typically does not improve, even after several minutes.
C. Is likely to improve over a period of 5 to 30 minutes.
Which of the following patients would MOST likely demonstrate typical signs of infection or fever? A. A 3-month-old female who was born prematurely B. A 17-year-old male with depression and anxiety C. A 35-year-old female in the later stages of AIDS D. An 88-year-old male with chronic renal problems
B. A 17-year-old male with depression and anxiety.
During the initial assessment of a semiconscious 70-year-old female, you should: A. Insert a nasopharyngeal airway and assist ventilations. B. Immediately determine the patient's blood glucose level. C. Ask family members if the patient has a history of stroke. D. Ensure a patent airway and support ventilation as needed.
D. Ensure a patent airway and support ventilation as needed.
You are dispatched to a residence for a 66-year-old male who, according to family members, has suffered a massive stroke. Your initial assessment reveals that the patient is pulseless and apneic. You should: A. Assess the patient for a facial droop and hemiparesis. B. Initiate CPR and attach an AED as soon as possible. C. Obtain a blood glucose sample to rule out hypoglycemia. D. Hyperventilate the patient to reduce intracranial pressure.
B. Initiate CPR and attach an AED as soon as possible.
You are caring for a conscious, confused patient with left-sided hemiparalysis. His airway is patent and his respirations are 22 breaths/min with adequate tidal volume. Treatment for this patient should include: A. Oxygen via nonrebreathing mask, left lateral recumbent position, and transport. B. Assisted ventilation with a BVM, right lateral recumbent position, and transport. C. An oral airway, assisted ventilation with a BVM, Fowler's position, and transport. D. Oxygen via nonrebreathing mask, supine position with legs elevated 6" to 12", and transport.
A. Oxygen via nonrebreathing mask, left lateral recumbent position, and transport.
Law enforcement has summoned you to a nightclub, where a 22-year-old female was found unconscious in an adjacent alley. Your initial assessment reveals that her respirations are rapid and shallow and her pulse is rapid and weak. She is wearing a medical alert bracelet that identifies her as an epileptic. There is an empty bottle of vodka next to the patient. You should: A. Place a bite block in her mouth in case she seizes again and transport at once. B. Apply oxygen via nonrebreathing mask, place her on her left side, and transport. C. Assist ventilations, perform a rapid exam, and prepare for immediate transport. D. Apply oxygen via nonrebreathing mask and transport her for a blood-alcohol test.
C. Assist ventilations, perform a rapid exam, and prepare for immediate transport.
You are caring for a 70-year-old female with signs and symptoms of an acute stroke. She is conscious, has secretions in her mouth, and is breathing at a normal rate with adequate depth. You should: A. Suction her oropharynx and apply 100% oxygen. B. Insert an oral airway, apply oxygen, and transport. C. Assist ventilations with a BVM and 100% oxygen. D. Place her on her side and prepare for rapid transport.
A. Suction her oropharynx and apply 100% oxygen.
A 58-year-old male presents with confusion, right-sided weakness, and slurred speech. As your partner is applying oxygen, it is MOST important for you to: A. List all of the patient's current medications. B. Obtain a complete set of baseline vital signs. C. Administer glucose to rule out hypoglycemia. D. Ask his wife when she noticed the symptoms.
D. Ask his wife when she noticed the symptoms.
You respond to a residence for a child who is having a seizure. Upon arrival at the scene, you enter the residence and find the mother holding her child, a 2-year-old male. The child is conscious and crying. According to the mother, the child had been running a high fever and then experienced a seizure that lasted approximately 3 minutes. You should: A. Cover the child with wet towels and give oxygen via nasal cannula. B. Transport the child to the hospital and reassure the mother en route. C. Advise the mother to take her child to the doctor the following day. D. Call medical control and request permission to give the child Tylenol.
B. Transport the child to the hospital and reassure the mother en route.
You arrive at a grocery store shortly after a 35-year-old male stopped seizing. Your assessment reveals that he is confused and incontinent of urine. The patient's girlfriend tells you that he has a history of seizures and takes Tegretol. When obtaining further medical history from the girlfriend, it is MOST important to: A. Determine if the patient is a known alcohol abuser. B. Obtain a description of how the seizure developed. C. Determine when he was last seen by his physician. D. Ask her how long the patient has been taking Tegretol.
B. Obtain a description of how the seizure developed.
You receive a call to a residence, where a neighbor has found the resident, a 40-year-old female, semiconscious on her living room floor. During your assessment, you discover a bottle of Dilantin on a nearby table. You should be MOST suspicious that this patient: A. Has experienced a stroke. B. Is being treated for depression. C. Is postictal following a seizure. D. Has a very low blood sugar level.
C. Is postictal following a seizure.
You are transporting a semiconscious patient to the hospital. En route, you note that the patient's mental status is not improving, despite 100% supplemental oxygen. You should suspect that this patient: A. Is hypoglycemic. B. Has a brain tumor. C. Is in a postictal state. D. Has experienced trauma.
A. Is hypoglycemic.
Which of the following most accurately describes the cause of an ischemic stoke? A. Acute atherosclerotic disease. B. Narrowing of a carotid artery. C. Rupture of a cerebral artery. D. Blockage of a cerebral artery.
D. Blockage of a cerebral artery.
Which of the following is not an assessment parameter included in the Cincinnati Stoke Scale? A. Memory. B. Arm drift. C. Facial droop. D. Speech.
A. Memory.
Which of the following conditions is not a common cause of seizures? A. Acute alcohol withdrawal B. Severe hypovolemia C. Acute hypoglycemia D. Poisoning or overdose
B. Severe Hypovolemia.
Which of the following conditions would most likely affect the entire brain? A. Blocked cerebral artery in the frontal lobe B. Respiratory failure or cardiopulmonary arrest C. Reduced blood supply to the left hemisphere D. Ruptured cerebral artery in the occipital lobe.
B. Respiratory failure or cardiopulmonary arrest.
Which of the following most accurately describes the postictal state that follows a seizure? A. Rapidly improving level of consciousness. B. Dazed, confused, or combative appearance. C. Hyperventilation and hypersalivation. D. Gradually decreasing level of consciousness.
B. Dazed, confused, or combative appearance.
Which of the following conditions would most likely mimic the signs and symptoms of a stroke? A. Alcohol Intoxication. B. Acute Poisoning. C. Hypoglycemia. D. High Fever.
C. Hypoglycemia.
Which of the following medications is not used to treat patients with a history of seizures? A. Tegretol. B. Dilantin. C. Dilaudid. D. Phenobarbital.
C. Dilaudid.
Interruption of cerebral blood flow may result from all of the following, except: A. A thrombus B. Cerebral vasodilation C. An acute arterial rupture D. An embolism
B. Cerebral Vasodilation
Which of the following most accurately describes a focal seizure? A. A seizure that begins in one extremity. B. Generalized seizures without incontinence. C. A seizure that is not preceded by an aura. D. A blank stare without muscle contractions.
A. A seizure that begins in one extremity.
Which of the following clinical signs would be most suggestive of a ruptured cerebral artery? A. Unilateral Hemiparesis B. Nasal discharge of blood C. Confusion and weakness D. Sudden, severe headache
D. Sudden, severe headache.
Which of the following conditions would be least likely to mimic the signs and symptoms of a stroke? A. Postictal State. B. Hypoglycemia. C. Intracranial Bleeding. D. Hypovolemia.
D. Hypovolemia.
Which of the following is a metabolic cause of a seizure? A. Brain Tumor. B. Massive Stroke. C. Poisoning. D. Head Trauma.
C. Poisoning.
Which of the following patients would most likely demonstrate typical signs of infection or fever? A. An 88-year-old male with chronic renal problems B. A 17-year-old male with depression and anxiety C. A 3-month-old female who was born prematurely D. A 35-year-old female in the later stages of AIDS
B. A 17-year-old male with depression and anxiety.
You arrive at a local grocery store approximately 5 minutes after a 21-year-old female stopped seizing. She is confused and disoriented; she keeps asking you what happened and tells you that she is thirsty. Her brother, who witnessed the seizure, tells you that she takes phenytoin (Dilantin) for her seizures, but has not taken it in a few days. He also tells you that she has diabetes. In addition to applying high-flow oxygen, you should: A. Administer one tube of oral glucose and prepare for immediate transport. B. Place her in the recovery position and transport her with lights and siren. C. Monitor her airway and breathing status and assess her blood glucose level. D. Give her small cups of water to drink and observe for further seizure activity.
C. Monitor her airway and breathing status and assess her blood glucose level.
When obtaining medical history information from the family of a suspected stroke patient, it is MOST important to determine: A. When the patient last appeared normal. B. If there is a family history of a stroke. C. If the patient has been hospitalized before. D. The patient's overall medication compliance.
A. When the patient last appeared normal.
Law enforcement has summoned you to a nightclub, where a 22-year-old female was found unconscious in an adjacent alley. your primary assessment reveals that her respirations are rapid and shallow and her pulse is rapid and weak. she is wearing a medical alert bracelet that identifies her as an epileptic. there is an empty bottle of vodka next to the patient. you should: A. Place a bite block in her mouth in case she has a seizure and transport at once. B. Apply oxygen via a nonrebreathing mask, place her on her left side, and transport. C. Assist ventilations, perform a rapid exam, and prepare for immediate transport. D. Apply oxygen via nonrebreathing mask and transport her for a blood-alcohol test.
C. Assist ventilations, perform a rapid exam, and prepare for immediate transport.
You are assessing a 49-year-old man who, according to his wife, experienced a sudden, severe headache and then passed out. He is unresponsive and has slow, irregular breathing. his blood pressure is 190/94 mm Hg and his pulse rate is 50 beats/min. his wife tells you that he has hypertension and diabetes. he has MOST likely experienced: A. Acute hypoglycemia. B. A ruptured cerebral artery. C. A complex partial seizure. D. An occluded cerebral artery.
B. A ruptured cerebral artery.